Sie sind auf Seite 1von 191

Paulo Almeida

Enide Andrade Martins


Soa Pinheiro
Maria Raquel Pinto
Roslia Rodrigues
Rita Simes

Apontamentos de
lgebra Linear

Departamento de Matemti a
Universidade de Aveiro
Janeiro de 2012

Contedo
1 Matrizes. Noes gerais

1.1

Denio. Algumas matrizes espe iais

. . . . . . . . . . . . . . .

1.2

Operaes om matrizes e suas propriedades . . . . . . . . . . . .

1.2.1

Adio de matrizes . . . . . . . . . . . . . . . . . . . . . .

1.2.2

Multipli ao por um es alar

1.2.3

Multipli ao de matrizes . . . . . . . . . . . . . . . . . .

1.2.4

Transposta de uma matriz . . . . . . . . . . . . . . . . . .

12

. . . . . . . . . . . . . . . .

2 Sistemas de equaes lineares

16

2.1

Sistemas e matrizes . . . . . . . . . . . . . . . . . . . . . . . . . .

17

2.2

Mtodo de eliminao de Gauss . . . . . . . . . . . . . . . . . . .

21

2.3

Dis usso de sistemas

. . . . . . . . . . . . . . . . . . . . . . . .

28

2.4

Sistemas homogneos . . . . . . . . . . . . . . . . . . . . . . . . .

31

3 Matrizes invertveis. Determinantes


3.1

3.2

35

Matrizes invertveis . . . . . . . . . . . . . . . . . . . . . . . . . .

36

3.1.1

Propriedades da inversa

. . . . . . . . . . . . . . . . . . .

37

3.1.2

Algoritmo de inverso

. . . . . . . . . . . . . . . . . . . .

39

Determinantes. Con eitos gerais . . . . . . . . . . . . . . . . . . .

42

3.2.1

Propriedades do determinante . . . . . . . . . . . . . . . .

46

3.2.2

Teorema de Lapla e

49

. . . . . . . . . . . . . . . . . . . . .

3.3

Condies de invertibilidade . . . . . . . . . . . . . . . . . . . . .

52

3.4

Cl ulo da inversa a partir da matriz adjunta . . . . . . . . . . .

55

3.5

Sistemas de Cramer

57

. . . . . . . . . . . . . . . . . . . . . . . . .

4 Espaos ve toriais sobre um orpo

60

4.1

Denio e propriedades . . . . . . . . . . . . . . . . . . . . . . .

4.2

Subespaos ve toriais . . . . . . . . . . . . . . . . . . . . . . . . .

61
66

4.3

Combinao linear de ve tores

. . . . . . . . . . . . . . . . . . .

69

4.4

Independn ia e dependn ia linear . . . . . . . . . . . . . . . . .

71

4.5

Subespao gerado por ve tores

. . . . . . . . . . . . . . . . . . .

78

4.6

Sistema de geradores . . . . . . . . . . . . . . . . . . . . . . . . .

80

4.7

Base e dimenso

. . . . . . . . . . . . . . . . . . . . . . . . . . .

81

4.8

Coordenadas de um ve tor relativamente a uma base . . . . . . .

90

CONTEDO
4.9

ii

Interse o, reunio e soma de subespaos . . . . . . . . . . . . .

4.10 Teorema das dimenses

. . . . . . . . . . . . . . . . . . . . . . .

4.11 Subespao omplementar

97

. . . . . . . . . . . . . . . . . . . . . . 103

5 Apli aes lineares


5.1

92

106

Denio e propriedades . . . . . . . . . . . . . . . . . . . . . . . 107


5.1.1

Classi ao de apli aes lineares . . . . . . . . . . . . . 111

5.1.2

Propriedades das apli aes lineares

. . . . . . . . . . . . 112

5.2

Imagem e imagem re pro a . . . . . . . . . . . . . . . . . . . . . 116

5.3

N leo e imagem . . . . . . . . . . . . . . . . . . . . . . . . . . . 118

5.4

Isomorsmos

5.5

Matriz de uma apli ao linear

. . . . . . . . . . . . . . . . . . . . . . . . . . . . . 126
. . . . . . . . . . . . . . . . . . . 128

L(E, E )

Mpn (K)

5.5.1

Isomorsmo entre

5.5.2

Matrizes invertveis e isomorsmos . . . . . . . . . . . . . 134

. . . . . . . . . . 134

5.6

Matriz de mudana de base

5.7

Relao entre matrizes de uma mesma apli ao linear . . . . . . 137

. . . . . . . . . . . . . . . . . . . . . 135

6 Valores e ve tores prprios

142

6.1

Valores e ve tores prprios . . . . . . . . . . . . . . . . . . . . . . 143

6.2

Endomorsmos diagonalizveis

. . . . . . . . . . . . . . . . . . . 151

7 Produto interno

163

7.1

Denio e exemplos . . . . . . . . . . . . . . . . . . . . . . . . . 164

7.2

Norma de um ve tor . . . . . . . . . . . . . . . . . . . . . . . . . 166

7.3

ngulo entre ve tores

7.4

Ve tores ortogonais . . . . . . . . . . . . . . . . . . . . . . . . . . 173

7.5

Sistema ortogonal e sistema ortonormado

7.6

Base ortogonal e base ortonormada . . . . . . . . . . . . . . . . . 175


7.6.1

. . . . . . . . . . . . . . . . . . . . . . . . 169
. . . . . . . . . . . . . 174

Mtodo de ortonormalizao de Gram-S hmidt . . . . . . 176

7.7

Matriz da mtri a

7.8

Complemento ortogonal e proje es ortogonais . . . . . . . . . . 182

. . . . . . . . . . . . . . . . . . . . . . . . . . 179

7.9

Subespao ortogonal de um subespao ve torial . . . . . . . . . . 184

7.10 Distn ia entre ve tores

. . . . . . . . . . . . . . . . . . . . . . . 186

1. Matrizes. Noes gerais

1.1 Denio. Algumas matrizes espe iais

1.1 Denio. Algumas matrizes espe iais


Suponha-se que se est a trabalhar em estruturas algbri as onhe idas, omo
por exemplo,
Considere

C, R

ou

om as operaes usuais de adio e de multipli ao.

um desses onjuntos. Aos elementos de

Denio 1.1.

Uma

hamam-se es alares.

matriz do tipo (ou de tamanho) p q sobre K uma

tabela de dupla entrada om

linhas e

olunas ujas entradas perten em a

K.

Em termos de notao representam-se matrizes por letras mais ulas e usa-se


a tabela de nmeros dentro de parnteses re tos omo indi ado a seguir:

Os es alares

aij ,
A.

elementos ) de

om

a11
a21

A= .
..
ap1

e na oluna

Exemplo 1.2.

aij

om

(ou entrada
Seja

ap2
e

j {1, . . . , q}

dizem-se entradas (ou

i {1, . . . , p}

j {1, . . . , q}.

representa a entrada da matriz

A que se en ontra na linha


(i, j).

e usual referir omo sendo a entrada (ou o elemento)

A matriz

3 2 pois
(3, 1)) 6.

uma matriz do tipo

a31

.
.
.

Em termos gerais, tambm se es reve

 
A = aij ,

O termo genri o

.
.
.

i {1, . . . , p}

a1q
a2q

.
.
.

.
apq

a12
a22

uma matriz do tipo

quadrada de ordem

p.

1 4
A = 2 9
6 5

omposta por

p q.

Quando

linhas e

p=q

olunas. A entrada

diz-se que

uma matriz

Quando se tem uma matriz om uma s oluna (linha)

hama-se matriz oluna (matriz linha ).

Exemplo 1.3.

Considere as matrizes

A=
A matriz


2 3
,
3 1

B=

quadrada de ordem

 
2
3

2, B



C= 2 3 .

uma matriz oluna e

uma matriz

linha.

 
A = aij , om i, j {1, . . . , p}, uma matriz quadrada. As entradas aij
om i = j , isto , as entradas da forma aii , formam a diagonal prin ipal de A.
Os elementos aij e aji , om i 6= j , esto dispostos simetri amente em relao
Seja

diagonal prin ipal, e por isso dizem-se opostos.

1.1 Denio. Algumas matrizes espe iais


Exemplo 1.4.

Considere a matriz

2
A=0
2

Chama-se

0
4
0

3
1
5
2, 1

Os elementos da diagonal prin ipal so


As entradas mar adas por

Denio 1.5.

e esto assinalados por

matriz diagonal a uma matriz quadrada em que os

elementos que no so da diagonal prin ipal so iguais a zero, ou seja,


para

so um exemplo de elementos opostos.

aij = 0

i 6= j .

Exemplo 1.6.

A matriz

A=
uma matriz diagonal.

Denio 1.7.

2 0
0 1

matriz es alar a

Chama-se

uma matriz diagonal em que os

elementos da diagonal prin ipal so todos iguais entre si.

Exemplo 1.8.

As matrizes

A=

2 0
0 2

3 0
0
B = 0 3 0
0
0 3

so matrizes es alares.

Um aso espe ial de uma matriz es alar a matriz em que todos os elementos
da diagonal prin ipal so iguais a

1.

Assim, a matriz identidade de ordem

Essa matriz hama-se matriz identidade.

n,

representa-se por

1 0
0 1

In = . .
.. ..
0 0

..

In ,

e a matriz

0
0

. .
.
.
1

Se a sua ordem for depreendida do ontexto, representa-se simplesmente por

I.

Observe-se que

 
In = ij ,

sendo

ij =

A matriz nula do tipo

pq

ij

o smbolo de Krone ker, ou seja,

1
0

se
se

i=j
.
i 6= j

uma matriz em que todas as suas entradas

so iguais a zero e representa-se por

0pq .

Por vezes representa-se apenas por

(zero), quando no ontexto est subentendido o tipo da matriz.

1.2 Operaes om matrizes e suas propriedades


Denio 1.9.

Uma matriz quadrada diz-se matriz

aij = 0

i > j,

quando

triangular superior

nulos. Analogamente, uma matriz quadrada diz-se matriz


se

aij = 0

quando

se

isto , os elementos abaixo da diagonal prin ipal so

i < j,

triangular inferior

isto , os elementos a ima da diagonal prin ipal so

nulos.

Exemplo 1.10.

A matriz

Sejam

3
A = 0
0

4
2
0

1
2

0
1

2
B= 7
4

triangular superior e

Denio 1.11.

0
0
3
2

0
0 .
1

uma matriz triangular inferior.

 
 
A = aij e B = bij do tipo p q
i {1, . . . , p} e j {1, . . . , q}.

Duas matrizes

iguais se aij = bij , para todo

dizem-se

1.2 Operaes om matrizes e suas propriedades


1.2.1 Adio de matrizes
Seja

Mpq (K)

o onjunto das matrizes do tipo

pq

om elementos em

K.

A adio de matrizes uma apli ao denida no produto artesiano

Mpq (K) Mpq (K)1


que, a ada par de matrizes
de

Mpq (K)

(A, B),

geralmente denotada por

operao interna em

Mpq (K).

faz orresponder uma e uma s matriz

A + B.

Tambm se diz que esta uma

Em termos de representao das entradas da

matriz que resulta da adio de duas matrizes quaisquer apresenta-se a seguinte


denio.

Denio 1.12.

matriz soma

 
 
A, B Mpq (K) tais que A = aij , B = bij .
a matriz de Mpq (K) denida por:
 
A + B = cij ,

Sejam

A+B

om cij = aij +bij , para todo i {1, . . . , p} e




simplesmente A + B = aij + bij .

Exemplo 1.13.

Sejam

A=
Tem-se que

1
4

2 3
5 0

j {1, . . . , q}. Por vezes es reve-se

B=

6 1
7
8

2
9

 

1 + 6 2 + (1) 3 + 2
7 3 5
A+B =
=
.
4+7
5+8
0+9
11 13 9

1M

pq (K)

Mpq (K) = {(A, B) : A, B Mpq (K)}

1.2 Operaes om matrizes e suas propriedades

Propriedades da adio de matrizes


Apresentam-se agora algumas propriedades da adio de matrizes. Ir-se- provar
algumas destas propriedades e as restantes demonstraes so deixadas omo
exer io.
Sejam

A, B, C Mpq (K)

matrizes quaisquer.

Comutatividade: A + B = B + A.
Demonstrao. Sejam

j {1, . . . , q}.

A =

Tem-se:



A + B = aij + bij


= bij + aij

aij

B =

bij

, om

i {1, . . . , p}

por denio de adio de matrizes


pela omutatividade em

=B+A

Asso iatividade: (A + B) + C = A + (B + C).


Demonstrao. Sejam

 
 
 
A = aij , B = bij e C = cij , om i {1, . . . , p}

j {1, . . . , q}. Tem-se:


  

(A + B) + C = aij + bij + cij


= (aij + bij ) + cij


= aij + (bij + cij )

  
= aij + bij + cij
   
 
= aij + bij + cij
e

por denio de adio de matrizes


por denio de adio de matrizes
pela asso iatividade em

por denio de adio de matrizes


por denio de adio de matrizes

= A + (B + C).

Existn ia de elemento neutro: 0pq + A = A.

Existn ia
simtri o:
A + (A) = 0pq ,

  de elemento

A = aij ento A = aij .

onde, sendo

As demonstraes da existn ia do elemento neutro e simtri o  am omo

exer io.
Uma vez que so vlidas estas quatro propriedades, diz-se que

Mpq (K)

munido da adio de matrizes um grupo abeliano (ou omutativo).

Observao 1.14.
A + (B)

por

Dadas duas matrizes

A B.

A, B Mpq (K),

denota-se a matriz

1.2 Operaes om matrizes e suas propriedades


Exer io 1.15.

Considere as matrizes

Cal ule

4
9 10
A = 5 5 0
9
1 1

4 1 3
1
5 .
B= 9
2
0 1

A B.

1.2.2 Multipli ao por um es alar


Pode tambm denir-se uma operao externa entre o onjunto das matrizes
e o onjunto

K.

A multipli ao por um es alar uma apli ao denida no

K Mpq (K) que a ada par (, A) faz orresponder uma e


Mpq (K) geralmente denotada por A.
 
Denio 1.16. Seja A Mpq (K) tal que A = aij e seja K um es alar.
A matriz A a matriz do tipo p q que se obtm de A multipli ando todas
as entradas de A pelo es alar , ou seja:
 
A = cij ,
produto artesiano

uma s matriz de

om

cij = aij ,

para todo

Exemplo 1.17.

Seja

i {1, . . . , p}

A=

2 1
3 1

Cal ule

A=

2A 3B ,

4 5
1 0

j {1, . . . , q}.

, ento

2A =

Exer io 1.18.

4 2
6 2

sabendo que
e

B=

1 2
3 1

Propriedades da multipli ao por um es alar


Sejam
quer.

es alares quaisquer de

e sejam

A, B Mpq (K)

matrizes quais-

Distributividade da multipli ao por um es alar em relao


adio de matrizes: (A + B) = A + B .

1.2 Operaes om matrizes e suas propriedades


Demonstrao. Represente-se

j {1, . . . , q}.

Tem-se que:



(A + B) = aij + bij


= (aij + bij )


= aij + bij



= aij + bij
 
 
= aij + bij


 
A = aij

 
B = bij ,

om

i {1, . . . , p}

por denio de adio de matrizes


por denio de multipli ao por um es alar
pela distributividade da multipli ao em
relao adio em

por denio de adio de matrizes


por denio de multipli ao por um es alar

= A + B.

Distributividade da multipli ao por uma matriz em relao


adio de es alares: ( + )A = A + A.
Demonstrao. Represente-se A
Tem-se que:

 
( + )A = ( + ) aij


= ( + )aij


= aij + aij



= aij + aij
 
 
= aij + aij


 
= aij , om i {1, . . . , p} e j {1, . . . , q}.

por denio de multipli ao por um es alar


pela distributividade da multipli ao em
relao adio em

pela denio de adio de matrizes


por denio de multipli ao por um es alar

= A + A

Asso iatividade mista: (A) = ()A.

Existn ia de Elemento Neutro: 1K A = A, onde 1K o elemento


neutro da multipli ao em K (note-se que em R, Q ou C, 1K 1).
As demonstraes das Propriedades 3 e 4  am omo exer io.

Exer io 1.19.

1 3 tais


e
B = 2 3 1 .


1
A+B .
5(A + B) + 2
2

Considere as matrizes

2A = 2 3


4

Cal ule, apli ando as propriedades,

do tipo

que:

Prova-se tambm que vlida um tipo de lei do anulamento na multipli ao de uma matriz por um es alar.

1.2 Operaes om matrizes e suas propriedades


Teorema 1.20.

Sejam

A Mpq (K)

A = 0pq
onde

0K

K.

Ento

= 0K

se e s se

o elemento neutro da adio usual em

ou

A = 0pq ,
(ou seja, em

R, Q

ou

0).
Demonstrao. Seja
Ento

A = 0pq

 
A = aij ,

om

equivalente a


aij = 0pq

i {1, . . . , p}

j {1, . . . , q}.
por denio de
multipli ao es alar

aij = 0, i {1, . . . , p}, j {1, . . . , q}

por denio de

= 0K aij = 0, i {1, . . . , p}, j {1, . . . , q}

pela lei do anulamento

igualdade de matrizes

do produto em

= 0K A = aij = 0pq

1.2.3 Multipli ao de matrizes


Dadas duas matrizes

B,

a multipli ao

AB

s possvel se o nmero de

olunas da primeira matriz oin ide om o nmero de linhas da segunda matriz.


A matrizes que satisfazem esta relao hamam-se matrizes en adeadas. Assim,

B , se queremos efe tuar a multipli ao A B e se


p q ento B tem de ser uma matriz do tipo q m.
Nesse aso a matriz resultante, que se representa por AB , uma matriz do tipo
p m. Esquemati amente

dadas duas matrizes

uma matriz do tipo

A |{z}
B = |{z}
AB .
|{z}
pq

Sejam
que

AB

uma matriz do tipo

qm

pq

est denido. Relativamente a

BA

pm

B uma matriz do tipo q m. Note-se


BA, trs hipteses podero o orrer:

poder no estar denido; isso a onte e se

m 6= p;

BA est denido (isto , p = m) e BA ser uma matriz do tipo q q e AB


ser uma matriz do tipo m m; e neste aso podem surgir duas situaes:

se q 6= m, AB
AB 6= BA;
se

q = m, AB

BA

BA

so de tipos diferentes e, onsequentemente,

so do mesmo tipo mas podero ser diferentes.

1.2 Operaes om matrizes e suas propriedades


Exemplo 1.21.
C

10

A uma matriz do tipo 2 3, B uma matriz do


3 2 e D e E matrizes do tipo 2 2. Ento:

Sejam

uma matriz do tipo

AB

do tipo

24

BA

no est denida;

AC

do tipo

22

CA

do tipo

DE

do tipo

22

ED

do tipo

tipo

3 4,

3 3;
2 2.

Veja-se ento omo se multipli am matrizes. Considere-se primeiro o aso


parti ular do produto de uma matriz linha por uma matriz oluna.

Denio 1.22.

Sejam

A =

Observe-se que se
ento

AB

a1

a2

ap

Ento o

A pela matriz oluna B :




AB = a1 b1 + a2 b2 + + ap bp .

produto da matriz linha

p 1,

b1
b2

B = . .
..
bp

uma matriz do tipo

uma matriz do tipo

1 1.

1p

uma matriz do tipo

5
Exemplo 1.23. Sejam A = 1 0 2 e B = 3 . Ento
4

  
AB = 1 5 + 0 3 + (2) 4 = 3 .


Agora dene-se o produto entre duas matrizes en adeadas quaisquer tendo

por base a denio do aso parti ular anterior.

Denio 1.24.
O

Sejam

matrizes do tipo

se obtm onsiderando para elemento

pq

q m,

respe tivamente.

produto de A por B , que se representa por AB , a matriz do tipo pm que


pela oluna

da matriz

(i, j)

a multipli ao da linha

da matriz

B.

Formalmente tem-se que, para

 
A = aik

 
B = bkj , om i {1, . . . , p},
A por B a matriz AB do tipo

k {1, . . . , q} e j {1, . . . , m}, o produto de


p m denida por:
 
AB = cij , om i {1, . . . , p}

j {1, . . . , m}

onde

cij = ai1 b1j + ai2 b2j + + aiq bqj =

q
X

k=1

aik bkj .

1.2 Operaes om matrizes e suas propriedades

11

Esquemati amente

ai1

ai2

Exemplo 1.25.

Sejam

bqj
{z
B

A= 0
1

3
1
4

B=

.
.
.

}
|

.
.
.

b1j
b2j

aiq

}|

{z

.
.
.

cij
{z

AB

5 0
1 2

. Ento


2 5 + 3 (1)
20+32
7
AB = 0 5 + (1) (1) 0 0 + (1) 2 = 1
1 5 + 4 (1)
10+42
1

Exer io 1.26.

Cal ule, se possvel, o produto

A=

1
4

5 7
0 2

AB ,

6
2 .
8

sabendo que

3 0
B = 1 2 .
1 4

Observe-se que dada uma matriz A do tipo pq , om p 6= q , no est denido


A2 = AA. Fa ilmente se on lui que s se pode denir potn ia de

o produto

uma matriz para matrizes quadradas. De uma forma geral, se A uma matriz
k
quadrada de ordem n, A , om k 1, representa a matriz quadrada de ordem

denida por:

Ak = AA
A} .
| {z
k fa tores

Por onveno,

A = In .

A2 = 0pp e, no entanto, A 6= 0pp .




0 1
, tem-se que
matriz A =
0 0


0 0
A2 =
= 022 .
0 0

Observe-se que pode ter-se

Exemplo 1.27.

Dada a

Pode ento on luir-se que no vlida a lei do anulamento do produto no


onjunto

Mnn (K).

Exer io 1.28.

Sejam

A=

1 0
1 0

B=

0
1

0
1

. Cal ule

AB .

1.2 Operaes om matrizes e suas propriedades

12

Propriedades da multipli ao de matrizes


Sejam

um es alar e

A, B, C

matrizes quaisquer om tamanhos adequados.

Existn ia de elemento neutro:


e AIq = A;

A for do tipo p q , ento Ip A = A

Se

Asso iatividade: A(BC) = (AB)C ;

Asso iatividade mista: (AB) = (A)B = A(B);

Distributividade da multipli ao em relao adio: A(B +


C) = AB + AC e (A + B)C = AC + BC .
As demonstraes das propriedades enun iadas  am omo exer io.

Observao 1.29.

Re orde-se que a multipli ao de matrizes no omutativa

e, onsequentemente, multipli ar direita ou esquerda por uma matriz (no


nula) no a mesma oisa!

Exemplo 1.30.

Considere as matrizes

1
A=
3


2
4

0
B=
1


2
4


1
3

BA =

Ento

AB =


1
.
0
3
1


4
.
2

1.2.4 Transposta de uma matriz


 
A = aij

uma matriz do tipo p q . Chama-se transposta da matriz A, e representa-se por AT , matriz do tipo q p tal que

Denio 1.31.

om

aji = aij ,

Seja

para todo

j {1, . . . , q}

Assim, se

ento

 
AT = aji .

a11
a21

A= .
..
ap1

a11
a12

AT = .
..
a1q

Ou seja, as linhas da matriz

AT

a12
a22
.
.
.

i {1, . . . , p}.

..

ap2
a21
a22
.
.
.

a2q

a1q
a2q

. ,
.
.
apq

..

ap1
ap2

. .
.
.
apq

so as olunas da matriz

A.

1.2 Operaes om matrizes e suas propriedades


Exemplo 1.32.

Seja

A=


2
5

3 4
6 7

. A transposta de

2
AT = 3
4

13

a seguinte matriz:

5
6 .
7

Propriedades da transposta de uma matriz


Seja

um es alar e sejam A, B

matrizes quaisquer om os tamanhos adequados.

Transposta da transposta:


AT


T

= A.

Demonstrao. Seja A = aij , om i


 
T
= aji , om aji =
Por denio, A

i {1, . . . , p}.

Assim,

AT
aij = aji = aij ,

onde


T T

T

 T  
= aji = aij

para todo

= A.

{1, . . . , p} e j {1, . . . , q}.


aij , para todo j {1, . . . , q} e

i {1, . . . , p}

j {1, . . . , q}.

Logo

Transposta do produto de uma matriz por um es alar: (A)T =


AT .
 
Demonstrao. Seja A = aij , om i {1, . . . , p} e j {1, . . . , q}. Ora,



sendo A = aji , om aji = aij , para todo j {1, . . . , q} e i {1, . . . , p},


ento


 
T 

(A)T = aij = aji = aji = AT .

Transposta da soma de duas matrizes: (A + B)T = AT + B T .


 
 
Demonstrao. Sejam A = aij e B = bij , om i {1, . . . , p} e
 
j {1, . . . , q}. Por denio de adio de matrizes, A + B = cij , om
cij = aij + bij , para todo i {1, . . . , p} e j {1, . . . , q}. Assim,
 
(A + B)T = cji
cji = cij ,

j {1, . . . , q} e i {1, . . . , p}.


 
 
T
Por outro lado, omo A
= aji , om aji = aij , e B T = bji , om








bji = bij , vem que AT + B T = aji + bji = aji + bji = dji , onde a
entrada (j, i)
om

para todo

dji = aji + bji = aij + bij = cij = cji .

E, portanto,

AT + B T = (A + B)T .

1.2 Operaes om matrizes e suas propriedades

14

Transposta do produto de duas matrizes: (AB)T = B T AT .


 
 
A = aik e B = bkj ,
j {1, . . . , m}. Tem-se que

Demonstrao. Sejam

k {1, . . . , q}

AB = cij ,
para ada

i {1, . . . , p}

ail blj

l=1

 
 
AT = aki , om aki = aik , e B T = bjk , om bjk = bkj ,
i {1, . . . , p}, k {1, . . . , q} e j {1, . . . , m}. Assim,
 
B T AT = dji ,
onde a entrada (j, i)
dji =

q
X

bjl ali

(AB) = B A

Denio 1.33.

q
X

blj ail =

q
X

ail blj = cij = cji .

l=1

l=1

l=1

Logo

cij =

q
X

i {1, . . . , p},

j {1, . . . , m}. Assim


 
(AB)T = cji , onde cji = cij .

Por outro lado,


para todo

onde

om

Uma matriz

diz-se

simtri a se A = AT .
A seja uma matriz
A de ordem p simtri a se existir

Observe-se que esta denio obriga a que a matriz


quadrada. Alm disso, uma matriz quadrada

simetria relativamente diagonal prin ipal, isto , se da forma

 
A = aij ,

Exemplo 1.34.

A matriz

Denio 1.35.

om

aij = aji , i, j {1, . . . , p}.

1 2
2 3

Uma matriz

simtri a.

diz-se

Assim, a denio obriga a que

anti-simtri a se AT

= A.

A, para ser anti-simtri a, seja quadrada e os

elementos da sua diagonal prin ipal sejam todos nulos. Alm disso, em posies
opostas em relao diagonal prin ipal, esto elementos simtri os entre si.

Exemplo 1.36.

A matriz

Exer ios 1.37.


a)

A + AT

0
3
4

1. Seja

simtri a;

3 4
0 1
1 0

anti-simtri a.

uma matriz quadrada. Prove que:

1.2 Operaes om matrizes e suas propriedades


b)

A AT

15

anti-simtri a.

2. Mostre que qualquer matriz quadrada se pode de ompor na soma de uma


matriz simtri a om uma matriz anti-simtri a.
3. Em ada aso, prove que a armao verdadeira ou apresente um ontra-exemplo mostrando que falsa. Sejam

A, B

matrizes de tamanhos

adequados.
a) Se

A+B =A+C

b) Se

A + B = 0,

) Se a entrada

7.

d) Se

A = A,

ento

(2, 3)
ento

e) Para toda a matriz


f ) A igualdade

ento

so do mesmo tipo.

B = 0.

da matriz

7,

ento a entrada

(3, 2)

de

AT

A = 0.
A,

as matrizes

(A+B) = A +2AB+B

AT

tm a mesma diagonal.

sempre vlida para quaisquer

matrizes.
g) Se

A2 = A

ento

A=0

ou

A = I.

A Mpq (K) e B, C Mqm (K) matrizes quaisquer. Apli ando


as propriedades das operaes entre matrizes, mostre, de duas formas dis-

4. Sejam

tintas, que

(A(B + C)) = B T AT + C T AT .

2. Sistemas de equaes
lineares

2.1 Sistemas e matrizes

17

2.1 Sistemas e matrizes


Nesta se o apresentam-se algumas denies e nomen latura bsi as asso iadas aos sistemas de equaes lineares e a sua relao om as matrizes.

Denio 2.1.

Uma equao da forma

a1 x1 + a2 x2 + + an xn = b,

(2.1)

equao linear
oe iente

ai K, para ada i {1, . . . , n} e b K, hamada uma


nas in gnitas (ou indeterminadas) x1 , . . . , xn . A ada ai hama-se

onde

da equao e ao

hama-se

termo independente da equao.

Exemplo 2.2.

A equao x1 + 4x2 7x3 = 11 uma equao linear nas


x1 , x2 e x3 de oe ientes 1, 4 e 7 e termo independente 11.
equao 4x1 5x2 = x1 x3 no uma equao linear.

in gnitas
A

Re ordando o produto de uma matriz linha por uma matriz oluna, note-se
que a equao (2.1) pode ser representada matri ialmente por

Denio 2.3.

s1

s2

a1

a2

Diz-se que o
T
sn ,

x1


x2  
an . = b .
..
xn

nuplo (s1 , s2 , . . . , sn ),

soluo da equao

(2.1) (ou de (2.2)) se

a1

a2

s1


s2  
an . = b
..
sn

Ao onjunto de todas as solues de (2.1) hama-se

Exemplo 2.4.

onjunto soluo de (2.1).

3x1 x2 +4x3 = 5. Esta pode representar-se



 

 x1
3 1 4 x2 = 5 .
x3

Considere a equao

matri ialmente omo

dada

ou de forma equivalente,

a1 s1 + a2 s2 + + an sn = b

ou

Como

(2.2)

3x1 x2 + 4x3 = 5 x2 = 3x1 + 4x3 5,

o onjunto soluo da equao

S = {(x1 , 3x1 + 4x3 5, x3 ) : x1 , x3 R}.


Algumas solues so, por exemplo,

(0, 5, 0), (1, 2, 0)

(0, 1, 1).

2.1 Sistemas e matrizes

18

A uma  ole o de um nmero nito de equaes lineares hama-se sistema


de equaes lineares. Em seguida apresenta-se a denio formal:

Denio 2.5.

m, n N,

onjuno de

hama-se

por

equaes lineares em

sistema de equaes lineares

a11 x1

a21 x1

am1 x1

++
++

in gnitas, om

e pode ser representado

a1n xn = b1
a2n xn = b2

(2.3)

.
.
.

+ + amn xn = bm

oe ientes
termos independentes do

aij K, om i {1, . . . , m} e j {1, . . . , n}, so hamados


do sistema, os bi K, om i {1, . . . , m}, so os

onde

sistema e
Se

x1 , . . . , xn

so as in gnitas do sistema.

bi = 0, para todo i {1, . . . , m}, ento diz-se que o sistema homogneo ;


bi 6= 0, para algum i {1, . . . , m} ento o sistema diz-se

aso ontrrio, isto , se

ompleto.

Exemplo 2.6.
(S1 )

Considere os seguintes sistemas lineares

2 x1 + 3 x2 + 4 x3 = 1

1 x1 + 5 x2 + 0 x3 = 2

(S2 )

1 x1 2 x2 = 0

1 x1 + 3 x2 = 0

O sistema (S1 ) um sistema ompleto om 2 equaes e 3 in gnitas e (S2 )


um sistema homogneo om 2 equaes e 2 in gnitas. Em ambos os sistemas
os termos independentes esto mar ados por

e os oe ientes por

Atendendo denio de soluo de uma equao linear pode denir-se


soluo de um sistema de equaes lineares.

soluo

Denio 2.7.

O nuplo (s1 , s2 , . . . , sn )
do sistema de equaes
lineares na forma (2.3) se for soluo de todas as equaes que onstituem esse

sistema. Ao onjunto de todas as solues de (2.3) hama-se

onjunto soluo

desse sistema.

Exemplo
 2.8.
2, 0, 45

Considere o sistema linear (S1 ) do exemplo anterior, o terno


uma soluo desse sistema; de fa to,

2 (2) + 3 0 + 4 54 = 1
.
1 (2) + 5 0 + 0 45 = 2

Para a determinao do onjunto soluo de um sistema, podem apli ar-se


diversos mtodos que permitem obter um sistema equivalente mais simples.

Denio 2.9.
soluo.

Dois sistemas so

equivalentes se tiverem o mesmo onjunto

2.1 Sistemas e matrizes

19

Um dos mtodos para determinar o onjunto soluo de um sistema onsiste


em apli ar determinadas operaes sobre as equaes do sistema. Essas operaes so hamadas operaes elementares sobre equaes. Represente-se por
om

i {1, . . . , m},

i-sima

ei ,

equao de um sistema de equaes lineares na

forma (2.3). As operaes elementares sobre equaes so:


I. tro ar duas equaes
(representa-se por

ei ej );

II. multipli ar uma equao por um es alar no nulo

(representa-se por ei := ei , om 6= 0);


III. adi ionar a uma equao outra multipli ada por um es alar

(representa-se por ei := ei + ej , om K e i 6= j ).

Exer io 2.10.
nas equaes

Mostre que se se efe tuar operaes elementares sobre equaes

e1 , . . . , en

de um sistema de equaes lineares obtm-se um sistema

equivalente.
Veja-se um exemplo:

Exemplo 2.11.

Dado o sistema

tares a ima des ritas, obtm-se:

x+y =1
xy =2

|{z}

e2 :=e2 +e1

|{z}

e1 :=e1 e2

O onjunto soluo

1
3
2, 2

x+y =1
xy =2

x+y =1
2x = 3

y = 1
x = 23

3
2

, efe tuando operaes elemen-

|{z}

e2 := 21 e2

x+y =1
x = 32

= 21

Atendendo ao nmero de solues que um sistema de equaes lineares admite, este pode ser lassi ado da seguinte forma:

impossvel: quando
possvel:

no tem soluo;

quando admite uma ou mais solues; neste aso pode dizer-se

que :
-

possvel e determinado: quando tem apenas uma ni a soluo;


possvel e indeterminado: quando tem uma innidade de solues;
neste aso atribui-se ainda um grau de indeterminao ao sistema que
o nmero de variveis livres do sistema.

2.1 Sistemas e matrizes

20

Vejam-se alguns exemplos.

Exemplos 2.12.

1. O sistema do Exemplo 2.11 possvel e determinado.

2. O sistema

x1 x2 + x3 = 1
x1 + 2x2 x3 = 2

|{z}

e2 :=e2 +e1

|{z}

e1 :=e1 +e2

x1 x2 + x3 = 1
x2 = 3
x1 + x3 = 4
x2 = 3

{(4 x3 , 3, x3 ) : x3 R}. Logo trata-se de


um sistema possvel e indeterminado, om grau de indeterminao igual

tem o seguinte onjunto soluo


a

(neste aso, a ni a varivel livre

x3 ).

3. O sistema

x1 2x2 = 1
x1 + 2x2 = 3

|{z}

e2 :=e2 +e1

x1 2x2 = 1
0=2

impossvel. O seu onjunto soluo o onjunto vazio

A informao de um sistema de equaes lineares pode ser resumida numa


ni a matriz. Re ordando o produto entre matrizes, note-se que possvel representar o sistema de equaes lineares da forma (2.3) na seguinte forma matri ial:

a11
a21

AX = B .
..
am1
|

onde matriz

a12
a22
.
.
.

..

am2
{z
A


a1n
x1
b1
x2 b2
a2n

. . = .
. .
..
.
.
amn
xn
bm
} | {z } | {z }
X

A Mmn (K) hama-se matriz dos oe ientes


X Mn1 (K) hama-se matriz das in gnitas

simples), matriz

B Mm1 (K)

(ou matriz
e matriz

hama-se matriz dos termos independentes.

Pode ainda es rever-se uma ni a matriz om os oe ientes e os termos

independentes do sistema:

a11

a
 21
B = .
..
am1

a12
a22
.
.
.

am2

..

a1n
a2n
.
.
.

amn

b1
b2

. .
.
.
bm

A esta matriz hama-se matriz ampliada do sistema (2.3).

2.2 Mtodo de eliminao de Gauss


Exemplo 2.13.

21

Considere o sistema de equaes lineares

x1 + 2x2 x3 = 0
2x2 8x3 = 7

4x1 + 5x2 + 9x3 = 9

AX = B , om


1 2 1
x1
A = 0 2 8 , X = x2
4 5
9
x3

A sua forma matri ial

e a sua matriz ampliada

A B

1 2
= 0 2
4 5

0
B= 7
9

1
0
8
7 .
9 9

2.2 Mtodo de eliminao de Gauss


Re orde-se que nos exemplos anteriores usaram-se determinadas operaes elementares sobre equaes. Efe tuar operaes elementares sobre as equaes de
um sistema equivalente a efe tuar operaes elementares sobre as linhas da
matriz ampliada asso iada ao sistema. Represente-se por
a

i-sima

Li , om i {1, . . . , m},

linha da matriz ampliada. Veja-se um exemplo de omo resolver um

sistema usando a respe tiva matriz ampliada.

Exemplo 2.14.

Sistema

2x1 x2 = 4
x1 + x2 3x3 = 3

4x1 + 2x2 + 3x3 = 7

1o passo: e1 e2

x1 + x2 3x3 = 3
2x1 x2 = 4

4x1 + 2x2 + 3x3 = 7

Matriz ampliada

2 1
0 4
1
1 3
3
4
2
3
7

L1 L2

1
1 3
3
2 1
0 4
4
2
3
7

2.2 Mtodo de eliminao de Gauss

22

2o passo: e2 := e2 + 2e1
e3

L2 := L2 + 2L1
L3 := L3 4L1

:= e3 4e1

x1 + x2 3x3 = 3
x2 6x3 = 2

2x2 + 15x3 = 5

1
0
0

3o passo: e3 := e3 + 2e2

1 3
3
1 6
2
2 15 5

L3 := L3 + 2L2

x1 + x2 3x3 = 3
x2 6x3 = 2

3x3 = 1

1
0
0

4o passo: e3 := 13 e3

1
1
0

3
3
6
2
3 1

L3 := 31 L3

x1 + x2 3x3 = 3
x2 6x3 = 2

x3 = 31

1
0
0

1
1
0

3
3
6
2
1
1 3

Assim, obtm-se um sistema equivalente ao ini ial, que se resolve fa ilmente por
substituio,


x1 + x2 3x3 = 3
x1 = 3 0 + 3  31 = 2
x2 6x3 = 2
x2 = 2 + 6 31 = 0

x3 = 31
x3 = 31

E o onjunto soluo do sistema

2, 0, 31

mais f il trabalhar om a matriz ampliada. Pode-se ento resolver um sistema

AX = B

onsiderando a sua matriz ampliada

A B

e exe utando nela

de forma riteriosa operaes sobre as linhas. Este tipo de operaes hamamse operaes elementares sobre as linhas e so orrespondentes s operaes
elementares sobre equaes:
I. tro ar linhas
(representa-se por

Li Lj ,

om

i 6= j );

II. multipli ar uma linha por um es alar no nulo

(representa-se por Li := Li , om 6= 0);


III. adi ionar a uma linha um mltiplo de uma outra linha

(representa-se por Li := Li + Lj , om K e i 6= j ).

2.2 Mtodo de eliminao de Gauss


Exemplo 2.15.

Resolva-se o sistema

anteriormente.

3
1

1 1
1 5

23

3x y = 1
x+y =5

, usando a t ni a des rita





1
1 5
1

L1 L2
L2 := L2 3L1
3 1 1
0



1 1 5
L2 := 41 L2
0 1 72

1
5
4 14

Logo

3x y = 1

x + yy = 5

e o seu onjunto soluo

x+y =5

y = 72
 3 7 
.
2, 2

O que se fez foi um aso parti ular do

x=5
y = 27

7
2

107
4

3
2

mtodo de eliminao de Gauss

para a resoluo de sistemas de equaes lineares. Antes de formalizar este


mtodo apresentam-se algumas denies ne essrias.

Denio 2.16.

Diz-se que uma matriz est na

nhas se satiszer as seguintes ondies:

forma es alonada por li-

se h linhas nulas elas situam-se abaixo das linhas no nulas;

o primeiro elemento no nulo de ada linha ( om ex epo da primeira)


situa-se direita do primeiro elemento no nulo da linha anterior.

Aos primeiros elementos no nulos de ada linha da forma es alonada por


linhas hamam-se pivots.
Observe-se que, atendendo denio, os elementos que se situam na mesma
oluna e abaixo de um pivot na forma es alonada por linhas so todos nulos.

Exemplo 2.17.

1
A= 0
0

Considere as matrizes

2 1
3
1
0 1

As matrizes

Denio 2.18.

4
2
1

1 0
B= 0 1
0 0

4
0
0 1
0
0

0 0
C= 1 0
0 0

esto na forma es alonada; mas a matriz

Diz-se que uma matriz est na

3
0
0 1
0
1

no est.

forma es alonada reduzida

se estiver na forma es alonada por linhas, isto , satiszer as ondies anteriores da denio 2.16 e, alm disso, ada pivot igual a 1 e o ni o elemento
no nulo da sua oluna.

Observao 2.19.

Repare-se que a forma es alonada reduzida a forma es a-

lonada por linhas onde ada pivot


so nulos.

e, a ima destes, os elementos da matriz

2.2 Mtodo de eliminao de Gauss


Exemplo 2.20.

As matrizes

Sejam

1
A = 0
0

Teorema 2.21.

24

0 0
1 0
0 1

1
0
3

1
B = 0
0

4 0
0 1
0 0

esto na forma es alonada reduzida.

0 3
0 1
1 4

Toda a matriz pode ser olo ada na forma es alonada mediante

uma sequn ia nita de operaes elementares sobre as linhas.


Para onseguir transformar uma matriz numa matriz na forma es alonada
por linhas exe uta-se o mtodo de eliminao de Gauss.

Denio 2.22.

mtodo de eliminao de Gauss onsiste nos seguintes

passos.

Passo 1: Se a matriz tiver todos os elementos nulos, pra. A matriz j est na


forma es alonada por linhas.

Passo 2:

Caso ontrrio, en ontre-se a primeira oluna, da esquerda para a

direita, que tenha um elemento no nulo,

u.

Mova-se a linha que o ontm para

o topo da matriz. O pivot da primeira linha

u.

Passo 3: Anule-se ada elemento abaixo do pivot adi ionando s linhas orrespondentes mltiplos adequados da primeira linha.
(At aqui ompleta-se o pro esso no que diz respeito primeira linha. No
que se segue, usam-se as restantes linhas, ignorando a primeira linha.)

Passo 4: Repita-se os passos 1 a 3 na matriz formada

pelas restantes linhas,

at esgotar as linhas todas da matriz.


Para obter a forma es alonada reduzida de uma matriz apli a-se o mtodo

de eliminao de Gauss-Jordan.

Denio 2.23.

mtodo de eliminao de Gauss-Jordan

omposto

por duas fases.

1a fase: Apli ar o mtodo de eliminao

de Gauss at produzir a forma es a-

lonada por linhas.

2a fase: Transformar todos os pivots em 1, multipli ando ada linha no nula


pelo inverso do respe tivo pivot. Apli ar o mtodo de eliminao de Gauss de
baixo para ima por forma a anular todos os elementos da matriz situados a ima
e na mesma oluna dos pivots. Para isso, bastar omear na ltima linha no
nula e, de baixo para ima, adi ionar a ada linha mltiplos adequados das linhas
inferiores.

Exemplo 2.24.

Suponha-se que a matriz ampliada de um sistema tem a forma:

0
0

2
2

2
0
2
0

3 4 1
2
3 4

5
2 4
6
9 7

2.2 Mtodo de eliminao de Gauss

25

O Passo 1, do mtodo de eliminao de Gauss, ignorado uma vez que as


entradas da matriz no so todas nulas. No Passo 2 tem-se que en ontrar a
primeira oluna (da esquerda para a direita) om o primeiro elemento no nulo.
Neste aso a primeira oluna e o pivot pode ser a entrada

(3, 1).

Mova-se a

ter eira linha (a linha que o ontm) para o topo.

2
0

L1 L3
0
2

2 5
2 4
0
2
3 4

2
3 4 1
0 6
9 7

Agora basta operar om as linhas para obter zeros abaixo do pivot (Passo 3):

2
0

L4 := L4 L1
0
0

2 5
2 4
0
2
3 4

2
3 4 1
2 1
7 3

O Passo 4 manda onsiderar a submatriz obtida eliminando a primeira linha e


apli ar os Passos 1 a 3 at esgotar as linhas todas:

2
0

L2 L3
0
0

0
L4 := L4 L3
0
0

2 5
2 4

2
3 4 1

L := L4 + L2
0
2
3 4 4
2 1
7 3
2
2
0
0

5
2 4
3 4 1

2
3 4
0
0 0

2
0

0
0

2 5
2 4
2
3 4 1

0
2
3 4
0
2
3 4

O mtodo da eliminao de Gauss termina aqui e, omo tal, poder-se-ia j


determinar a soluo do sistema obtido, que equivalente ao sistema ini ial,
resolvendo-o por substituio.

2x + 2y 5z + 2t = 4
2y + 3z 4t = 1

2z + 3t = 4

No entanto, pode ontinuar-se a apli ar o mtodo de eliminao de GaussJordan matriz ampliada es alonada por linhas obtida e s depois determinar
a soluo. A segunda fase desse mtodo manda multipli ar ada linha no nula
pelo inverso do pivot orrespondente e operar om as linhas da matriz de modo
a obter zeros a ima dos pivots.

2.2 Mtodo de eliminao de Gauss

26

Obtm-se assim

2
0

0
0

2
2
0
0

5
2 4
3 4 1

2
3 4
0
0 0

L1 :=

1
L1
2

L2 :=

1
L2
2

L3 :=

1
L3
2

1
1
0
0

25

0 19
4
0 17
4
3
1
2
0
0

1
0

0
0


5
1 1
L1 := L1 + L3
0 1
2

0 0
3

L2 := L2 L3
0 0
2

1 0 0
0 1 0

L1 := L1 L2
0 0 1
0 0 0

9
17
4
3
2

1
2

3
2

3
2

1
0

2
0

1
2

25

2
0

19
2

25

2
0

A matriz ampliada obtida j est na forma es alonada reduzida. Agora basta


passar novamente para sistema e resolver de baixo para ima, por substituio:

x = 19
x + 9t = 19
2
2 9t
17
5
y 4 t = 2
y = 52 + 17
t R.
4 t ,

z + 32 t = 2
z = 2 23 t
 19


5
17
3
O onjunto soluo
2 9t, 2 + 4 t, 2 2 t, t : t R . Observe-se que
existe uma varivel livre, a varivel t, pelo que o sistema possvel e inde-

terminado, om grau de indeterminao igual a um.

Exer io Resolvido 2.25.


S:

(S) o seguinte sistema

x + 4z = 0
y = 1
.

x + y + 4z = 1

Seja

de equaes lineares

Determine, aso exista, o seu onjunto soluo.


Resoluo: A sua matriz ampliada :

1 0
M = 0 1
1 1

4
0
0 1
4 1

Passo 1: Anular os elementos da 1a oluna que se en ontram abaixo do pivot da


1a linha. Para isso efe tua-se a operao L3 := L3 L1 , obtendo-se

1 0
M1 = 0 1
0 1

4
0
0 1
0 1

2.2 Mtodo de eliminao de Gauss


Passo 2: Anular o elemento da 2
a

27

oluna que se en ontram abaixo do pivot da


L3 := L3 L2 e obtm-se

linha. Para isso efe tua-se a operao

1 0
M2 = 0 1
0 0

4
0
0 1
0
0

Passo 3: Transformar os pivots de ada linha em 1; para isso basta fazer a

operao L1 := L1 , obtendo-se

0 4
0
1
0 1
0
0
0

A matriz

M3

1
M3 = 0
0

a matriz ampliada de um sistema

S :

(S )

equivalente a

(S):

x 4z = 0
x = 4z
y = 1
y = 1

0=0
0=0

que um sistema possvel e indeterminado; qualquer terno da forma (4z, 1, z),


z R, soluo de (S ) e, onsequentemente, o onjunto soluo de (S)

om

{(4z, 1, z) : z R}.

Veja-se agora um exemplo em que a transformao dos pivots feito no nal


para evitar trabalhar om nmeros fra ionrios.

Exer io Resolvido 2.26.

Resolva o sistema

ando o mtodo de eliminao de Gauss.


Resoluo: A sua matriz ampliada :

2x y + z = 3
2x y + 4z = 2

4x + 2y + z = 5

2 1 1
2 1 4
4
2 1

3
2 .
5

, apli-

Apli a-se o mtodo

de eliminao de Gauss matriz ampliada do sistema.

2 1 1
3

2 1 4
2 L2 := L2 L1
4
2 1 5

2 1 1
3

0
0 3 1 L3 := L3 L2
L3 := L3 + 2L1
0
0 3
1

2 1 1
3
0
0 3 1
4
2 1 5

2 1 1
3
0
0 3 1
0
0 0
2

A matriz resultante a matriz ampliada do sistema seguinte, equivalente ao

dado:

O sistema impossvel.

2x y + z = 3
3z = 1

0=2

2.3 Dis usso de sistemas


Exer io 2.27.

28

Resolva o sistema

3y + z = 2
x + 2y z = 4

x + 5y + 2z = 5

, usando o mtodo de

eliminao de Gauss ou de Gauss-Jordan.

2.3 Dis usso de sistemas


No mtodo de eliminao de Gauss (ou de Gauss-Jordan) obtm-se, para uma
dada matriz, sempre o mesmo nmero de pivots. Isto , matrizes es alonadas
obtidas de uma mesma matriz tm o mesmo nmero de pivots.

Denio 2.28.

ara tersti a de uma matriz A o nmero de pivots de

uma qualquer matriz es alonada obtida de


elementares sobre as linhas de
Note-se que se

A.

A por apli ao su essiva de operaes


r(A) ou car(A).

Representa-se por

uma matriz do tipo

mn

ento

r(A) min{m, n}.

Considere-se o sistema de equaes lineares uja forma matri ial dada por

AX = B ,

om

uma matriz do tipo

mn

uma matriz do tipo

m 1.

Na

resoluo pelo mtodo de eliminao de Gauss (ou de Gauss-Jordan) faz-se:

Passo 1:

Forma-se a matriz ampliada

Passo 2:

Apli ar a

M=

A B

o mtodo em ausa. No de urso da apli ao do mtodo,

podem a onte er duas situaes:

a)

se surgir uma linha do tipo

, om

6= 0,

ento

o sistema impossvel. Assim, qualquer matriz es alonada obtida

b)

a partir de M


r( A B ).

teria um pivot na ltima oluna, pelo que

se no surgir, terminar o pro esso at obter uma matrix es alonada

por linhas (ou reduzida). Represente-se essa matriz por

Passo 3:

r(A) <



f) = r( A B ),
aso, r(A) = r(M
en ontram nas primeiras n olunas.

(somente no aso

b))

Na matriz

f,
M

f.
M

pois todos os pivots de

Neste

f
M

se

o nmero de olunas sem pivot or-

responde ao nmero de variveis livres a onsiderar. Isto , o nmero de


variveis livres dado por:

f)
n r(A) = n r(M

Para es olher as variveis dependentes e as livres pode-se efe tuar o seguinte ra io nio:
- variveis livres so as que orrespondem a olunas sem pivot;
- variveis dependentes so as outras, isto , as que orrespondem a
olunas om pivot.

2.3 Dis usso de sistemas


Se
,

29

f tiver pivots em todas as olunas orrespondentes s in gnitas, isto


M
f) = n ento no existem variveis livres e o sistema
r(A) = r(M

possvel e determinado.

Pode ento enun iar-se o seguinte teorema:

Teorema 2.29.
matriz do tipo

Seja

mn

AX = B um sistema de equaes lineares, onde A uma


e B uma matriz do tipo m 1. H exa tamente trs

possibilidades para a sua lassi ao:


1.
2.

3.


A B
= n;


A B
<n
AX = B possvel e indeterminado se e s se r(A) = r
e, nesse aso, diz-se que o sistema tem grau de indeterminao n r (A);


A B .
AX = B impossvel se e s se r(A) 6= r
AX = B

possvel e determinado se e s se

Exemplo 2.30.

onde
e

aeb

r(A) = r

Considere o sistema

xy+z =0

x + y + z = 2
,
y+z =a

z=b

so parmetros reais. Para determinar a relao entre os valores de

para que o sistema seja possvel, ome e-se por onstruir a matriz ampliada

do sistema e aplique-se o mtodo de eliminao de Gauss matriz obtida:

1
1

0
0

1
1
1
0

0

2

L2 := L2 L1

a
b

1 1 1
0
2 0
0
1 1
0
0 1

1 1
0


0
1
1

L := L3 L2
0
0
a 3
0
0
b

1 1 1
0
0

1
0
1

L4 := L4 L3
0
0 1
a+1
0
0 0 ba1


A B , isto
seja possvel r(A) = r

1
0

L2 := 2 L2
0
0

Para que o sistema

1
1
1
1
1
1
1
0

1
0
1
1

b a 1 = 0 b a = 1.

0
2

a
b
1
0
1
1

0
1

a+1
b

2.3 Dis usso de sistemas


Exer io Resolvido 2.31.

onde

30
Considere o sistema linear:

x+y+z+t=0

y 2z + 2t = 0
,
x + 2z 2t = 0

x 2z + at = b

so parmetros reais.

1. Determine os valores de

para os quais o sistema

i) impossvel;
ii) possvel e determinado.
2. Resolva-o, pelo mtodo da eliminao de Gauss, para

a=2

b = 0.

Resoluo:

1
0

1
1

1
1 0

2
2 0
L3 := L3 L1
2 2 0 L4 := L4 + L1
2
a b

L3 := L3 + L2

L4 := L4 L2

1
1
0
0

1.

1
1
0
1
0 1
0
1
1
0
0
0

1
1
0
0

1
1
2
2
1 1
1 a1

1 1
0 1

L4 := L4 + L3
0 0
0 0

i) O sistema impossvel se e s se
s se

a=2

b 6= 0.

1
1
2
2
1 3
1 a + 1

0
0

0
b

0
0

0
b

1
1
0
2
2
0

1 1 0
0 a2 b


A B , isto
r(A) 6= r

, se e

ii) O sistema possvel e determinado se e s se

r(A) = r
isto , se e s se
2. Se

a=2

b=0

a 6= 2 (b

A B



= 4,

pode tomar qualquer valor).

ento, pelo que foi feito anteriormente,

1
0

0
0

1
1
1 0
1 2
2 0
.
0 1 1 0
0
0
0 0

uma forma es alonada da matriz, ou seja, o sistema ini ial equivalente


ao sistema:

x+y+z+t=0

x = 4t

y 2z + 2t = 0
y = 4t

z t = 0

z = t

0=0

2.4 Sistemas homogneos

31

Assim o onjunto soluo

{(4t, 4t, t, t) : t R}.

2.4 Sistemas homogneos


Foi visto anteriormente que um sistema de equaes lineares homogneo um
sistema ujos termos independentes so todos nulos, ou seja, um sistema om
forma matri ial

AX = 0,

onde

A Mmn (K)

uma matriz oluna nula.

f il veri ar que este tipo de sistemas tem pelo menos uma soluo, a
soluo nula


X= 0 0

T

Mn1 (K).

Esta soluo designada soluo trivial do sistema homogneo. Assim, um sistema homogneo sempre possvel.

Exemplos 2.32.

1. O sistema homogneo

luo, a soluo nula


2. O sistema

(0, 0).

xy+z =0
x+y+z =0

xy =0
x+y =0

tem uma ni a so-

um sistema homogneo possvel e indeter-

minado, ujo onjunto soluo


soluo trivial

(0, 0, 0).

{(z, 0, z) : z R},

ao qual perten e a

Note-se que, nos exemplos anteriores, o primeiro tem o mesmo nmero de


equaes e in gnitas enquanto que o segundo exemplo tem mais in gnitas do
que equaes.

Exer io Resolvido 2.33.

Se um sistema de equaes lineares homogneo

tem mais in gnitas do que equaes, ento existe uma soluo no trivial.
Resoluo: Seja

AX = 0,

onde

uma matriz do tipo

signi a que h mais in gnitas que equaes). Como

m < n,

m n om n > m (que


A 0

r(A) = r

o sistema possvel e indeterminado e, onsequentemente, existe uma

soluo no trivial.

Os sistemas de equaes lineares homogneos possuem propriedades muito


simples mas bastante teis.

Proposio 2.34.

A Mmn (K). Se Xh Mn1 (K) uma soluo


AX = 0, ento Xh tambm soluo, para qualquer

Seja

do sistema homogneo

K.
Demonstrao. Se

AXh = 0.

Xh

uma soluo do sistema homogneo

Logo

A (Xh ) = (AXh ) = 0 = 0,
ou seja,

Xh

soluo do sistema

AX = 0.

AX = 0,

ento

2.4 Sistemas homogneos


Exemplo 2.35.
o terno

32

O sistema homogneo

(0, 1, 1),

2x y z = 0
x+y+z =0

admite omo soluo

logo tambm so soluo os ternos

2(0, 1, 1) = (0, 2, 2), (0, 1, 1) = (0, 1, 1), 10(0, 1, 1) = (0, 10, 10),

Proposio 2.36.

Seja

A Mmn (K). Se X1 , X2 Mn1 (K) so duas


AX = 0, ento X1 + X2 tambm soluo.

solues do sistema homogneo

X1 e X2
AX2 = 0. Logo

Demonstrao. Se

AX1 = 0

so solues do sistema homogneo

AX = 0,

ento

A (X1 + X2 ) = AX1 + AX2 = 0 + 0 = 0,


ou seja,

X1 + X2

soluo do sistema

Exemplo 2.37.

AX = 0.

solues os quadruplos

(0, 1, 1, 0)

2x y z w = 0
x+y+z+w =0
(0, 3, 2, 1), logo tambm

O sistema homogneo

admite omo

(0, 1, 1, 0) + (0, 3, 2, 1) = (0, 4, 3, 1)


uma soluo do sistema.
As duas proposies anteriores podem-se generalizar omo segue:

Proposio 2.38.

A Mmn (K). Se X1 , . . . , Xp Mn1 (K) so solues


AX = 0, ento 1 X1 + + p Xp tambm soluo,
1 , . . . , p K.
Seja

do sistema homogneo
para todos

A demonstrao  a omo exer io.


Observe-se que este fa to no vlido para sistemas de equaes lineares
ompletos.

Exemplo 2.39.
(1, 1, 1)

O sistema

(1, 2, 0),

mas

2x y z = 0
x+y+z =3

(2, 3, 1)

admite omo solues os ternos

no soluo do sistema.

Para qualquer sistema de equaes lineares ompleto uja forma matri ial
om A Mmn (K) e B Mn1 (K), podemos onsiderar o sistema
homogneo asso iado uja forma matri ial AX = 0.

AX = B ,

Exemplo 2.40.

Dado o sistema

x+y+z =2
2x + y = 3
,

xyz =4

2.4 Sistemas homogneos

a sua forma matri ial

33

AX = B

onde

O sistema homogneo asso iado

uja forma matri ial

1
A= 2
1

1
1
1
0
1 1

2
B = 3 .
4

x+y+z =0
2x + y = 0

xyz =0

AX = 0.

O prximo resultado estabele e que o onjunto soluo de um sistema de


equaes lineares ompleto pode ser obtido adi ionando uma soluo parti ular desse sistema aos elementos do onjunto soluo do sistema homogneo
asso iado.

Teorema 2.41.

Sejam A Mmn (K) e B Mm1 (K). Seja Xp Mn1 (K)


uma soluo parti ular do sistema de equaes lineares AX = B . Ento, X0

Mn1 (K) soluo desse sistema se e s se existe uma soluo Xh


do sistema homogneo asso iado AX = 0 tal que X0 = Xp + Xh .
Demonstrao. Por hiptese,
ou seja,

()

Xp

Mn1 (K)

uma soluo do sistema ompleto

AX = B ,

AXp = B .

Suponhamos que

X0

tambm uma soluo de

AX = B , isto , AX0 = B .

Ento

AXp = AX0 AXp AX0 = 0 A(X0 Xp ) = 0


Xh = X0 Xp
AX = 0.
ou seja,

()

Suponhamos agora que

uma soluo do sistema homogneo asso iado

Xh uma soluo do sistema homogneo asso iado


AXh = 0. Seja X0 = Xp + Xh . Ento

ao sistema dado, ou seja,

AX0 = A(Xp + Xh ) = AXp + AXh = B + 0 = B.


Donde

X0

uma soluo do sistema ompleto

Observao 2.42.

Se

S = {Xh : AXh = 0}

AX = B .

X0 uma soluo parti ular do sistema AX = B e se


o onjunto soluo do sistema AX = 0, tem-se que o

onjunto soluo do sistema

AX = B

X0 + S = {X0 + Xh : Xh S}1 .

1 A soma de um elemento a om um onjunto C dene-se omo sendo o onjunto formado


pelas somas de a om todos os elementos de C , ou seja, a + C = {a + u : u C}

2.4 Sistemas homogneos


Exemplo 2.43.

34

Considere-se o sistema de equaes lineares

O sistema homogneo

x y 4z = 4
x + 2y + 5z = 2

x + y + 2z = 0

x y 4z = 0
asso iado
x + 2y + 5z = 0

x + y + 2z = 0

. Construdas as matrizes

ampliadas e apli ado o mtodo de eliminao de Gauss obtm-se:

A B

1
= 1
1

A 0

1 4 4
2
5
2
1
2
0

1
= 1
1

1 4 0
2
5 0
1
2 0

1 1 4
0
1
3
0
0
0

1
0
0

4
2
0

1 4 0
1
3 0
0
0 0

(2.4)

(2.5)

Logo, de (2.4), vem que o sistema ompleto equivalente a

x y 4z = 4

y + 3z = 2

x=z2
y = 2 3z

e, portanto, signi a que um sistema possvel e indeterminado e o seu onjunto

{(z 2, 2 3z, z) : z R}.

soluo

De (2.5), vem que o sistema homogneo asso iado equivalente a

x y 4z = 0

y + 3z = 0

x=z
y = 3z

o que signi a que tambm um sistema possvel e indeterminado e o seu onjunto soluo

{(z, 3z, z) : z R}.

Repare-se que, se no onjunto soluo do sistema ompleto es olhermos uma


soluo parti ular, por exemplo, aquela que orresponde a

(2, 2, 0)

z = 0,

ou seja,

ento podemos es rever o onjunto soluo do sistema ompleto omo

soma desta soluo parti ular om o onjunto soluo do sistema homogneo


asso iado:

(2, 2, 0) + {(z, 3z, z) : z R}.

3. Matrizes invertveis.
Determinantes

3.1 Matrizes invertveis

36

3.1 Matrizes invertveis


Note-se que tm sido onsideradas propriedades das operaes om as matrizes
semelhantes s que j so familiares om os nmeros reais. Dada uma matriz
quadrada, formule-se, no onjunto das matrizes quadradas, a noo orrespondente ao inverso de um nmero real no nulo. Considere-se ento o on eito de
matriz invertvel.
Nesta se o onsideram-se apenas matrizes quadradas.

Denio 3.1. Seja A Mnn (K). A matriz A diz-se invertvel


singular) se existe uma matriz quadrada B Mnn (K) tal que

(ou

no

AB = BA = In .
A matriz

designa-se por

inversa de A.

Se no existir inversa, a matriz diz-se singular.

Exemplo 3.2.

Seja

A=

1
0

1
1

. Esta matriz invertvel. De fa to, seja

B=
Ento

AB = BA = I ,

Proposio 3.3.
B

1
0

ou seja, a matriz

1
1
B

inversa de

A.

A inversa de uma matriz quadrada ni a.

Demonstrao. Seja
que

A uma matriz quadrada de ordem n invertvel e suponha-se


A. Por denio,

so inversas de

AB = BA = In = AC = CA.
Logo

B = BIn = B(AC) = (BA)C = In C = C .

Portanto, a inversa de

ni a.

Uma vez que a inversa ni a, representa-se a inversa de

Observao 3.4.

Dada uma matriz quadrada

de ordem

Teorema 3.43) que se, para alguma matriz quadrada


ento

BA = In

e, onsequentemente,

Ento, para veri ar se uma dada matriz


veri ar que

AB = In

ou

BA = In ,

inversa de

B
A.

por

n,

prova-se (ver

de ordem

a inversa de

A1 .

n, AB = In

A apenas ne essrio

ou seja, no ne essrio veri ar as duas

igualdades.

Exer io 3.5.
admite inversa.

Mostre, usando a denio, que a matriz

A =

2 3
0 0

no

3.1 Matrizes invertveis

37

3.1.1 Propriedades da inversa


Sejam

A, B Mnn (K).

Algumas propriedades da inversa so:

Propriedade 1: In
Propriedade 2:

Se

invertvel e

(In )1 = In .

A invertvel, ento A1

Propriedade 3: Se A
(AB)1 = B 1 A1 .

invertvel e

so invertveis, ento

AB

A1

1

= A.

invertvel e

Demonstrao. Como A, B Mnn (K) so matrizes invertveis, existem


A1 , B 1 Mnn (K) tais que AA1 = In e BB 1 = In . Pelo que:



(AB) B 1 A1 = A BB 1 A1
1

= AIn A

pela asso iatividade do produto de


matrizes
por denio de inversa

= AA

por denio de elemento neutro


do produto de matrizes

= In

por denio de inversa.

1 1
Pela observao 3.4, e omo a inversa de uma matriz ni a, B
A a
1
1 1
inversa de AB , isto , (AB)
= B A e, portanto, AB invertvel.
Informalmente, pode dizer-se que a inversa do produto o produto das inversas pela ordem inversa. Este resultado pode ser generalizado ao produto
de vrias matrizes (mostre):

(A1 A2 Ak )
Propriedade 4: Se A
1
k
k N, Ak
= A1 .

1 1
= A1
k A2 A1 ,

invertvel, ento

Ak

invertvel e, para todo

Demonstrao. Pode demonstrar-se esta propriedade por dois pro essos

distintos:

1o

pro esso

Como

A Mnn (K)

uma matriz invertvel, existe

A1 Mnn (K)

tal

3.1 Matrizes invertveis


que

AA1 = In .

Ak

A1

k

38

Donde

= A A(A A1 )A1 A1
{z
}
| {z } |
k vezes

pela asso iatividade

k vezes

= A A In A
| {z }
|
k1 vezes

do produto de matrizes

A
{z
}

por denio de inversa

k1 vezes
1
1

= A A(A A
| {z } |
k1 vezes

)A

{z

A1
}

pela asso iatividade

k1 vezes

do produto de matrizes

.
.
.

.
.
.

= AA1 = In

por denio de inversa.

Ak

Pela observao 3.4, pode-se on luir que



k
k
k 1
= A1 .
inversa de A , isto , A

2o

invertvel e

A1

k

pro esso

Demonstre-se, por induo matemti a, que


onde

P(k)

a proposio

Ak
ou seja, tem-se que:
a) provar que
b) supor

P(k)

P(k 1)

1

= A1

verdadeira para

P(k) verdadeira para k N,

k

(3.1)

k = 1;

verdadeira e provar que

P(k)

verdadeira.

Assim
a) Para

k = 1,

a proposio (3.1)  a

que laramente verdadeira;

(A1 )1 = (A1 )1 A1 = A1 ,

b) Suponha-se agora que a proposio (3.1) vlida para k 1, isto ,



k1
k1 1
= A1
vlida a igualdade A
. Ento pode-se on luir
que:

Ak

1

1
= Ak1 A
1
= A1 Ak1
k1
= A1 A1
k
= A1

por denio de potn ia de matrizes


pela propriedade 3 da inversa
por hiptese de induo
por denio de potn ia de uma matriz.

3.1 Matrizes invertveis


Propriedade 5:

Se

39
A

1
AT

= (In )T

por denio de inversa

= In

por denio de transposta.

Pela observao 3.4, podemos on luir que



T
T
T 1
= A1 .
inversa de A , isto , A

Propriedade 6:
1
(A)1 = A1 .

Se

Exer io 3.6.

1, 2

A, B e C matrizes
T
C 1 AT
= In ;

Sejam

a)

C T B(AB)1

b)

A2 = In A = A1 ;

A2 = B 2 = (AB)2 = In

ento

AT

invertvel e

6= 0,

invertvel e

As demonstraes das propriedades

) se

invertvel e

A uma matriz invertvel, existe A1 e tem-se que:


T
T
A1 = A1 A
pela propriedade transposta do produto

Demonstrao. Como

AT

AT
T
= A1 .

invertvel, ento

ento

A1

T

invertvel e

so deixadas omo exer io.

quadradas de ordem

n.

Prove que:

AB = BA.

3.1.2 Algoritmo de inverso


Veja-se agora omo determinar a inversa de uma matriz ou de idir que uma
matriz no invertvel.

Exemplo 3.7.

Seja

A=

1 2
1 3

. Pretende-se averiguar se

uma matriz

invertvel e, em aso
determinar a sua inversa, ou seja, determinar
 armativo,

uma matriz

B=

x y
z t

tal que

AB = I2 .

Re orde-se que, pela observao 3.4, basta mostrar uma das igualdades da

denio de inversa. Ora

AB = I2

1
1

2
3



x y
z t

1
0

0
1

ou seja,

 

x + 2z y + 2t
1 0
=
x + 3z y + 3t
0 1

  

  
x + 2z
1
y + 2t
0

=
.
x + 3z
0
y + 3t
1

3.1 Matrizes invertveis

40

Assim, o sistema anterior equivalente resoluo dos dois sistemas de equaes lineares seguintes:

AX1 = B1
e

AX2 = B2

1 2
1 3
1
1

2
3





x
z
y
t

1
0

0
1

que tm a parti ularidade de terem a mesma matriz dos oe ientes. Apli ando
operaes elementares na matriz ampliada de ada sistema obtm-se:

1 2
1 3

1 2
1 3

1
0
0
1

L2 := L2 L1

L2 := L2 L1

1
0

2
1
1 1

1 2
0 1

0
1

L1 := L1 2L2

x
z

3
1

ou seja,

B=A

1
0

2
1

L1 := L1 2L2

3 2
1
1

y
t

1 0
0 1

Tem-se ento para solues dos dois sistemas:

3
1

0 2
1
1

(3.2)

(3.3)

Repare-se que as operaes efe tuadas em (3.2) e (3.3) so as mesmas. De fa to,


pode aglutinar-se as operaes num s pro esso, da seguinte forma:

A I


L2 := L2 L1

1 2
0 1

e, novamente, se onrma que

1 0
1 1

B = A1 =

L1 := L1 2L2

3 2
1
1

3 2
1
1

1 0
0 1

Note-se que no exemplo anterior partiu-se de uma matriz ampliada da forma

A I2

A1 .

e hegou-se a uma matriz ampliada da forma

I2

, onde

B=

Apresenta-se assim um algoritmo para inverter uma matriz quadrada.

Algoritmo de inverso
Seja

A Mnn (K).

1

Formar a matriz

2

Exe utar em

A In

In


uma sequn ia de operaes elementares sobre

as linhas que transformem a matriz


no nal do pro esso a matriz

 Caso no seja possvel obter In


a matriz A no invertvel.

In

A na matriz
A1 ;

identidade

In ,

obtendo-se

no lado esquerdo da matriz ampliada, ento

3.1 Matrizes invertveis


Exemplo 3.8.


I3

Seja

0
= 1
1

41

1
1 . Apli ando o algoritmo de inverso:
2

0 0
1 2 1 0 1 0

1 0 L1 L2 0 3
1 1 0 0
0 1
1 5
2 0 0 1

0 3
A= 1 2
1 5
3
1 1
2 1 0
5
2 0

1 2 1

1
L3 := L3 L1 0 3
0 3
3

1 2 1

1
L3 := L3 L2 0 3
0 0
2
1 2 1
1
L2 := L2
1
3
0 1
1
3
L3 :=

L3

Con lui-se

1 0

35

0
1
0
0
1
1
0
1
3
21

32

1 0
0 0
1 1

1 0
0 0
1 1

1 0

0 0
12 12
1

1
1
0 0
L1 := L1 2L2 0 1
3
3
0 0
1 12 21 12

1
5
1 0 0 3
5
2
6
6

L1 := L1 + L3

1
1
3
61
0 1 0
1
2
6

L2 := L2 L3
1
3
0 0 1 21 12
2
3
1
5
2
6
6
1
1
1
1
1

ento que A
= 2
= I3 .
6
6 . Verique que AA
1
1
1
2 2
2

Exer ios 3.9.

1. Mostre que a matriz

ule a sua inversa.

0 0
A = 1 2
3 7

2
6
9

invertvel e al-

2. Uma das seguintes matrizes singular. Cal ule a inversa no aso em que
possvel.

1
0
(a)
1
1

1
0
1
0

0
1
1
0

1
1

1
1

1
0
(b)
1
1

1
1
0
1

0
1
1
0

1
1

0
1

3.2 Determinantes. Con eitos gerais

3. Determine o valor de

42

para o qual a matriz

singular.

1
2
k
1
A = 3 1
5
3 5

3.2 Determinantes. Con eitos gerais


Denio 3.10.

Uma

n elementos apresentados por


(i1 , i2 , . . . , in ), onde ik {1, 2, . . . , n},
para todo j 6= k .

uma lista desses


-se por

ik 6= ij ,

permutao dos elementos do onjunto {1, 2, . . . , n}

Exemplo 3.11. (6, 3, 1, 5, 2, 4)

uma erta ordem. Representapara todo

uma permutao de

O onjunto de todas as permutaes de

k {1, 2, . . . , n},

{1, 2, 3, 4, 5, 6}.

{1, 2, ..., n}

representa-se por

Observe-se que a ardinalidade, isto o nmero de elementos, de

Exemplo 3.12. S3
3! = 6

o onjunto de todas as permutaes de

elementos. De fa to,

Sn

Sn .

n!.

{1, 2, 3}

e tem

S3 = {(1, 2, 3), (1, 3, 2), (2, 1, 3), (2, 3, 1), (3, 1, 2), (3, 2, 1)}.

Observao 3.13.

Note-se que para inferir que a ardinalidade de S4 4!, ou


4 vezes a ardinalidade de S3 , basta notar que, para ada permutao de
{1, 2, 3} existem 4 permutaes distintas de {1, 2, 3, 4}; de fa to, por exemplo,
das permutaes (1, 2, 3), (3, 2, 1) S3 podem onstruir-se as seguintes permutaes de S4
seja,

(1, 2, 3)

(4, 1, 2, 3)
(1, 4, 2, 3)
(1, 2, 4, 3)
(1, 2, 3, 4)

(3, 2, 1)

(4, 3, 2, 1)
(3, 4, 2, 1)
(3, 2, 4, 1)
(3, 2, 1, 4)

esse o aminho para a demonstrao por induo sobre


dade de

Sn

Denio 3.14.
k < j,

de que a ardinali-

n!.
Dada uma permutao

designa-se uma

Note-se que par

(i1 , i2 , . . . , in ) Sn ,

inverso se ik > ij .

(ik , ij ),

om

k < j,

uma inverso se

ik

o par

ij

(ik , ij ),

om

apare em na

permutao por ordem de res ente.

Exemplo 3.15.

(2, 1, 6, 3, 5, 4) S6 , o par (2, 1) uma inver(6, 3), (6, 5), (6, 4) e (5, 4) so inverses. Ao todo nesta

Na permutao

so. Tambm os pares


permutao o orrem

inverses.

3.2 Determinantes. Con eitos gerais


Observao 3.16.

43

Para determinar todas as inverses de uma permutao

(i1 , i2 , . . . , in )

basta onsiderar o primeiro elemento da permutao i1 e en ontrar todos os elementos que so menores que i1 e esto olo ados do lado direito
de i1 ; depois repetir o pro esso para os restantes elementos i2 , . . . , in1 .

Uma permutao (i1 , i2 , . . . , in ) Sn par (respe tivamente,


mpar) se o nmero total de inverses que nela o orrem par (respe tivamente,

Denio 3.17.
mpar).

Exemplos 3.18.
{1, 2}
i)

ii)

Vai-se estudar a paridade das permutaes dos onjuntos

{1, 2, 3}.

n=2
Permutao

Total de inverses

Paridade

(1, 2)
(2, 1)

0
1

mpar

Permutao

Total de inverses

Paridade

(1, 2, 3)
(2, 3, 1)
(3, 1, 2)
(3, 2, 1)
(2, 1, 3)
(1, 3, 2)

0
2
2
3
1
1

par

par

n=3

par
mpar
mpar
mpar

 
A = aij Mnn (K). O determinante
det A ou |A|, o es alar
X
(1) a1i1 a2i2 anin ,
det A =

Denio 3.19.
senta- -se por

par

Seja

de

A,

(i1 ,...,in )Sn

onde

o nmero de inverses da permutao

(i1 , i2 , . . . , in ).

Resulta imediatamente da denio (e do exemplo anterior) que:

 
det a11 = a11


a11 a12
= a11 a22 a12 a21
det
a21 a22

a11 a12 a13


= a11 a22 a33 + a12 a23 a31 + a13 a21 a32
det a21 a22 a23
a13 a22 a31 a12 a21 a33 a11 a23 a32
a31 a32 a33

repre-

3.2 Determinantes. Con eitos gerais


Exer io 3.20.
Sabendo que

Considere as matrizes

A = BT ,

A =

determine


a

c

4

44


a2
2

0
b+c


b 1
1
1 .
5
2

B =

4
b

c
a

Uma mnemni a para o l ulo de determinante de uma matriz do tipo


onhe ida por Regra de Sarrus e tem duas verses:

33

3.2 Determinantes. Con eitos gerais

45

a verso

Repetir as duas primeiras olunas da matriz da seguinte forma:

a11
A = a21
a31

a12
a22
a32

a13 a11
a23 a21
a33 a31

a12
a22
a32

Feito este pro esso, veri a-se a presena de

trs diagonais prin ipais: a diagonal prin ipal

trs diagonais se undrias: a diagonal se undria

nais paralelas a ela:

alelas a ela:

a12 , a23 , a31

a11 , a23 , a32

a11 , a22 , a33

e duas diago-

a13 , a21 , a32 ;


a13 , a22 , a31

e duas par-

a12 , a21 , a33 .

O determinante ser al ulado por meio da diferena entre o somatrio do


produto dos elementos das trs diagonais prin ipais e o somatrio do produto
dos elementos das trs diagonais se undrias, isto :

det A = (a11 a22 a33 + a12 a23 a31 + a13 a21 a32 ) (a13 a22 a31 + a11 a23 a32 + a12 a21 a33 ) .

(3.4)

a verso

A regra de Sarrus pode tambm ser apli ada repetindo-se as duas primeiras
linhas da matriz

da seguinte forma:

A =

a11
a21
a31
a11
a21

a12
a22
a32
a12
a22

a13
a23
a33

a13
a23

Novamente se veri a a presena de trs diagonais prin ipais e trs diagonais se undrias. O determinante al ulado da mesma forma, agora om estas
diagonais:

det A = (a11 a22 a33 + a21 a32 a13 + a31 a12 a23 ) (a13 a22 a31 + a23 a32 a11 + a33 a12 a21 ) .

(3.5)

Note que (3.4) e (3.5) so a mesma expresso.

1 1 2
3 1 . Apli ando
Exemplo 3.21. Seja A = 1
2
1 1


1 1 2 1 1


3 1 1
3
det A = 1
2
1 1 2
1

a regra de Sarrus,

= (1 3 1 + (1) 1 2 + 2 (1) 1)
(2 3 2 + 1 1 1 + (1) (1) 1)

= 3 2 2 12 1 1 = 15.

3.2 Determinantes. Con eitos gerais


Exer io 3.22.

46

Cal ule, apli ando a Regra de Sarrus,

3.2.1 Propriedades do determinante


Sejam

matrizes quadradas de ordem

n.


1

0

2

5
3
7
0
4 4

Ento:

(P1 ) Se A tem uma linha (respe tivamente, oluna) de zeros ento det A = 0;
Exemplo 3.23.

(P2 ) Se A = aij


0

0

5


1 3
0 0 = 0.
0 1

uma matriz triangular (superior ou inferior), ento

det A = a11 a22 ann

Exemplo 3.24.

(P3 )

5
0
0
0

1
2
0
0

0
3
1
0

4
0
1
1





= 5 (2) 1 1 = 10.


Se a uma linha (respe tivamente, oluna) de uma matriz

adi ionar-

se um mltiplo qualquer de uma outra linha (respe tivamente, oluna), o


valor do determinante no se altera;

Exemplo 3.25.



1
1 1 3



0
1 1 1
=



|{z}

0 0 1
L := L2 + L1 0
2


1 3
2 4 |{z}
= 2.
0 1 P2

(P4 ) Se A tem duas linhas (respe tivamente, olunas) iguais ou propor ionais,
ento

det A = 0;

Exemplo 3.26.


5

0

10

5

1
0
2
1

3
3
1
3


4
8
= 0.
1 |{z}
C1 =5C2

1

3.2 Determinantes. Con eitos gerais


(P5 )

47

Se se tro ar entre si duas linhas (respe tivamente, olunas) de

A,

ento

o valor do determinante muda de sinal;

Exemplo 3.27.

(P6 ) Se a matriz B



1
1 3

3 0 |{z}
= 0
0 1 L1 L2 5


0

46 = 1
5


3 0
1 3 = 46
0 1

se obtm a partir de uma matriz

A multipli ando uma das


, ento

suas linhas (respe tivamente, olunas) por um es alar

det B = det A
Isto :


a11

..
.

ai1 1

ai1

ai+1 1

.
..

an1

a12
.
.
.

ai1 2
ai2
ai+1 2
.
.
.

an2

Exemplo 3.28.

Sejam



a11
a1n

..

.
.
.

.


ai1 1

ai1 n


ain = ai1
ai+1 1
ai+1 n

.

.
.
..

.


an1
ann

1
A= 1
2

1 3
1 0
0 1

a12
.
.
.

ai1 2
ai2
ai+1 2
.
.
.

an2
e


a1n

.
.

.

ai1 n
ain
ai+1 n

.
.

.

ann

a matriz que se obtm

1
1 3
multipli ando a ltima linha de A por 2, ou seja, B = 1 1 0 .
4
0 2
Ento det B = 8 e det A = 4, isto , det B = 2 det A.

(P7 ) det(A) = n det(A), para todo K.

2
2 6
1
1 3
Exemplo 3.29. Sejam A = 1 1 0 e B = 2A = 2 2 0 .
4
0 2
2
0 1
3
Ento det B = 32 e det A = 4, isto , det B = 2 det A.


(P8 ) det AT = det A.


Exemplo 3.30.

1
2

1 1



= 3


1
1

2 1



= 3

3.2 Determinantes. Con eitos gerais


(P9 )

A duas matrizes tais que a linha (respe tivamente, oluna)


i da matriz A igual soma das linhas (respe tivamente, olunas) i das

matrizes A e A e as outras linhas (respe tivamente, olunas) das matrizes

A e A so iguais s linhas (respe tivamente, oluna) orrespondentes da


matriz A, ento
det A = det(A ) + det(A )
Sejam

48

Isto ,


a11


.
.

.

ai1 1

ai1 + bi1

ai+1 1


.
.

.

an1


a11

..
.

ai1 1

= ai1
ai+1 1

.
..

an1

.
.
.

ai1 2
ai2
ai+1 2
.
.
.

Exemplo 3.31.

.
.
.


a1n a11
..
.
.
.
.

ai1 n ai1 1
ain + bi1
ai+1 n ai+1 1
.
.
.
..
.

ann an1
an2


1 0 1
2 1 = 2
1 3 0

(P10 ) det(AB) = det A det B .


Exemplo 3.32.

Sejam

det B = 4.


1 7
Ento AB =
2 2

a1n

ai1 2
ai2 + bi2
ai+1 2

an2


1

20 = 5
0

.
.
.

a12





.
.

.

ai1 n
ain + bin =
ai+1 n

.
.

.

ann

a12

A=


1 0 1
1 0 + 3
1 3 0


2 1
1
1

e tem-se que

B=

a12
.
.
.

ai1 2
bi2
ai+1 2
.
.
.

an2


a1n

.
.

.

ai1 n
bin
ai+1 n

.
.

.

ann


1 0
1 1 = 9 + 11
1 3


1
3
1 1

, om

det(AB) = 12 = det A det B .




(P11 ) Se A uma matriz invertvel ento det A 6= 0 e det A1 =


Exemplo 3.33.
lado,

A1 =

1
3
2
3

A=

1

Seja

3
31

1
2

det A = 3

1
1

. Ento


det A1 = 13 .

det A = 3 6= 0.

1
.
det A
Por outro

3.2 Determinantes. Con eitos gerais


Exer io Resolvido 3.34.
3

tais que

det A = 2

a) det(3A)

b) det

1
det B = .
4

A3 B 1

Resoluo:

Sejam

49

matrizes reais quadradas de ordem

Determine, usando as propriedades:

) det

BAT

d) det



1
1
BT
2

det(3A) = 33 det A = 54;





det A3 B 1 = det A3 det B 1 = (det A)3 det1 B = 2;


det BAT = det(B) det AT = (1)3 det B det A = 12 ;


1 
3
1 
1
1
1
= 21 det B T
= 8 det(B
det 12 B T
T ) = 8 det B = 2

a)
b)
)
d)

Exer io 3.35.
det B = 5

Sejam

1
det C = .
2

a) det(ABC)

A, B, C M44 (R)

matrizes tais que

det A = 2,

Determine, usando as propriedades:

b) det(B

A C

) det(2B)

e) det

1 T 1
C A
3

3.2.2 Teorema de Lapla e


A denio de determinante pode tornar-se pesada se a matriz for de ordem

3. Observe-se que, por exemplo, existem 4! = 24 permutaes


{1, 2, 3, 4}. O prximo resultado permite al ular o determinante

superior a

do

onjunto

de

uma forma mais prti a.

Denio 3.36.
de

A,

de ordem

e da oluna

j.

 
A = aij Mnn (K). Seja A(i|j)
n 1, que se obtm desta a partir da
Seja

Chama-se

representa-se por

Aij ,

omplemento algbri o

(ou

a submatriz quadrada
supresso da linha

aij ,

o-fa tor)

ao es alar

Aij = (1)i+j det A(i|j).

3 2 4
Exemplo 3.37. Seja A = 0 1 2 . Ento
1 0 1






2+1 2 4
1+1 1
A21 = (1)
0 1 = 2 e A11 = (1)
0


2
= 1.
1

de

3.2 Determinantes. Con eitos gerais


Teorema 3.38 (Teorema de Lapla e).
det A =

n
X

Seja

aij Aij =

j=1

para quaisquer

50

n
X

 
A = aij Mnn (K).

Ento

ars Ars ,

r=1

i, s {1, . . . , n}.

Este teorema tambm onhe ido omo o desenvolvimento em o-fa tores


para o l ulo do determinante. Na prti a, onsiste em es olher uma linha (ou
uma oluna) e multipli ar ada entrada dessa linha (ou oluna) es olhida pelo
ofa tor orrespondente, e adi ionar os resultados. Isto , se es olher a linha
om

i {1, . . . , n},

det A = ai1 Ai1 + ai2 Ai2 + + ain Ain =


Se es olher a oluna

s,

om

s {1, . . . , n},

nante de

Seja

n
X

aij Aij .

j=1

ento:

det A = a1s A1s + a2s A2s + + ans Ans =

Exemplo 3.39.

i,

ento

3 2
7 0
1 2 3 8
.
A =
6
0 1 8
1
2
5 2

n
X

ars Ars .

r=1

Vai-se al ular o determi-

por apli ao dire ta do Teorema de Lapla e. Es olha-se a primeira

linha:



3 2
7 0

1 2 3 8
= 3A11 + (2)A12 + 7A13 + 0A14

6
0 1 8

1
2
5 2


2 3 8


= 3(1)1+1 0 1 8
2
5 2


1 3 8


+ (2)(1)1+2 6 1 8
1
5 2


1 2 8


0 8
+ 7(1)1+3 6
1
2 2

= 3(4 48 + 16 + 80) + 2(2 + 24 + 240 8 40 + 36)


+ 7(16 + 96 16 + 24)
= 1496.

3.2 Determinantes. Con eitos gerais

Exemplo 3.40.
B,

Seja

2
3
B =
2
1

6
5
4
2

8
9
.
7
2

Vai-se al ular o determinante de

utilizando o Teorema de Lapla e e as propriedades dos determinantes.


2

3

2

1

4
6
1
2

6
5
4
2


8
9
7
2

P6
=

P3
=

T.L. C1

T.L. C1

Exer io Resolvido 3.41.



a1

b1

c1

4
6
1
2

51

a2
b2
c2


a3
b3 .
c3



1 2 3 4


3 6 5 9


2

2 1 4 7
1 2 2 2

1
2
3

0
0
4
2
0 3 2
0
0 1


4
3
1
2


0 4 3


2 1 (1)1+1 3 2 1
0 1 2




2+1 4 3
2(3)(1)
1 2

6(8 3) = 30.

Sabendo que


2a1

2c1

2b1

a2 + a3
c2 + c3
b2 + b3


a3
c3 = 10,
b3

al ule

3.3 Condies de invertibilidade

52

Resoluo:

10

P9
=

P6

P4

P6
=

P5
=

Portanto,


a1

b1

c1

a2
b2
c2

Exer ios 3.42.


a3
b3 = 5.
c3

2. Para quaisquer



2a1 a2 + a3 a3


2c1 c2 + c3 c3


2b1 b2 + b3 b3


2a1 a2 a3 2a1


2c1 c2 c3 + 2c1


2b1 b2 b3 2b1


a1 a2 a3


2 c1 c2 c3 + 0
b1 b2 b3


a1 a2 a3


2 c1 c2 c3
b1 b2 b3


a1 a2 a3


2 b1 b2 b3 .
c1 c2 c3

1. Sabendo que


a1

a2

a3

x1 , x2 , y1 , y2

1 x1

1 y1

1 y1


a1

b1

c1

2b1
2b2
2b3

a2
b2
c2


a3
c3
b3

a3
c3
b3


a3
b3 = 2,
c3

al ule


4c1 + a1
4c2 + a2 .
4c3 + a3

R, mostre que

x2
x2 = (y1 x1 )(y2 x2 ).
y2

3.3 Condies de invertibilidade


possvel estabele er uma relao entre a existn ia ou no da inversa de uma
matriz atravs do determinante, bem omo lassi ar o sistema de equaes
lineares asso iado a essa matriz.

Teorema 3.43.

Seja

maes so equivalentes:

uma matriz quadrada de ordem

n.

As seguintes ar-

3.3 Condies de invertibilidade


a)

invertvel;

AX = 0

b) O sistema
)

53

r(A) = n

tem apenas a soluo trivial;

e a matriz

In

pode ser reduzida matriz

por operaes ele-

mentares sobre linhas;


d) O sistema
tipo

AX = B

possvel e determinado, para qualquer matriz

do

n 1;

e) Existe uma matriz quadrada

de ordem

tal que

AC = In .

Demonstrao. Prova-se este resultado atravs da sequn ia de impli aes

a) b) c) d) e) a).
a) b)
Se A
Mn1 (K)
ambos os

A1 Mnn (K). Suponha-se que X1


uma soluo do sistema AX = 0, isto , AX1 = 0. Multipli ando
1
, pode deduzir-se que:
membros da equao anterior por A
invertvel ento existe

A1 (AX1 ) = A1 0
(A1 A)X1 = 0

AX1 = 0

In X1 = 0 X1 = 0.
Portanto,

X1 = 0

e, dada a arbitrariedade de

X1 , AX = 0

tem uma ni a

soluo que a soluo trivial.

b) c)

Se o sistema

AX = 0

tem apenas a soluo trivial, ento um sistema

r(A) = r
reduzida a In por

possvel e determinado. Logo


impli a que

pode ser

c) d)
Seja B Mn1 (K).

Como



= n,

o que por sua vez

operaes elementares sobre linhas.

pode ser reduzida a

In

por operaes elemen-

tares por linhas, ento onsegue-se efe tuar a reduo por linhas



In B ,


A B
= n = r(A)
r
B

B Mn1 (K). Logo


AX = B possvel e determinado.
om

d) e)

Para ada

In .

e, portanto, o sistema

i {1, . . . , n}, dena-se Bi Mn1 (K) omo a oluna i da matriz

3.3 Condies de invertibilidade

54

i {1, . . . , n}, o sistema AX = Bi tem uma


Xi Mn1 (K). Seja C = X1 X2 Xn . Ento


AC = A X1 X2 Xn


= AX1 AX2 AXn por denio de produto entre matrizes


= B1 B2 Bn
por denio de Xi (AXi = Bi )
= In
por denio de Bi .
Ento, por hiptese, para ada

ni a soluo, diga-se

Ou seja, existe

C Mnn (K)

tal que

AC = In .

e) a)

Por hiptese, existe uma matriz quadrada

Para mostrar que

de ordem

Primeiro mostra-se que o sistema homogneo


X Mn1 (K) tal que CX =

soluo trivial. Seja

AC = In .
CA = In .

tal que

invertvel basta mostrar que tambm se tem

CX = 0
0. Ento

admite apenas a

X = In X = (AC)X = A(CX ) = A0 = 0.
Ou seja,

b) e),

X = 0

CX = 0 tem
C Mnn (K) tal

e, portanto,

existe uma matriz

apenas a soluo trivial. Como


CC = In . Mas ento

que

A = AIn = A(CC ) = (AC)C = In C = C .


E, portanto,

CA = AC = In ,

ou seja,

invertvel.

Foi visto nas propriedades dos determinantes que se uma matriz invertvel
ento o seu determinante no nulo. Prova-se mesmo que esta ondio
ne essria e su iente.

Teorema 3.44.

Uma matriz quadrada

Demonstrao. Seja

invertvel se e s se

det A 6= 0.

A Mnn (K).

  Suponha-se que A uma matriz invertvel. Ento existe uma matriz


A1 Mnn (K) tal que AA1 = In . Como o determinante do produto de

matrizes o produto dos determinantes de ada uma das matrizes, tem-se pela

P10 )

propriedade (

Logo

det A 6= 0.



det (A) det A1 = det AA1 = det (In ) = 1.

(3.6)

  A demonstrao da impli ao re pro a mais ompli ada e no ser

efe tuada.

1 0

2 1
,
Exer io Resolvido 3.45. Considere a matriz A =
1
1
1 0
0 2 2 0
onde um parmetro real. Determine os valores de para os quais a matriz
A invertvel.

3.4 Cl ulo da inversa a partir da matriz adjunta


Resoluo: Come e-se por al ular o determinante de

1 0


2 1

1
1
1 0

0 2 2 0

T.L. C1

(2 + 2 )

( 2).

det A 6= 0. Portanto, A inA invertvel para qualquer

Por teorema anterior,

e s se

vertvel se e s se

ou seja,

R \ {0, 2}.

A.

1

1
1
1(1)2+4 1
0 2 2


1
(1)(1)2+1
2 2

T.L. C4

A invertvel se
6= 0 e 2 6= 0,

55

6
1
1 , onde um parmetro real.
Exer io 3.46. Seja A = 0 1
0
1
+5
Determine os valores de para os quais o sistema homogneo AX = 0 admite
apenas a soluo trivial.

3.4 Cl ulo da inversa a partir da matriz adjunta


Seja

A Mnn (K). Chama-se matriz dos omplementos algbri os


A, matriz quadrada de ordem n denida por:

de

A,

representa-se por

onde

Aij

 
A = Aij

o omplemento algbri o da entrada

Denio 3.47.

Seja

representa-se por

adj A,

A,

A Mnn (K).

aij ,

Chama-se

para todo

i, j {1, . . . , n}.

matriz adjunta de

isto ,

T.
adj A = (A)

Exemplo 3.48.

Seja

A,

transposta da matriz dos omplementos algbri os de

1
A= 0
4

A11
A = A21
A31

A12
A22
A32

1 3
2 2 .
0 5

Ento

A13
10
8 8
A23 = 5 7
4
A33
4 2
2

3.4 Cl ulo da inversa a partir da matriz adjunta


Logo

4
2 .
2

10 5
T = 8 7
adj A = (A)
8
4

Exer io 3.49.

Considere a matriz

A = adj A.

56

4 3 3
0
1 .
A = 1
4
4
3

Verique que

O prximo resultado estabele e uma propriedade que permitir al ular a


inversa de uma matriz a partir da sua matriz adjunta.

Teorema 3.50.

Seja

A Mnn (K).

Ento

A (adj A) = (det A) In .
Mais, se

invertvel ento

1
adj A.
det A
 
A = aij Mnn (K),

A1 =
Demonstrao. Dada uma matriz

A (adj A) =

a11
a21

..
.
an1

a11
a21

..
.
an1

a12
a22
.
.
.

an2
a12
a22
.
.
.

an2

.
.
.

.
.
.

a1n

a2n

.
.

.
ann

a1n

a2n

.
.

.
ann

A11
A21

A12
A22

.
.
.

.
.
.

An1

An2

A11
A12

A21
A22

.
.
.

.
.
.

A1n

A2n

Pela denio de produto de matrizes, para ada


da matriz

A (adj A)

tem-se que

.
.
.

.
.
.

i {1, . . . , n},

T
A1n
A2n

.
.

.
Ann

An1
An2

.
.
.

.
Ann
a entrada

(i, i)

ai1 Ai1 + ai2 Ai2 + + ain Ain = det A,


i da matriz A.
i, j {1, . . . , n}, om i 6= j e onsidere-se a matriz B que se obtm
da matriz A, substituindo a linha j por uma linha igual linha i de A. Ento
det B = 0, porque a matriz B tem duas linhas iguais. Pelo Teorema de Lapla e,
apli ado linha j da matriz B , tem-se
pelo Teorema de Lapla e apli ado linha
Sejam

0 = det B = ai1 Aj1 + ai2 Aj2 + + ain Ajn ,

3.5 Sistemas de Cramer


que a entrada

(i, j)

57

da matriz

A (adj A).

Se

det A
0
0
det
A

A (adj A) =
.
.
.
.

.
.
0
0

invertvel, ento existe

A1 = A1

det A
det A

.
.
.

.
.
.

det A
e

= (det A) In .

det A 6= 0.

Logo

pela primeira parte do teorema


por denio de inversa

Considere-se a matriz

ento

1
A1 =
6

Seja

0
0

In

1
A1 A (adj A)
det A
1
adj A
=
det A

Exer io 3.52.

A1 Mnn (K)

Exemplo 3.51.

Portanto,

1
A = 0
4

do Exemplo 3.48. Como

det A = 6,

10 5 4
8 7 2
8
4
2

1 2
1 3.
2 1

Cal ule

A1 ,

usando a matriz adjunta.

3.5 Sistemas de Cramer


Denio 3.53.

Sejam A Mnn (K) e B Mn1 (K). Diz-se que um sistema


de equaes lineares na forma matri ial AX = B um

se a matriz

sistema de Cramer

invertvel.

Repare-se que se existe

A1

ento

AX = B A1 (AX) = A1 B X = A1 B.
Ou seja, um sistema de Cramer sempre possvel e determinado e a sua ni a
soluo dada por:

X = A1 B.
Observe-se que, neste tipo de sistemas, basta al ular a matriz inversa de A e
1
B para obter a soluo. No entanto existe um mtodo,
efe tuar o produto A
denominado Regra de Cramer, que permite a obteno da soluo destes sistemas de uma forma mais e iente.

3.5 Sistemas de Cramer

58

Teorema 3.54 (Regra de Cramer).

Sejam A Mnn (K) e B Mn1 (K)


matrizes tais que o sistema de equaes lineares na forma matri ial AX = B

j {1, . . . , n}, seja Aj a matriz que se obtm


pela ni a oluna da matriz B .

um sistema de Cramer. Para ada


de

substituindo a oluna

A soluo (ni a) do sistema

j =

AX = B

det(Aj )
,
det A

Demonstrao. Note-se que, sendo

n-uplo (1 , . . . , n ),

onde

para todo

j {1, . . . , n}.

AX = B

um sistema de Cramer, ento A


X = A1 B . Seja
T
bn . Ento:

uma matriz invertvel e a soluo do sistema dada por


T


1 . . . n essa soluo e suponha-se que B = b1

Repare-se que a entrada

.
.
.

(j, 1)

A1 B
1
(adj A) B.
det A

(adj A) B

da matriz oluna

A1j b1 + A2j b2 + + Anj bn = det Aj ,


por apli ao do Teorema de Lapla e oluna

j =

Exemplo 3.55.
ial

AX = B ,

det Aj
,
det A

para qualquer

da matriz

Portanto

j {1, . . . , n}.

Considere-se o sistema de equaes lineares uja forma matrionde

1 1
0
A= 1
1
1

3
1
3

Note-se que um sistema de Cramer uma vez que

Aj .

invertvel.


2
B = 1 .
0

det A = 2 6= 0

e, portanto,

De a ordo om o teorema anterior, a soluo deste sistema o terno

(x, y, z)

onde



2 1 3


1
0
1

0
1
3
x=
= 1, y =
2


1

1

1

2 3
1
1
0
3
2

Ou seja, a soluo do sistema dado

= 1

(1, 1, 0)

z=

(verique!).


1

1

1


1 2
0 1
1 0
=0
2

3.5 Sistemas de Cramer


Exer io 3.56.
tri ial

AX = B ,

59

Considere-se o sistema de equaes lineares uja forma maonde

1
A= 0
1

2 3
2 1
1 1


2
B = 1 .
3

Mostre que este sistema s admite uma soluo e al ule-a, apli ando a regra
de Cramer.

4. Espaos ve toriais sobre


um orpo

4.1 Denio e propriedades

61

4.1 Denio e propriedades


Seja

um onjunto no vazio onde esto denidas duas operaes internas,

representadas por

+ K

operao

em

, isto ,
K.

e por

e um e um s

para todo

hama-se adio em

, K,

e operao

K.
Estas operaes internas onferem a

Ko

existe um e um s

hama-se multipli ao

que habitualmente se hama estru-

tura algbri a, uja designao depende das propriedades que estas operaes
satisfazem.
Se a operao adio

veri ar as propriedades (P1 ) a (P4 ), onde

(P1 ) omutatividade: + = + , , K

(P2 ) asso iatividade: ( + ) + = + ( + ), , , K

(P3 ) existn ia de elemento neutro: 0K K, K: 0K + =

(P4 ) existn ia de elemento simtri o: K, K: +() = 0K


diz-se que a estrutura algbri a
Se a operao multipli ao

(K, +)

um grupo abeliano.

veri ar as propriedades (P5 ) a (P8 ), onde

(P5 ) omutatividade: = , , K

(P6 ) asso iatividade: ( ) = ( ), , , K

(P7 ) existn ia de elemento neutro: 1K K, K: 1K =

(P8 ) existn ia de elemento inverso: K \ {0K }, 1 K \ {0K }:


1 = 1K
diz-se que a estrutura algbri a

Denio 4.1.
resentadas por

Seja

(K \ {0K }, )

um grupo abeliano.

um onjunto munido de duas operaes internas, rep-

e por , tais que so vlidas as propriedades (P1 ) a (P8 ). Se

tambm for vlida a propriedade distributiva de

em relao

+,

isto , se

(P9 ) , , K, ( + ) = + ,
diz-se que a estrutura algbri a

(K, +, )

um

orpo.

0K , hama-se zero do orpo K e ao elemento


1K , hama-se identidade do orpo K.

Ao elemento neutro da adio,


neutro da multipli ao,

Exemplo 4.2. (R, +, ), (Q, +, ) e (C, +, ) so orpos, onde + e representam,

respe tivamente, a adio e a multipli ao usuais entre nmeros.

4.1 Denio e propriedades


Denio 4.3.

Seja

62

um orpo e seja

um onjunto no vazio onde esto

denidas duas operaes:

uma operao interna, representada por

u, v E

tal que a todos

e uma operao externa, representada por

E , tal
u E.

por es alar em
elemento

e designada por adio em

que a todo

E,

u v E;

faz orresponder o elemento

e designada por multipli ao


u E faz orresponder o

e a todo

(E, , ) um espao ve torial sobre o


orpo K se so satisfeitas as seguintes propriedades, para quaisquer u, v, w E

Diz-se que a estrutura algbri a


e quaisquer

, K:

(A1 ) u v = v u

( omutatividade de

(A2 ) (u v) w = u (v w)
(A3 ) 0E E , u 0E = u

(asso iatividade de

(existn ia de elemento neutro de

(A4 ) u E , u (u) = 0E

)
)

(existn ia de elemento simtri o em

e ainda

(M1 ) ( + ) u = ( u) ( u) (distributividade
(M2 ) (u v) = u v
(M3 ) ( u) = ( ) u
(M4 ) 1K u = u

(distributividade de

em relao a

em relao a

+)

(asso iatividade mista)

(existn ia de elemento neutro de

Aos elementos de

de

hamam-se ve tores e aos elementos de

es alares. O elemento neutro para a adio de

(representado por 0E ). Quando K = R (respe tivamente,


espao ve torial real (respe tivamente, omplexo ).
Os es alares so aqui representados por letras gregas
por letras mins ulas do alfabeto

u, v, w, . . . .

hamam-se

toma o nome de ve tor nulo

C),

diz-se que

, , , . . .

e os ve tores

Como as operaes entre ve tores

e entre um es alar e um ve tor no se onfundem fa ilmente, em vez de


usaremos simplesmente a notao

um

(sendo este ltimo smbolo omitido). O

ontexto determinar qual a operao que est em ausa.


Apresentam-se de seguida alguns exemplos de espaos ve toriais reais.

Exemplos 4.4.

1. O onjunto

R2 = {(x1 , x2 ) : x1 , x2 R}

om as opera-

es usuais entre pares ordenados:

(x1 , x2 ) + (y1 , y2 ) = (x1 + y1 , x2 + y2 )


(x1 , x2 ) = (x1 , x2 ),
para todo
real.

(x1 , x2 ), (y1 , y2 ) R2

e para todo

R,

O ve tor nulo (0, 0) e o simtri o do ve tor


(x1 , x2 ) R2 .

um espao ve torial

(x1 , x2 ) R2

o ve tor

4.1 Denio e propriedades

63

2. Generalizando o exemplo anterior, para

n N,

o onjunto

Rn = {(x1 , x2 , . . . , xn ) : x1 , x2 , . . . , xn R},
om as operaes usuais entre

n-uplos:

(x1 , x2 , . . . , xn ) + (y1 , y2 , . . . , yn ) = (x1 + y1 , x2 + y2 , . . . , xn + yn )


(x1 , x2 , . . . , xn ) = (x1 , x2 , . . . , xn ),
para todo

(x1 , x2 , . . . , xn ), (y1 , y2 , . . . , yn ) Rn

e para todo

espao ve torial real.

(0, 0, . . . , 0) e o simtri o
(x1 , x2 , . . . , xn ) Rn .

O ve tor nulo
o ve tor

do ve tor

R,

um

(x1 , x2 , . . . , xn ) Rn

3. O onjunto

{p(x) : p(x)

P [x] =

um polinmio om oe ientes reais

{p(x) = an xn + + a1 x + a0 : an , . . . , a0 R, n N0 }

om a adio usual de polinmios e a multipli ao de um polinmio


por um nmero real um espao ve torial real. Note-se que, dados dois
polinmios

p(x), q(x) P [x]


p(x)
q(x)

de graus

m,

respe tivamente, tais que

= an xn + + a1 x + a0
= bm xm + + b1 x + b0

a adio e a multipli ao por um nmero real esto denidas da seguinte


forma: supondo, sem perda de generalidade, que

nm

ento

p(x) = 0xm + + 0xm + an xn + + a1 x + a0


e, sendo assim,

p(x) + q(x) = (0 + bm )xm + + (0 + bn+1 )xn+1 + (an + bn )xn +


+ + (a1 + b1 )x + (a0 + b0 )
e, para todo

R,
p(x) = an xn + + a1 x + a0

O ve tor nulo o polinmio nulo 0(x) = 0 e o simtri o do polinmio


p(x) = an xn + + a1 x + a0 P [x] o polinmio

p(x) = an xn a1 x a0 P [x].
Mmn (R) = {A : A uma matriz real do tipo m n}, om a
adio usual de matrizes e a multipli ao por um es alar, um espao

4. O onjunto

ve torial real.

 e o simtri o
m n
A = aij Mmn (R).

O ve tor nulo a matriz nula do tipo

 
A = aij Mmn (R)

a matriz

da matriz

4.1 Denio e propriedades


5. O onjunto

64

F (R) das funes reais de varivel real om domnio R, munido

da adio usual de funes e multipli ao duma funo por um nmero


real um espao ve torial real. Note-se que, dadas as funes

f +g

uma funo real de varivel real tal que

(f + g)(x) = f (x) + g(x),


e

para todo o

f, g F (R),

xR

uma funo real de varivel real tal que

(f )(x) = (f (x)) ,

xR

para todo o

O ve tor nulo a funo identi amente nula, isto ,

x R e o simtri o de uma funo f F (R)


que (f )(x) = f (x), para todo x R.

f (x) = 0, para todo


f F (R) tal

a funo

Vejam-se agora algumas propriedades dos espaos ve toriais.

Proposio 4.5.
ve tores

u, v E

Seja

um espao ve torial sobre

e quaisquer es alares

, K,

K.

Ento, para quaisquer

tem-se que:

(b) 0E = 0E

(a) 0K u = 0E
( ) ()u = (u)

(d) (u) = (u)

(e) ( )u = u u

(f) (u v) = u v.

Notao: u v = u + (v) e = + ().


Demonstrao. Prove-se

(a). Por denio de zero do orpo, sabe-se que


0K = 0K + 0K

e, portanto,

0K u = (0K + 0K )u. Por (M1 ), obtm-se


0K u = 0K u + 0K u.

Por (A4 ), existe simtri o de

0K u,

isto , existe

(0K u).

Somando a ambos os

membros da equao, esse simtri o e utilizando (A2 ), tem-se que:

0K u + (0K u) = (0K u + 0K u) + (0K u) = 0K u + (0K u 0K u) .


Logo, pelas ondies (A3 ) e (A4 ), vem que:

0E = 0K u + 0E 0E = 0K u.
Prove-se

(b), analogamente. Por (A3 ), 0E = 0E +0E , logo 0E = (0E +0E ).

Pela ondio (M2 ), vem que:

0E = 0E + 0E .

4.1 Denio e propriedades

65

Somando a ambos os membros o simtri o de

0E ,

isto , somando

usando (A1 ), (A4 ) e (A3 ), tem-se que:

0E + (0E ) =
0E
0E
Prove-se
de

u.

=
=

(0E + 0E ) + (0E )
0E + (0E + (0E ))
0E .

( ). Repare-se que ()u = (u) signi a que ()u o simtri o

Ento, tem que se mostrar que

()u + u = 0E .

()u + u = ( + )u
= 0K u

Prove-se

por denio de simtri o em


por

(a).

u,

ou seja, mostre-se que

(u) + u = (u + u)
= 0E

Ora

por (A1 ) e (A4 )


por

(b).

(e). Repare-se que


( )u = ( + ())u

por notao

= u + ()u

por (M1 )

= u + (u)
= u u

por

( )

por notao

(f), analogamente. Repare-se que


(u v) = (u + (v))
= u + (v)

por notao
por (M2 )

= u + (v)
= u v

Exer io 4.6.
ve tores

(u) + u = 0E .
por (M2 )

= 0E

Prove-se

(d), de forma anloga. Repare-se que (u) = (u) signi a que

o simtri o de

Prove-se

Ora

por (M1 )

= 0E
(u)

0E ,

Seja

u, v, w E

(a) (u) = u;
se

u = 0E

( )

se

u+v =u+w

(d)

se

u = u

ento

= 0K

ento

u 6= 0E

ou

u = 0E ;

v = w;

ento

= .

(d)

por notao

K. Prove que, para


, K, tem-se que:

um espao ve torial sobre

e para quaisquer es alares

(b)

por

quaisquer

4.2 Subespaos ve toriais

66

4.2 Subespaos ve toriais


Denio 4.7.
no vazio de
por
em

E.

Seja

um

F E , se F for um espao
F pelas operaes de E .
E

f il veri ar que

um espao ve torial sobre

Diz-se que

ve torial sobre

{0E }

e seja

um sub onjunto

subespao ve torial de E , e representa-se


K

para as operaes induzidas

so subespaos ve toriais de

E.

Estes subes-

paos hamam-se subespaos triviais. Todos os outros subespaos ve toriais so


no triviais.

Exemplo 4.8.

O sub onjunto de

P [x] onstitudo pelos polinmios


n , isto , o onjunto

de oe-

ientes reais de grau menor ou igual a

Pn [x]

=
=

{p(x) P [x] : grau(p(x)) n}

{p(x) = an xn + + a1 x + a0 : ai R, i {0, 1, . . . , n}}

P [x] para as operaes denidas em P [x]. Note-se


Pn [x] um sub onjunto no vazio de P [x] pois o polinmio nulo perten e
a Pn [x]. Alm disso, para as operaes denidas em P [x], Pn [x] satisfaz todas
as ondies da denio de espao ve torial. As ondies (A1 ), (A2 ), (M1 ),
(M2 ), (M3 ) e (M4 ) so vlidas porque se so vlidas para todos os polinmios
de P [x] tambm so vlidas para os polinmios de grau menor ou igual a n. O
polinmio nulo perten e a Pn [x] e, portanto, vlida a ondio (A3 ). Por m,
o simtri o de um polinmio de grau menor ou igual a n ainda um polinmio
de grau menor ou igual a n, logo vlida (A4 ).
um subespao ve torial de

que

Seguidamente apresenta-se uma ara terizao para que um sub onjunto


dum espao ve torial

Proposio 4.9.
de

E.

Ento

Seja

seja um seu subespao ve torial.

um espao ve torial sobre

um subespao ve torial de

e seja

um sub onjunto

se e s se

(i) F 6= ;

(ii) u, v F , u + v F (F

(iii) K, u F ,
es alar).

Exemplo 4.10.
(no trivial) de

fe hado para a adio);

u F

(F fe hado para a multipli ao por um

Qualquer re ta que passa na origem um subespao ve torial

R2 .

De fa to, seja

m N.

O onjunto

F = {(x, y) R2 : y = mx} = {(x, mx) : x R}


um subespao ve torial de

R2 .

De fa to

disso,

(i) (0, 0) F

j que

(0, 0) = (0, m0)

F R2 ,

e, portanto,

por denio de

F 6= ;

F.

Alm

4.2 Subespaos ve toriais


(ii)

sejam

67

(x1 , mx1 ), (x2 , mx2 ) F

arbitrrios, ento

(x1 , mx1 )+(x2 , mx2 ) = (x1 +x2 , mx1 +mx2 ) = (x1 +x2 , m(x1 +x2 )) F ;

(iii)

sejam

(x, mx) F

arbitrrios, ento

(x, mx) = (x, (mx)) = (x, m(x)) F.


Pela Proposio 4.9,

um subespao ve torial de

Exer io Resolvido 4.11.


um subespao ve torial de

R2 .

H = {ax2 + bx + c P2 [x] : c = 0}
P2 [x] = {ax + bx + c : a, b, c R}.
Mostre que
2

Resoluo: Note-se que

H = {ax2 + bx + c P2 [x] : c = 0} = {ax2 + bx : a, b R}


Ora

(i)

H P2 [x],

x2 x H
H 6= ;

o polinmio
portanto,

(ii)

por denio de

sejam

H.

Alm disso:

pois da forma

ax2 + bx, cx2 + dx H ,

ax2 + bx

(iii)

seja

b = 1,

e,

fe hado para a adio de polinmios.

ax2 + bx H

(ax2 + bx) + (cx2 + dx) = (a + c) x2 + (b + d) x H;


| {z }
| {z }
e seja

R,

ento

(ax2 + bx) = (a) x2 + (b) x H;


|{z}
|{z}
R

Logo

a=1

ento

Logo

om

fe hado para a multipli ao de um polinmio por um nmero

real.
Logo, pela Proposio 4.9,

Exer io Resolvido 4.12.


perten e a

S . Logo
R3 .

Verique se

P2 [x].

S = {(x, y, z) R3 : z = 1}

um

R .
que (0, 0, 1), (1, 1, 1) S e (0, 0, 1) + (1, 1, 1) = (1, 1, 2) no
S no fe hado para a adio e portanto no subespao

subespao ve torial de
Resoluo: Note-se

um subespao ve torial de

ve torial de

Exer io 4.13.

Averige se os seguintes onjuntos so subespaos ve toriais

dos espaos ve toriais indi ados.

4.2 Subespaos ve toriais

68

a)

F = {(x, y) R2 : x = 0}

b)

G = {(x, y, z) R3 : y = 2}

S = {ax2 + bx + c P2 [x] : a 6= 0}

de

R2 ;

de

R3 ;
de

P2 [x].

O prximo resultado apresenta algumas propriedades dos subespaos ve toriais.

Proposio 4.14.
ve torial de

E.

Seja

um espao ve torial sobre

e seja

um subespao

Ento:

(a) 0E F ;

(b)

se

( )

uF

se

u, v F

ento

u F ,

ento

u v F,

Demonstrao. Prove-se
Por outro lado, omo
Logo, omo

para todo

Prove-se

para todo

u, v F .

(a). Como K orpo, existe o zero do orpo, 0K K.

F 6= ,

existe

u F.

um subespao ve torial, pela ondio

4.9, vem que:

(iii)

da Proposio

0K u = 0E F.

(b),

dade do orpo,

u F;

analogamente. Como

1K K.

Seja

Novamente, pela ondio

orpo, existe o simtri o da identi-

u F.

(iii) da Proposio 4.9, vem que:


(1K )u F.

Como

um subespao ve torial, pela ondio ( ) da Proposio 4.5 e pela

ondio (M4 ), vem que


Prove-se

( ).

Sejam

(1K )u = (1K u) = u.

Logo

u F .

Por (b), v F . Assim, omo F


(ii) da Proposio 4.9, tem-se que

u, v F .

subespao ve torial, pela ondio

um

u + (v) = u v F.

Atendendo proposio anterior, a ondio


ser substituda por

0E F .

(i)

da Proposio 4.9 pode

Alm disso, as ondies

(ii)

(iii)

podem ser

substitudas por uma s ondio.

Proposio 4.15.
de

E.

Ento

(1) 0E F ;

Seja

um espao ve torial sobre

um subespao ve torial de

(2) , K , u, v F , u + v F .

K e seja F

se e s se

um sub onjunto

4.3 Combinao linear de ve tores


Demonstrao.   Suponhamos que

E . Pela
u, v F e , K ento,
pela ondio (iii) da Proposio 4.9, u F e v F . Logo, novamente pela
Proposio 4.9, mas pela ondio (ii), u + v F .

(1).

proposio anterior, vale

69
um subespao ve torial de

Alm disso, dados

  Repare-se que:

(i) F 6= pois, por (1), 0E F .

(ii)
(iii)

Sejam

u, v F .

Por

Analogamente, por
on lui-se que

Pela Proposio 4.9,

(2), fazendo = = 1K , on lui-se que u + v F .

(2),

u F .
F

sejam

uF

K,

um subespao ve torial de

fazendo

= 0K

v = 0E ,

E.

4.3 Combinao linear de ve tores


Denio 4.16.

E um espao ve torial sobre K e u1 , u2 , . . . , uk E .


v E ombinao linear dos ve tores u1 , u2 , . . . , uk
es alares 1 , 2 , . . . , k K tais que
Sejam

Diz-se que o ve tor


se existem

v = 1 u1 + 2 u2 + + k uk .

Exemplos 4.17.

1. Considere no espao ve torial real

v = (2, 1, 3), v1 = (1, 0, 0), v2 = (1, 0, 1)


O ve tor

ombinao linear dos ve tores

De fa to, mostre-se que existem

1 , 2 , 3 R

os ve tores

v3 = (1, 2, 1).

v1 , v2

R3 ,

v3 .

tais que

(2, 1, 3) = 1 (1, 0, 0) + 2 (1, 0, 1) + 3 (1, 2, 1).


Ou seja, pretende-se veri ar que possvel o sistema

2 = 1 + 2 + 3
1 = 23

3 = 2 + 3

Sendo assim, passando o sistema para a sua matriz ampliada

1 1
2
1 1 1
2

0 2 1 L2 L3 0 1 1
3
1 1
3
0 0 2 1


A B
Note-se que r(A) = r
e, portanto, o sistema anterior
svel e determinado. Ou seja, v ombinao linear de v1 , v2 , v3 .
1
0
0

pos-

4.3 Combinao linear de ve tores

70

Continuando om o es alonamento da matriz at sua forma reduzida


obtm-se:

1 1
L3 := 2 L3 0 1
0 0

1 0

L2 := L2 L3 0 1
0 0

Donde

Assim,

1
2

3 L1 := L1 L2 0
0
21

1
7
2
21

1
1
1
0
0
1

1 = 1
2 = 72

3 = 21

ombinao linear de

v1 , v2

v3

pois

v3

ombinao linear de

6x3 x2 + 3x 11 =

Assim

1 = 3

2 = 1.

Exer io Resolvido 4.18.


v = (1, 3, 1)
v3 = (0, 1, 0).

v2 .

De fa to,

1 (2x3 + x 3) + 2 (x2 + 2)

21 x3 + 2 x2 + 1 x 31 + 22 .

Donde

(0, 1, 0)

se e s se

v = 3v1 v2 .

R3 , averige se o ve tor
v1 = (1, 0, 1), v2 = (1, 1, 1) e

No espao ve torial real

ombinao linear dos ve tores

Resoluo: Ora, o ve tor

v1 = 2x3 + x 3,

6x3 x2 + 3x 11 = 21 x3 + 2 x2 + 1 x 31 + 22

6=6
21 = 6

2 = 1
2 = 1

1 = 3

=
3

11 = 11
22 31 = 11

E, portanto,

(1, 1, 1)

v1

1
3
1
2

v = v1 + 27 v2 12 v3 .

2. No espao ve torial real P [x], onsiderem-se os ve tores


v2 = x2 + 2 e v3 = 6x3 x2 + 3x 11.
O ve tor

0 0
1 1
0 1

(1, 3, 1) ombinao
1 , 2 , 3 R tais

se existirem

linear dos ve tores


que

(1, 3, 1) = 1 (1, 0, 1) + 2 (1, 1, 1) + 3 (0, 1, 0)


ou seja,

(1, 3, 1) = (1 + 2 , 2 3 , 1 + 2 ),
isto , se possvel o sistema

1 + 2 = 1
2 3 = 3

1 + 2 = 1

(1, 0, 1),

4.4 Independn ia e dependn ia linear

71

Passando para a matriz ampliada asso iada

1
0 1
1 1
0 1

1 1
3 L3 := L3 L1 0 1 1
3
1
0 1
0 0
0
0


A B
Tem-se ento r(A) = r
e, portanto, o sistema possvel
v ombinao linear dos outros trs ve tores v1 , v2 , v3 .
1
0
1

e o ve tor

Continuando a resoluo do sistema, e usando a matriz ampliada anterior

obtm-se:

Tomando

3 = 0

1 + 2 = 1

2 3 = 3

temos

1 = 4 e 2 = 3

1 = 4 + 3
2 = 3 + 3

Assim,

(1, 3, 1) = 4(1, 0, 1) + 3(1, 1, 1) + 0(0, 1, 0),


ou, fazendo

3 = 1

temos

1 = 5 e 2 = 4.

Assim,

(1, 3, 1) = 5(1, 0, 1) + 4(1, 1, 1) + 1(0, 1, 0).


Observe-se assim que
de

v1 , v2

no se es reve de forma ni a omo ombinao linear

v3 .

Exer io 4.19.
a) o ve tor
em R3 ;

Nos espaos ve toriais indi ados, verique se:

(1, 3, 1)

ombinao linear dos ve tores

(1, 0, 0)

(0, 1, 0),

(2, 2, 5) ombinao linear dos ve tores (1, 1, 1), (1, 1, 0) e


R3 ;



 

1 1
1 0
1 1
a matriz
ombinao linear das matrizes
,
0 2
2 1
0
3

2 2
e
, em M22 (R).
1 1

b) o ve tor

(1, 0, 1),
)

em

4.4 Independn ia e dependn ia linear


Seja

K. O ve tor nulo de E
u1 , u2 , . . . , uk E . Com efeito

um espao ve torial sobre

linear de quaisquer ve tores

sempre ombinao

0E = 0K u1 + 0K u2 + + 0K uk .
A esta ombinao linear nula (isto , ujo resultado o ve tor nulo) d-se
o nome de ombinao linear nula trivial.
Ser esta a ni a forma de o fazer? Nem sempre!

4.4 Independn ia e dependn ia linear


Exemplo 4.20.
e

No espao ve torial real

R3 ,

72

onsidere os ve tores

v1 = (1, 1, 1)

v2 = (2, 2, 2).

Fa ilmente se veri a que:

(0, 0, 0) = 0v1 + 0v2

(0, 0, 0) = 2v1 + v2 .

Neste aso, o ve tor nulo no se es reve de forma ni a omo ombinao linear


dos ve tores

v1

v2 .

De fa to,

v1 + v2 = (0, 0, 0) ( 2, 2, 2) = (0, 0, 0)
equivalente a
valor de

= 2

R.

R.

E se forem os ve tores

Donde,

v1 = (1, 1, 1)

(0, 0, 0) = 2v1 + v2 ,
v3 = (1, 0, 0)?

para qualquer

Neste aso,

+ =0
v1 + v3 = (0, 0, 0) ( + , , ) = (0, 0, 0)
=0

=0

Assim, = 0 e = 0. Ento, para os ve tores v1 e v3 , o ve tor nulo es reve-se


de forma ni a omo a ombinao linear nula trivial.
Apresentam-se assim as seguintes denies:

Denio 4.21.

Seja

Diz-se que os ve tores

E um espao ve torial sobre K e sejam u1 , u2 , . . . , uk E .


u1 , u2 , . . . , uk so:

(i) linearmente independentes

se

1 u1 + 2 u2 + + k uk = 0E 1 = 2 = = k = 0K
Isto , existe apenas uma ni a forma de es rever o ve tor nulo de
omo ombinao linear de

u1 , u2 , . . . , uk

e que , neste aso, a ombinao

linear nula trivial.

(ii) linearmente dependentes


existem

1 , . . . , k K

se

no todos nulos tais que

1 u 1 + + k u k = 0 E

Isto , existem es alares em que pelo menos um deles diferente do zero


do orpo que do origem a uma ombinao linear nula, ou seja, existem
outras ombinaes lineares nulas para alm da ombinao linear nula
trivial.

Exemplos 4.22.

1. No espao ve torial real

os ve tores

v1 = (1, 2) e v2 = (2, 1)
, R tais que

R2 ,

so linearmente independentes.

De fa to, sejam

(1, 2) + (2, 1) = (0, 0) ( + 2, 2 + ) = (0, 0)

4.4 Independn ia e dependn ia linear


Ento

73

+ 2 = 0

2 + = 0

=0
=0

Ou seja, s existe a ombinao linear nula trivial.

os ve tores

v1 = (1, 2)

v2 = (2, 4)

so linearmente dependentes.

Existe, obviamente, a ombinao linear nula no trivial

2v1 v2 = (0, 0) 2(1, 2) (2, 4) = (0, 0)


Em termos geomtri os, a dependn ia linear destes dois ve tores
evidente pois os ve tores so olineares.
2. No espao ve torial real

R3 ,

os ve tores

v1 =
independentes.

(1, 1, 1), v2 = (1, 0, 1) e v3 = (0, 0, 1) so linearmente

De fa to, sejam

, , R

tais que

(1, 1, 1) + (1, 0, 1) + (0, 0, 1) = (0, 0, 0)


Ento

v1
dependentes.

os ve tores

=0
+ = 0
=0
=0

=0
++ = 0

= (1, 1, 1), v2 = (1, 0, 1) e v3 = (2, 1, 2) so linearmente

De fa to, sejam

, , R

tais que

(1, 1, 1) + (1, 0, 1) + (2, 1, 2) = (0, 0, 0).


Ento

+ + 2 = 0
+ =0

+ + 2 = 0

Passando matriz ampliada

1
1
1

1
0
1

1
2 0
2 0
1

2 := L2 L1 0
1 1 0
1 0 L
L3 := L3 L1
0
0
0 0
2 0

O sistema orrespondente, equivalente ao ini ial, :


Assim, para

R,

+ + 2 = 0

= 0

=
=

tem-se:

()(1, 1, 1) + ()(1, 0, 1) + (2, 1, 2) = (0, 0, 0).

4.4 Independn ia e dependn ia linear


Por exemplo, para

=1

74

tem-se

(1)(1, 1, 1) + (1)(1, 0, 1) + 1(2, 1, 2) = (0, 0, 0).


Existe assim uma ombinao linear nula no trivial, isto , em que
os es alares no so todos nulos.

Exer io 4.23.

Verique se os ve tores dados nos espaos ve toriais indi ados

so linearmente independentes.

(a) u = (1, 1, 1), v = (1, 1, 0) e w = (1, 0, 1) de R3 ;


(b) p(x) = x + x2

q(x) = x3

de

P3 [x];

( ) p(x) = 1 + x, q(x) = x + x2 , r(x) = x3


(d) f (x) = 1, g(x) = cos2 x

t(x) = 1 + 2x + x2

h(x) = sin2 x, x R,

de

de

P3 [x];

F (R).

As noes de independn ia e dependn ia lineares dizem respeito a um


onjunto de ve tores de um espao ve torial
um elemento? Veja-se o prximo resultado.

E.

E se esse onjunto apenas tiver

4.4 Independn ia e dependn ia linear


Proposio 4.24.
ainda

v E.

75

K =
6 {0K } e E um espao ve torial sobre K. Seja
v linearmente independente se e s se v 6= 0E .
linearmente dependente o ve tor nulo 0E .

Sejam

Ento o ve tor

Isto , o ni o ve tor
Demonstrao.

Note-se que 1K 6= 0K . De fa to, suponha-se,


1K = 0K . Ento, para todo K \ {0K },

()

um absurdo, que

om vista a

= 1K = 0K = 0K ,

1K 6= 0K .

v linearmente independente
v = 0E . Como 1K v = 1K 0E = 0E ,
existe uma ombinao linear nula no trivial. Logo v linearmente dependente.
Absurdo! Logo v 6= 0E .
() Suponha-se agora que v 6= 0E . Novamente, om vista a um absurdo,
suponha-se que v linearmente dependente. Ento existe K \ {0K } tal que
v = 0E . Como existe 1 K, tem-se que:
o que absurdo! Logo

Suponha-se que

e, om vista a um absurdo, suponha-se que

v = 0E 1 (v) = 1 0E ,
que, pela asso iatividade mista e pelas propriedades do espao ve torial, equivalente a:

E, portanto,

v = 0E ,


1 v = 0E 1K v = 0E .

o que absurdo pois ontraria a hiptese

linearmente independente.
A ombinao linear nula no trivial

0E

1K 0 E = 0 E

v 6= 0E .

Logo

mostra que o ve tor nulo

linearmente dependente.
Portanto,

o ve tor

linearmente independente se e s se

o ve tor

linearmente dependente se e s se

v 6= 0E .

v = 0E .

Para onjuntos no singulares de ve tores tem-se a seguinte proposio que


estabele e outra ara terizao de independn ia e dependn ia lineares.

Proposio 4.25.
om

kN

Sejam

k 2.

K e v1 , v2 , . . . , vk E ,
so linearmente dependentes

um espao ve torial sobre

Ento os ve tores

v1 , . . . , vk

se e s se pelo menos um deles ombinao linear dos restantes.


Demonstrao.

()

Suponha-se que

dentes. Ento, por denio, existem

v1 , . . . , vk E so linearmente depen1 , . . . , k K no todos nulos tais que

1 v1 + + k vk = 0E .
Sem perda de generalidade, suponha-se que
mas da denio de espao ve torial,

1 6= 0K .

Ento, apli ando os axio-

1 v1 = (2 v2 ) + (3 v3 ) + (k vk ),

4.4 Independn ia e dependn ia linear


e assim, omo existe

v1

11 ,

i = 11 i ,

tem-se que:

(11 2 )v2 + (11 3 )v3 (11 k )vk

2 v2 + 3 v3 + + k vk ,

=
om

76

para todo

restantes ve tores.

i {2, . . . , k}.

Donde

v1

ombinao linear dos

() Por hiptese, suponha-se que um dos ve tores dados ombinao linear


v1 ombinao linear
dos restantes, isto , existem 2 , . . . , k K tais que

dos restantes. Sem perda de generalidade, suponha-se que

v1 = 2 v2 + + k vk .
Apli ando os axiomas da denio de espao ve torial, obtm-se

1K v1 2 v2 k vk = 0E .
Ou seja, existe uma ombinao linear nula no trivial de

v1 , v2 , . . . , vk .

Logo os

ve tores so linearmente dependentes.

Exemplo 4.26.
e

v = (2, 4)

No espao ve torial real

R2 ,

j se viu que os ve tores

so linearmente dependentes. bvio que

Proposio 4.27.

Sejam

um espao ve torial sobre

ve tores linearmente independentes. Seja ainda


so linearmente dependentes. Ento

u = (1, 2)

v = 2u.

wE

K e v1 , v2 , . . . , vk E
tal que v1 , v2 , . . . , vk , w

ombinao linear de

v1 , v2 , . . . , vk .

v1 , v2 , . . . , vk , w so linearmente dependentes, existem es1 , 2 , . . . , k , k+1 K no todos nulos tais que

Demonstrao. Como
alares

1 v1 + 2 v2 + + k vk + k+1 w = 0E .
Por reduo ao absurdo, suponha-se que

k+1 = 0K .

Ento,

1 v1 + 2 v2 + + k vk = 0E ,

1 = 2 = = k = 0K pois v1 , . . . , vk
1 = 2 = = k = k+1 = 0K , o
k+1 6= 0K . Mas ento existe 1
k+1 e

o que impli a que

so linearmente

independentes. Logo

que ontraria a

hiptese. Donde

1
1
w = (1
k+1 1 )v1 + (k+1 2 )v2 + + (k+1 k )vk .
ou seja,

ombinao linear de

Observao 4.28.

v1 , v2 , . . . , vk .

Note-se que da proposio anterior, resulta que, se a um

v1 , v2 , . . . , vk E , juntarmos
w E que no ombinao linear de v1 , v2 , . . . , vk ento os ve tores
v1 , v2 , . . . , vk , w so linearmente independentes.
onjunto de ve tores linearmente independentes

um ve tor

4.4 Independn ia e dependn ia linear


Exemplo 4.29.
A=
As matrizes

M22 (R), onsidere as matrizes





4
0
1
e
C=
.
1 1
1

No espao ve torial real

1
0

5
2

B=

A, B

1
1

so linearmente independentes pois

A + B = 022

As matrizes

77

5 + = 0

= 0
= = 0.

2
+ =0

=0

so linearmente dependentes (verique!) e

C = A B.

Teorema 4.30.

Seja E um espao ve torial sobre K e sejam v1 , . . . , vk E . Se


v1 , . . . , vk so linearmente dependentes ento, para qualquer l N,
os ve tores v1 , v2 , . . . , vk , vk+1 , vk+2 , . . . , vk+l so linearmente dependentes, onde
vk+1 , vk+2 , . . . , vk+l E .

os ve tores

Demonstrao. Como

1 , . . . , k K

v1 , . . . , vk

so linearmente dependentes, existem es alares

no todos nulos tais que

1 v1 + 2 v2 + + k vk = 0E .
Logo

1 v1 + 2 v2 + + k vk + 0K vk+1 + + 0K vk+l = 0E .

uma ombinao linear nula no trivial, ou seja, os ve tores

vk+1 , vk+2 , . . . , vk+l

Corolrio 4.31.

v1 , v2 , . . . , vk ,

so linearmente dependentes.

Qualquer sub onjunto de um onjunto de ve tores linearmente

independentes ainda linearmente independente.

Corolrio 4.32.

Qualquer onjunto de ve tores que in lua o ve tor nulo li-

nearmente dependente.

Corolrio 4.33.

Qualquer onjunto de ve tores que in lua dois ve tores iguais

linearmente dependente.
As demonstraes destes orolrios  am omo exer io.

Observao 4.34.

Um onjunto de ve tores diz-se linearmente independente

(respe tivamente, dependente) se os ve tores que onstituem esse onjunto so


linearmente independentes (respe tivamente, dependentes).

4.5 Subespao gerado por ve tores

78

4.5 Subespao gerado por ve tores


Denio 4.35.

Sejam

K e v1 , v2 , . . . , vk E . Ao
v1 , v2 , . . . , vk , isto , ao onjunto

um espao ve torial sobre

onjunto de todas as ombinaes lineares de

G = {1 v1 + 2 v2 + + k vk : 1 , 2 , . . . , k K}

hama-se

subespao gerado por

v1 , v2 , . . . , vk

e representa-se por

hv1 , v2 , . . . , vk i.

Proposio 4.36.
Ento

Sejam

hv1 , v2 , . . . , vk i

um espao ve torial sobre

um subespao ve torial de

G = hv1 , v2 , . . . , vk i.

Demonstrao. Seja
Mais,

(i) G 6= uma vez que 0E

Ento

E.
G E

ombinao linear de

v1 , v2 , . . . , vk E .

por denio de

G.

v1 , v2 , . . . , vk :

0E = 0K v1 + + 0K vk .

(ii)

Sejam

u, v G.

Ento

1 , 2 , . . . , k K : u = 1 v1 + 2 v2 + + k vk
1 , 2 , . . . , k K :

v = 1 v1 + 2 v2 + + k vk

Assim,

u + v = (1 v1 + 2 v2 + + k vk ) + (1 v1 + 2 v2 + + k vk )
Apli ando os axiomas de espao ve torial e obtm-se:

u + v = 1 v1 + 2 v2 + + k vk ,
onde

i = i + i K,

para todo

fe hado para a adio.

(iii)

i {1, 2, . . . , k}.

Fi a omo exer io mostrar, de forma anloga, que

E, portanto,

K, u G, u G.
G = hv1 , v2 , . . . , vk i

fe hado para a

multipli ao por um es alar, isto ,

Pela Proposio 4.9,

subespao ve torial de

E.

4.5 Subespao gerado por ve tores


Exemplo 4.37.

79

R3 . Para determinar o subespao gerado pelos ve tores (1, 1, 1) e (1, 0, 1), ou seja, h(1, 1, 1), (1, 0, 1)i tem que
se atender denio: (x, y, z) h(1, 1, 1), (1, 0, 1)i se e s se existem es alares
1 , 2 R tais que
Considere o espao ve torial real

(x, y, z) = 1 (1, 1, 1) + 2 (1, 0, 1).


Ou seja, se e s se possvel o sistema nas in gnitas

1 + 2 = x
1 = y

1 + 2 = z

2 ,

Passando para a matriz ampliada, e es alonando obtm-se:

1
1
1

1 x
1
1

L2 := L2 L1
0 y L
0
1

3 := L3 L1
1 z
0
0

E, portanto, o sistema possvel se e s se

x
y x .
zx

z x = 0 z = x.

Ento

h(1, 1, 1), (1, 0, 1)i = {(x, y, z) R : z = x}.

Exer io Resolvido 4.38.


gerado pelos ve tores

(0, 1)

No espao ve torial real

R2 , determine o subespao

(0, 2).

h(0, 1), (0, 2)i determinado da


(x, y) h(0, 1), (0, 2)i se e s se existem 1 , 2 R

Resoluo: O subespao

seguinte forma: um ve -

tor

tais que

(x, y) = 1 (0, 1) + 2 (0, 2).


Ou seja, se e s se possvel o sistema nas in gnitas

2 ,

1 + 22 = y
0=x

Atendendo sua matriz ampliada,

1
0

2 y
0 x

o sistema anterior possvel se e s se

x = 0.

Ento

h(0, 1), (0, 2)i = {(x, y) R2 : x = 0}.

Exer ios 4.39.

1. No espao ve torial real

rado pelos ve tores

(1, 1, 0, 2)

R4 ,

determine o subespao ge-

(0, 1, 2, 3).

E um espao ve torial sobre K e sejam X e Y sub onjuntos nitos


E . Seja ainda S um subespao ve torial de E . Prove que:
(b) h0E i = {0E }
(a) hEi = E
( ) X hXi
(d) Se X Y ento hXi hY i
(e) Se X S ento hXi S .

2. Seja
de

4.6 Sistema de geradores

80

4.6 Sistema de geradores


Denio 4.40.

E um espao ve torial sobre K e sejam v1 , v2 , . . . , vk E .


v1 , v2 , . . . , vk so geradores de E (ou geram E ou ainda
formam um sistema de geradores de E ) se qualquer ve tor de E se es reve
omo ombinao linear dos ve tores v1 , v2 , . . . , vk , isto :
Seja

Diz-se que os ve tores

E = hv1 , v2 , . . . , vk i.

Exemplo 4.41.
v=

2
No espao ve torial real R , onsidere os ve tores u = (1, 1) e
2
(0, 1). Mostre-se que R = hu, vi, ou seja, que qualquer ve tor de R2 pode

u e v.
(a, b) ombinao linear de u
tais que (a, b) = u + v , ou seja:

es rever-se omo ombinao linear de


2
Seja (a, b) um ve tor de R . Ento
s se existem nmeros reais

se e

(a, b) = (1, 1) + (0, 1)


= (, ) + (0, )
= (, + ).
Donde

=a
+ =b

1 0
1 1

a
b

L2 := L2 L1

1 0
0 1

a
ba

que sempre um sistema possvel qualquer que seja (a, b) R2 , isto , qualquer
2
ve tor de R pode ser es rito omo ombinao linear dos ve tores u e v da
seguinte forma:

(a, b) = au + (b a)v = a(1, 1) + (b a)(0, 1).


Por exemplo,
Logo

(1, 1)

(13, 15) = 13(1, 1) + 2(0, 1).


e (0, 1) so geradores de R2 .

Observao 4.42.

Um espao ve torial E 6= {0E } om um nmero innito de


ve tores tem uma innidade de sistemas de geradores.
Por exemplo, verique que:

R2

=
=
=
=

Denio 4.43.

h(1, 1), (0, 1)i

h(1, 0), (0, 1)i


h(1, 2), (2, 3), (3, 4)i

h(0, 0), (2, 3), (3, 4), (0, 1)i

E diz-se nitamente
v1 , v2 , . . . , vk E que geram E ,

Um espao ve torial

um nmero nito de ve tores

E = hv1 , v2 , . . . , vk i.

gerado se existe
ou seja, tais que

4.7 Base e dimenso


Exemplo 4.44.

81

Atendendo ao exemplo anterior, o espao ve torial real

R2

nitamente gerado.

Exemplo 4.45.

O espao ve torial real

P [x]

no nitamente gerado. Para

provar esta armao, suponha-se por absurdo que

P [x] tem um sistema nito de

geradores p1 (x), . . . , pk (x). Seja n o mximo do graus de p1 (x), . . . , pk (x). Claran+1


mente, x
no pode ser es rito omo ombinao linear de p1 (x), . . . , pk (x).
Portanto,

P [x]

no nitamente gerado.

Observao 4.46.

Re orde-se que qualquer subespao ve torial um espao

ve torial, por denio. Logo todos on eitos e resultados vlidos para espaos
ve toriais so tambm vlidos para subespaos ve toriais.

Exer io Resolvido 4.47.


R

um subespao ve torial de

F = {(x, y, z, w) R4 : x + y + z + w = 0}
Mostre que F nitamente gerado.

Seja
.

F R4 .

Resoluo: Verique que

F nitamente gerado,
F.
x = y z w. Logo, os ve tores

Para mostrar que

ne essrio en ontrar um sistema de geradores de

(x, y, z, w) F ento
(y z w, y, z, w). Mas

Observe-se que se
de

so da forma

(y z w, y, z, w) = y(1, 1, 0, 0) + z(1, 0, 1, 0) + w(1, 0, 0, 1),


isto , qualquer ve tor de

(1, 1, 0, 0), (1, 0, 1, 0)

pode ser es rito omo ombinao linear dos ve tores

(1, 0, 0, 1),

que perten em a

F.

Portanto,

F = h(1, 1, 0, 0), (1, 0, 1, 0), (1, 0, 0, 1)i.


Como gerado por um nmero nito de ve tores (neste aso, apenas trs ve tores), ento

um (sub)espao ve torial nitamente gerado.

Exer io 4.48.

Mostre que o espao ve torial real

M22 (R)

nitamente

gerado, mostrando que as matrizes

 
1 0
0
,
0 0
0

 

1
0 0
,
0
1 0


0 0
.
0 1

so geradores desse espao ve torial.

4.7 Base e dimenso


Denio 4.49.

Seja

B = {e1 , e2 , . . . , en }

um espao ve torial nitamente gerado e seja ainda

um sub onjunto de

E.

Diz-se que

uma

base de

se

4.7 Base e dimenso


(i) B

82

um onjunto de ve tores linearmente independentes;

(ii) e1 , e2 , . . . , en

um sistema de geradores de

Observao 4.50.
uma base

B,

E,

isto ,

E = he1 , e2 , . . . , en i.

Quando dada uma ordenao espe  a aos elementos de

hama-se a

base ordenada, e representa-se por


B = (e1 , e2 , . . . , en ),

onde

ei

representa o i-simo ve tor da base.

Por onveno, o espao trivial

{0E }

.
{0E } = h0E i, porm,

tem omo base o onjunto vazio,

Note-se que este espao nitamente gerado, uma vez que

no existem ve tores linearmente independentes neste espao ve torial trivial.

Exemplo 4.51.

3
O onjunto B = {(1, 1, 1), (1, 0, 1), (1, 2, 0)} uma base de R .
3
Mostre-se primeiro que os ve tores (1, 1, 1), (1, 0, 1) e (1, 2, 0) geram R .
Seja (x, y, z) R3 e veja-se que possvel es rever (x, y, z) omo ombinao

linear dos ve tores indi ados, isto , veja-se que existem nmeros reais
e

1 , 2

tais que

(x, y, z) = 1 (1, 1, 1) + 2 (1, 0, 1) + 3 (1, 2, 0)


= (1 + 2 + 3 , 1 + 23 , 1 + 2 ).
O que equivalente a que seja possvel o sistema nas in gnitas

1 + 2 + 3 = x
1 + 23 = y

1 + 2 = z

1 , 2

Es alonando a orrespondente matriz ampliada, obtm-se:

1 1
1 0
1 1

1 x
1

L
:=
L

L
2 y L2 := L2 L1 0
3
1
3
0 z
0

1
1
x
1
1 yx
0 1 z x

E, portanto, este sistema sempre possvel para todo (x, y, z) R3 , ou seja,


3
qualquer (x, y, z) R ombinao dos ve tores (1, 1, 1), (1, 0, 1) e (1, 2, 0).
Logo esses ve tores geram R3 .
Falta provar que estes ve tores so linearmente independentes, o que orresponde a provar que h uma ni a maneira de es rever o ve tor nulo omo
ombinao linear destes ve tores. Suponha-se que

(0, 0, 0) = 1 (1, 1, 1) + 2 (1, 0, 1) + 3 (1, 2, 0).


Repare-se que esta equao orresponde equao anterior substituindo

x, y e
x, y

por zero. Assim substituindo, na matriz ampliada do sistema anterior,

e z por zero, on lui-se que 1 = 0, 2 = 0 e 3 = 0. Logo, os ve tores so


linearmente independentes e, portanto, B = {(1, 1, 1), (1, 0, 1), (1, 2, 0)} uma
base de R3 .

4.7 Base e dimenso


Exer io 4.52.

83

Mostre que

{1, x, x2 }

uma base de

P2 [x].

O prximo resultado til para onstruir uma base para um espao ve torial
nitamente gerado.

Teorema 4.53.
que, para algum
Ento

Sejam E um espao ve torial sobre K e v1 , v2 , . . . , vk E tais


i {1, . . . , k}, vi ombinao linear dos restantes ve tores.

hv1 , . . . , vi1 , vi , vi+1 , . . . , vk i = hv1 , . . . , vi1 , vi+1 , . . . , vk i.


Demonstrao. Como

vi

ombinao linear dos restantes ve tores, tem-se que

vi = 1 v1 + + i1 vi1 + i+1 vi+1 + + k vk ,


para alguns

1 , . . . , i1 , i+1 , . . . , k K.

(4.1)

Seja

x hv1 , . . . , vi1 , vi , vi+1 , . . . , vk i.


Ento

x = 1 v1 + + i1 vi1 + i vi + i+1 vi+1 + + k vk ,


Para alguns

1 , . . . , k K.

Veja-se que

(4.1)) e (4.2) obtm-se que

(4.2)

x hv1 , . . . , vi1 , vi+1 , . . . , vk i.

De

x = 1 v1 + + i1 vi1 + i (1 v1 + + i1 vi1 + i+1 vi+1 + + k vk )


+ i+1 vi+1 + + k vk
= 1 v1 + + i1 vi1 + i+1 vi+1 + + k vk ,

onde

j = j + i j ,

para

j {1, . . . , i 1, i + 1, . . . , k}.

Con lui-se assim que

hv1 , . . . , vi1 , vi , vi+1 , . . . , vk i hv1 , . . . , vi1 , vi+1 , . . . , vk i.


Para mostrar a in luso ontrria onsidere-se
Logo

x hv1 , . . . , vi1 , vi+1 , . . . , vk i.

x = 1 v1 + + i1 vi1 + i+1 vi+1 + + k vk


= 1 v1 + + i1 vi1 + 0K vi + i+1 vi+1 + + k vk ,
para alguns

1 , . . . , i1 , i+1 , . . . , k K.

Donde

x hv1 , . . . , vi1 , vi , vi+1 , . . . , vk i.


Consequentemente

hv1 , . . . , vi1 , vi+1 , . . . , vk i hv1 , . . . , vi1 , vi , vi+1 , . . . , vk i.


Portanto,

hv1 , . . . , vi1 , vi , vi+1 , . . . , vk i = hv1 , . . . , vi1 , vi+1 , . . . , vk i.

4.7 Base e dimenso


Exemplo 4.54.

84
R2 = h(1, 1), (1, 0), (0, 1)i
(1, 0) e (0, 1) uma vez que

Sabe-se que

ombinao linear de

(verique!). Como

(1, 1)

(1, 1) = (1, 0) + (0, 1)


ento

R2 = h(1, 0), (0, 1)i.

Por outro lado, os ve tores


(prove!) ento

B = ((1, 0), (0, 1))

Proposio 4.55.

(1, 0)

(0, 1)

so linearmente independentes
R2 .

uma base ordenada de

Todo o espao ve torial nitamente gerado tem uma base.

Demonstrao. Seja

um espao ve torial nitamente gerado. Se

ento tem omo base o onjunto vazio. Portanto pode-se assumir que
Ento existem ve tores

u1 , u2 , . . . , un E

tais que

E = {0E }
E 6= {0E }.

E = hu1 , u2 , . . . , un i.
Como

E 6= {0E },

ento um destes ve tores diferente do ve tor

ve tor linearmente independente. Se


dentes ento formam uma base de

u1 , u2 , . . . , un

0E ,

logo esse

so linearmente indepen-

e o resultado  a demonstrado. Caso sejam

linearmente dependentes ento existe

i {1, . . . , n} tal que ui

ombinao lin-

ear dos restantes. Pelo teorema anterior,

E = hu1 , . . . , ui1 , ui+1 , . . . , un i.


Agora, se

u1 , . . . , ui1 , ui+1 , . . . , un so linearmente independentes ento obtmE ; aso ontrrio, repete-se o pro edimento anterior. Como E

se uma base de

tem um nmero nito de geradores este pro edimento repetido at produzir

{u1 , u2 , . . . , un } formado por ve tores linearmente


E e, portanto, onstituem uma base de E .

um sub onjunto de
dentes que geram

Corolrio 4.56.
geradores de

Seja

um espao ve torial sobre

ontm uma base de

K.

indepen-

Qualquer sistema de

E.

Nos exemplos que se seguem apresenta-se um pro esso para, a partir de um


sistema de geradores de um espao ve torial nitamente gerado, onstruir uma
base desse espao ve torial.

Exemplo 4.57.

R3 = h(1, 0, 1), (0, 1, 1), (1, 1, 1), (1, 2, 3)i (veri3


des obrir uma base de R ontida em

Sabe-se que

que!) e pretende-se

S = {(1, 0, 1), (0, 1, 1), (1, 1, 1), (1, 2, 3)}.


Come e-se por veri ar se os ve tores de
Sejam

1 , 2 , 3 , 4 R

so linearmente independentes.

tais que

1 (1, 0, 1) + 2 (0, 1, 1) + 3 (1, 1, 1) + 4 (1, 2, 3) = (0, 0, 0).

4.7 Base e dimenso

85

Esta igualdade equivalente ao sistema

1 + 3 4 = 0
2 + 3 + 24 = 0

1 2 + 3 + 34 = 0

Es alonando a matriz ampliada asso iada, obtm-se

1
0
1

0 1 1 0
1 1
2 0
1 1
3 0

Donde se obtm

1
0
L3 := L3 L1
0

0
L3 := L3 + L2
0

L1 := L1 L3
0

L2 := L2 L3
0

0 1
1 1
1 0

0
1
0

1
1
1

0
1
0

0
0
1

1 = 74
2 = 44

3 = 64

1 0
2 0
4 0

1 0
2 0
6 0

7 0
4 0
6 0

Como este sistema admite pelo menos uma soluo no nula (por exemplo,

1 = 7, 2 = 4, 3 = 6

e 4 = 1) os ve tores onsiderados so linearmente dependentes. Logo um deles pode es rever-se omo ombinao linear dos

restantes.
Da soluo no nula onsiderada, obtm-se

7(1, 0, 1) + 4(0, 1, 1) 6(1, 1, 1) + (1, 2, 3) = (0, 0, 0).


Donde resulta que

(1, 2, 3)

(1, 2, 3) = 6(1, 1, 1)7(1, 0, 1)4(0, 1, 1), ou seja, o ve tor


(1, 0, 1), (0, 1, 1) e (1, 1, 1). Como

ombinao linear dos ve tores

R3 = h(1, 0, 1), (0, 1, 1), (1, 1, 1), (1, 2, 3)i


ento

R3 = h(1, 0, 1), (0, 1, 1), (1, 1, 1)i .


(1, 0, 1), (0, 1, 1)
1 , 2 , 3 R tais que

Veja-se, agora, se os ve tores


dependentes. Sejam

(1, 1, 1)

so linearmente in-

1 (1, 0, 1) + 2 (0, 1, 1) + 3 (1, 1, 1) = (0, 0, 0).


Esta igualdade equivalente ao sistema

1 = 0
1 + 3 = 0
1 = 3
2 = 0
2 + 3 = 0
2 = 3

3 = 0
3 = 0
1 2 + 3 = 0

4.7 Base e dimenso


Ento os ve tores
portanto,

uma base de

86

(1, 0, 1), (0, 1, 1) e (1, 1, 1)

so linearmente independentes e,

B = {(1, 0, 1), (0, 1, 1), (1, 1, 1)}

R3 .

Observao 4.58.

Rn ,

Considere-se o espao ve torial real

e1

(1, 0, 0, . . . , 0, 0, 0)

e2

(0, 1, 0, . . . , 0, 0, 0)

en1

(0, 0, 0, . . . , 0, 1, 0)

en

(0, 0, 0, . . . , 0, 0, 1)

para

n N.

Sejam

.
.
.

Fa ilmente se prova que estes ve tores so linearmente independentes e que


Rn , pelo que formam uma base de Rn . A esta base ordenada hama-se
Rn , e representa-se por BRn :

geram

base anni a de

BRn = (e1 , e2 , . . . , en )
Alm da base anni a, existem outras bases para

Rn .

v1
v2

=
=

(1, 1, 1, . . . , 1, 1)
(0, 1, 1, . . . , 1, 1)

v3

(0, 0, 1, . . . , 1, 1)

(0, 0, 0, . . . , 0, 1),

Por exemplo, os ve tores

.
.
.

vn
tambm onstituem uma base

Exer io 4.59.
de

Pn [x],

B = {v1 , v2 , . . . , vn }

de

Rn

BPn [x] = (1, x, x2 , . . . , xn )


hama base anni a de Pn [x].

uma base (ordenada)

Mostre que

qual se que

Na observao anterior en ontrou-se duas bases de

(prove!).

Rn

om

ve tores. De

fa to, todas as bases de um espao ve torial tm o mesmo nmero de ve tores:

Proposio 4.60.
E.

Ento

B1

B2

Seja

um espao ve torial e sejam

tm o mesmo nmero de ve tores.

B1

B2

duas bases de

Atendendo a este resultado pode denir-se o seguinte on eito:

Denio 4.61.
ve torial

Ao nmero de ve tores de uma qualquer base de um espao

hama-se

dimenso de

e representa-se por

dim E .

4.7 Base e dimenso

87

Exemplo 4.62.
lar

dim R = 2

dim Rn = n,

Pela observao 4.58, on lui-se que


3
e dim R = 3.

Exer io 4.63.
Exemplo 4.64.

Justique que

Seja

ve torial

m R.

em parti u-

dim Pn [x] = n + 1.

No espao ve torial

R2 ,

onsidere-se o subespao

F = {(x, y) R2 : y = mx} = {(x, mx), x R}.


(x, mx) = x(1, m), para qualquer x R. Donde F = h(1, m)i. Como
(1, m) 6= (0, 0) ento (1, m) linearmente independente. Assim, B = ((1, m))
uma base ordenada de F e dim F = 1.
Ento

Exer io Resolvido 4.65.

3
Seja A = {(x, y, z) R : x = 0}. Sabendo que
3
um subespao ve torial de R , determine a dimenso de A.
Resoluo: Seja

v A,

ento existem nmeros reais

tais que

v = (0, y, z) = y(0, 1, 0) + z(0, 0, 1)


v A se es reve omo ombinao linear de v1 = (0, 1, 0)
v2 = (0, 0, 1). Logo A h(0, 1, 0), (0, 0, 1)i. Por outro lado, omo (0, 1, 0) e
(0, 0, 1) perten em a A, todas as ombinaes lineares de (0, 1, 0) e (0, 0, 1) tambm perten em a A, pois A fe hado para a adio e para a multipli ao por um
es alar. Logo h(0, 1, 0), (0, 0, 1)i A e, portanto, A = h(0, 1, 0), (0, 0, 1)i. Alm
disso, v1 e v2 so linearmente independentes (verique!). Logo B = (v1 , v2 )
uma base ordenada de A. Como qualquer base tem o mesmo nmero de elementos, tem-se que dim A = 2.
Ento qualquer ve tor
e

Exer io Resolvido 4.66.


ou igual a

n,

om

n N,

Seja

o onjunto dos polinmios de grau menor

om termo independente nulo, ou seja,

G = {p(x) Pn [x] : p(0) = 0}.


Mostre que

um subespao ve torial de

Resoluo: Mostre-se que

(i)
(ii)

Pn [x]

e determine a dimenso de

um subespao ve torial de

Claramente que o polinmio nulo perten e a


Sejam

p(x), q(x) G.

Ento

p(0) = 0

G.

Pn [x].

Donde

q(0) = 0.

Logo

fe hado para a adio de polinmios.

G 6= .

(p + q)(x) G

(p + q)(0) = p(0) + q(0) = 0 + 0 = 0


E, portanto,

G.

pois

4.7 Base e dimenso


(iii)

88

Analogamente, sejam

pois

p(x) G e R.

p(0) = 0. Donde (p)(x)

Ento

(p)(0) = (p(0)) = 0 = 0
Logo

fe hado para a multipli ao por um nmero real.

Portanto, pela Proposio 4.9,

r(x) G.

Seja

um subespao ve torial de

Pn [x].

Ento

r(x) = an xn + an1 xn1 + + a2 x2 + a1 x,


xn , xn1 , . . . , x2 , x

isto , os ve tores

geram

G.

Mais, estes ve tores so linear-

mente independentes pois formam um sub onjunto da base anni a de


Logo uma base ordenada de

Pn [x].

B = (xn , xn1 , . . . , x2 , x)
e, onsequentemente,

Exer io 4.67.
E22 =

0 0
0 1

dim G = n.

Sejam

E11 =

1 0
0 0

E12 =

0 1
0 0

E21 =

0
1

0
0

Mostre que Bc = (E11 , E12 , E21 , E22 ) uma base de M22 (R) ( qual se
hama base anni a de M22 (R)) e on lua que dim M22 (R) = 4.
O prximo resultado estabele e o nmero mnimo de geradores e o nmero
mximo de ve tores linearmente independentes num espao ve torial, atendendo
dimenso desse espao ve torial.

Teorema 4.68.
i. Quaisquer

Seja

um espao ve torial sobre

ve tores de

tal que

dim E = n.

Ento:

linearmente independentes formam uma base

E;

de

ii. Qualquer sistema de geradores de


de

om

om mais de

elementos formam uma base

E;

iii. Qualquer onjunto de ve tores de

elementos linear-

mente dependente.

Observao 4.69.
n

ve tores

base de

dim E = n, dados
B = (v1 , v2 , . . . , vn ) uma

Pelo teorema anterior, se se souber que

v1 , v2 , . . . , vn E ,

para se veri ar que

basta veri ar apenas uma das seguintes ondies:

v1 , v2 , . . . , vn

so linearmente independentes;

v1 , v2 , . . . , vn

geram

E.

4.7 Base e dimenso


Corolrio 4.70.

89

Seja

um espao ve torial sobre

qualquer onjunto de ve tores de


dido a uma base de

tal que

dim E = n.

Ento

linearmente independentes pode ser exten-

E.

u1 , u2 , . . . , uk E ve tores linearmente independentes.


k = n, pelo teorema 4.68, (u1 , u2 , . . . , uk ) uma base de E . Se k < n,
u1 , u2 , . . . , uk no formam uma base de E e, portanto, no geram E . Logo existem um ve tor uk+1 E , que no ombinao linear de u1 , u2 , . . . , uk . Pela
observao 4.28, tem-se que u1 , u2 , . . . , uk , uk+1 so linearmente independentes.
Se k + 1 = n ento (u1 , u2 , . . . , uk , uk+1 ) uma base de E , aso ontrrio,
repete-se o pro esso adi ionando su essivamente um ve tor at obter-se n ve -

Demonstrao. Sejam
Se

tores linearmente independentes.

Exemplo 4.71.

Sejam

u = (1, 0, 1), v = (1, 1, 0)

w = (0, 1, 1).

Mostre-se que

B = ((1, 0, 1), (1, 1, 0), (0, 1, 1))


uma base ordenada de

R3 .

Como

dim R3 = 3,

de a ordo om o teorema an-

terior, basta, por exemplo, ver se os ve tores so linearmente independentes.


Sejam

1 , 2 , 3 R,

tais que

1 (1, 0, 1) + 2 (1, 1, 0) + 3 (0, 1, 1) = (0, 0, 0).


Ento

1 + 2 = 0
2 + 3 = 0

1 + 3 = 0

Resolvendo o sistema, obtm-se

1 = 0
2 = 0

3 = 0

Logo, os ve tores so linearmente independentes e, portanto, formam uma base


R3 .

de

Exer io Resolvido 4.72.


que

S.

S = {(x, y, z) R3 : x y + 3z = 0}. Mostre


de R3 e determine um sistema de geradores de

Seja

um subespao ve torial

Verique se esse onjunto formado por ve tores linearmente independentes

e indique, justi ando, a dimenso de

S.

Resoluo: Tem-se que:

= {(x, y, z) R3 : x y + 3z = 0}

= {(y 3z, y, z) : y, z R}
= {y(1, 1, 0) + z(3, 0, 1) : y, z R}

= h(1, 1, 0), (3, 0, 1)i.

4.8 Coordenadas de um ve tor relativamente a uma base


S um subespao
(1, 1, 0) e (3, 0, 1).

E, portanto,
ve tores

ve torial de

R3
S

Veja-se se estes ve tores geradores de

90

pois o subespao gerado pelos


so linearmente independentes.

Suponha-se que

(1, 1, 0) + (3, 0, 1) = (0, 0, 0),


, R. Ento, obtm-se = 0 e = 0. Portanto, os ve tores (1, 1, 0)
(3, 0, 1) so linearmente independentes. Logo formam uma base.
Como B = ((1, 1, 0), (3, 0, 1)) uma base de S ento dim S = 2.
om

Exer io 4.73.
V = {(x, y, z)

No espao ve torial real R3 , onsidere os subespaos ve toriais


R3 : 2x y + 3z = 0} e W = {(x, y, z) R3 : y = 5x}. Indique,

justi ando, a dimenso de

e de

W.

4.8 Coordenadas de um ve tor relativamente a


uma base
Proposio 4.74.

Seja

B = {e1 , e2 , . . . , en }

uma base de

um espao ve torial sobre

E.

de dimenso

Ento qualquer ve tor

forma ni a omo ombinao linear dos ve tores da base


es alares ni os

1 , 2 , . . . , n K

tais que

uE
B , ou

e seja

se es reve de
seja, existem

u = 1 e1 + 2 e2 + + n en .
Demonstrao. Como

B = {e1 , e2 , . . . , en }

uma base de

ento

E = he1 , e2 , . . . , en i.
Logo qualquer elemento de

e1 , e2 , . . . , en .

Seja

uE

se es reve omo ombinao linear dos ve tores

e suponha-se que

u = 1 e1 + 2 e2 + + n en ,
om

1 , 2 . . . , n K,

u = 1 e 1 + 2 e 2 + + n e n .
om

1 , 2 . . . , n K .

Ento

1 e1 + 2 e2 + + n en = 1 e1 + 2 e2 + + n en
ou seja,

(1 1 )e1 + (2 2 )e2 + + (n n )en = 0E .

Como

e1 , e2 , . . . , en

so linearmente independentes, tem-se que:

i i = 0K i = i ,
para qualquer

i {1, 2, . . . , n}. Con lui-se assim que h


u omo ombinao linear dos ve tores

maneira de es rever

apenas uma ni a
da base

B.

4.8 Coordenadas de um ve tor relativamente a uma base


Denio 4.75.
(e1 , e2 , . . . , en )

Seja

91

n. Sejam ainda B =
n-uplo (1 , 2 , . . . , n ), de

um espao ve torial de dimenso

uma base ordenada de

u E.

Ao

es alares univo amente determinados, tais que

u = 1 e1 + 2 e2 + + n en

oordenadas (ou omponentes) de u na base (ou relativamente


base) B e es reve-se

hama-se

u = (1 , 2 , . . . , n )B .

Exemplo 4.76.
real

R2 .

Seja

Sabe-se que

(x, y) R2 .

B = ((1, 2), (3, 1)) uma base

do espao ve torial

Para determinar as oordenadas deste ve tor na base

tem de se determinar os es alares

, R

tais que

(1, 2) + (3, 1) = (x, y),


isto ,

+ 3 = x
2 = y

Passando para a matriz ampliada e es alonando, obtm-se

Donde

1
3
2 1

2xy
e
7

x
y

1
3
x
0 7 y 2x



1 3
x
L2 := 17 L2
0 1 2xy
7

x+3y 

1
0
7
.
L1 := L1 3L2
0 1 2xy
7

L2 := L2 2L1

x+3y
7 . Portanto,

(x, y) =
Observe-se que, para

x + 3y 2x y
,
7
7

(x1 , . . . , xn ) Rn ,

(x1 , x2 , . . . , xn ) = (x1 , x2 , . . . , xn )BRn ,


BRn a base
a1 x + a0 Pn [x],
onde

anni a de

Rn .

Assim omo, para

an xn + an1 xn1 + +

an xn + an1 xn1 + + a1 x + a0 = (a0 , a1 , . . . , an1 , an )BPn [x] ,


onde

BPn [x]

a base anni a de

Exer io Resolvido 4.77.

Pn [x].

No espao ve torial real

R4

onsidere-se a base

B = ((1, 1, 0, 0), (0, 1, 1, 0), (1, 0, 0, 0), (0, 0, 0, 1)) .

4.9 Interse o, reunio e soma de subespaos


u = (1, 3, 2, 0)

(a) Determine as oordenadas de


(b) Indique o ve tor

v R4

tal que

92

relativamente base

B.

v = (1, 2, 3, 1)B .

Resoluo:

(a) Sejam

1 , 2 , 3 , 4 R4

tais que

(1, 3, 2, 0) = 1 (1, 1, 0, 0) + 2 (0, 1, 1, 0) + 3 (1, 0, 0, 0) + 4 (0, 0, 0, 1),


ento

1 + 3 = 1
3

1 + 2 = 3
1

2 = 2
2

4 = 0
4

Portanto,

= 2
=1
=2
=0

(1, 3, 2, 0) = (1, 2, 2, 0)B .

(b) Como

v = (1, 2, 3, 1)B

ento

v = 1(1, 1, 0, 0) + 2(0, 1, 1, 0) + 3(1, 0, 0, 0) + 1(0, 0, 0, 1) = (2, 1, 2, 1).

Exer io 4.78.

P3 [x], onsidere as bases


B2 = (2, x 3x3 , 2x2 + x3 , x x2 ).

No espao ve torial real

B1 = (1 x, x + x2 , x2 , x3 )

(a) Determine as oordenadas dos polinmios


x2 + 3x + 2 nas bases dadas.
(b) Indique o polinmio

r(x) P3 [x]

tal que

ordenadas

p(x) = x3 4x2 x

q(x) =

r(x) = (1, 2, 0, 1)B2 .

4.9 Interse o, reunio e soma de subespaos


Denio 4.79.

Seja E um espao ve torial sobre K e sejam F e G subespaos


E . Dene-se a interse o dos subespaos F e G, e representaF G, omo sendo o sub onjunto de E denido por:

ve toriais de
se por

F G = {u E : u F u G}.

Proposio 4.80.
paos ve toriais de
Demonstrao.

Seja

E.

(i)

um espao ve torial sobre

Ento

Como

F G

0E F G

e sejam

so subespaos ve toriais de

0E F
Logo

um subespao ve torial de

e, portanto,

0E G.

F G 6= .

F
E.

ento:

subes-

4.9 Interse o, reunio e soma de subespaos


(ii)

Sejam

u, v F G.

Por denio de interse o de

uF G uF
v F G v F

Por outro lado,

(iii)

u + v G,

pelo que

G so subespaos
u + v F G.

De forma anloga, seja

e seja

Logo, pela Proposio 4.9,

Exemplo 4.81.

G,

vem que:

E,

logo

u+v F

Ento

u G.
E

so subespaos ve toriais de

u F

vG

u F G.
e

uG

e
e

ve toriais de

uF GuF
Mas omo

93

ento

u G u F G.

F G

um subespao ve torial de

E.

No espao ve torial real R3 , onsidere os subespaos ve toriais


R3 : x+y+3z = 0} e G = h(1, 0, 1), (1, 1, 2)i. Para determinar

F = {(x, y, z)
F G determine-se

primeiro a ondio que dene

G:

(x, y, z) G 1 , 2 R : (x, y, z) = 1 (1, 0, 1) + 2 (1, 1, 2)


Ou seja, se e s se a matriz ampliada asso iada orresponde a um sistema
possvel:

1 1 x
1 1
x

0
1 y L3 := L3 L1 0
1
y
1
2 z
0
3 zx

1 1
x

1
y
L3 := L3 3L2 0
0
0 z x 3y

Para que o sistema seja possvel a ondio que se impe


G = {(x, y, z) R3 : z x 3y = 0}. Donde

Assim,

z x 3y = 0.

F G = {(x, y, z) R3 : x + y + 3z = 0 z x 3y = 0}.
Resolvendo o sistema


obtm-se que

x + y + 3z = 0

z x 3y = 0

x = 25 y

z = x + 3y

x = 52 y
,
z = 21 y




5
1
F G=
y, y, y : y R .
2
2

4.9 Interse o, reunio e soma de subespaos


Exer io 4.82.

No espao ve torial real

U = {(x, y, z) R3 : x = y + 2z}
Determine uma base de

Denio 4.83.
ve toriais de

E.

Seja

R3 ,

onsidere os subespaos ve toriais

V = h(1, 0, 1), (2, 0, 4), (0, 3, 1)i.

U V.

um espao ve torial sobre

reunio dos subespaos

o sub onjunto de

94

denido por:

e sejam

subespaos

e G, representa-se por F G,

F G = {u E : u F u G}.
O exemplo que se apresenta a seguir permite on luir que, em geral, a reunio
de subespaos ve toriais de um espao ve torial
de

no um subespao ve torial

E.

Exemplo 4.84.

No espao ve torial real

R2 ,

onsidere-se os subespaos ve to-

riais

H = {(0, y) : y R}
Repare-se que
ve torial de

H F = {(x, y) R2 : x = 0 y = 0}

no um subespao

pois no fe hado para a adio de ve tores. De fa to,

(0, 1) H F
e, no entanto,

F = {(x, 0) : x R}

(1, 0) H F

(0, 1) + (1, 0) = (1, 1)


/ H F.

A proposio que se segue estabele e uma ondio ne essria e su iente


para que a reunio de subespaos seja um subespao ve torial.

Proposio 4.85.

Seja

espaos ve toriais de

E.

se

um espao ve torial sobre

Ento

F G

F G
Demonstrao.

()

ou

e, no entanto,

f F : f
/G
f, g F G,

F *G

G * F.

sub-

Ento,

F G

um

F G um subespao ve torial, fe hado para a adio e, omo


f + g = s F G. Tem-se ento que s F ou s G. Assim,
ento

g = s f F,

o que absurdo!

se

sG

ento

f = s g G,

o que absurdo!

ou

G F.

F G ento F G = G, que um subespao ve torial de E .


G F ento F G = F , que tambm um subespao
de E .

Se

Analogamente, se
ve torial

se s

ento

sF

()

g G : g
/ F.

se

F G

G F.

Logo

e sejam

Suponha-se, om vista a um absurdo, que

subespao ve torial de

Uma vez que

um subespao ve torial de

4.9 Interse o, reunio e soma de subespaos


Teorema 4.86.

Seja

um espao ve torial sobre

E.
F = E.

um subespao ve torial de
se

dim F = n

ento

Por onveno diz-se que


ve torial de

F 6= {0E }

Denio 4.87.
ve toriais de

E.

Seja

o sub onjunto de

Ento

n e seja F
dim F n. Mais,

de dimenso

tem dimenso nita e

dim{0E } = 0. Observe-se que se F um subespao


dim F 1. Portanto, dim F = 0 F = {0E }.

ento

K e sejam F e G subespaos
G, representa-se por F + G,

um espao ve torial sobre

soma dos subespaos

95

dado por:

F + G = {u + v : u F v G}.

Proposio 4.88.
Demonstrao.

E um espao ve torial sobre K e sejam F e G


E . Ento F + G um subespao ve torial de E .

Seja

subespaos ve toriais de

(i)

Como

so subespaos ve toriais de

0E F
Logo

(ii)

0E = 0E + 0E F + G.

Sejam

Assim,

u, v F + G.

0E G.

E, portanto,

F + G 6= .

u = u1 + u2 ,

tais que

v = v1 + v2 ,

tais que

u + v = (u1 + v1 ) + (u2 + v2 ).
| {z } | {z }

u1 F, u2 G

v1 F, v2 G.
Logo

u + v F + G.

K e u F + G. Ento u = u1 + u2 para alguns u1 F


u = (u1 + u2 ) = u1 + u2 F + G.
|{z} |{z}

Sejam
Logo

ento

Ento

(iii)

Logo, pela Proposio 4.9,

Exemplo 4.89.

dois

F +G

G
um subespao ve torial de

No espao ve torial real

R3 ,

onsidere os seguintes subespaos

G = {(0, y, 0) : y R}

Ento

F +G =
=
=

u2 G.

E.

ve toriais:

F = {(0, 0, z) : z R}

{(0, 0, z) + (0, y, 0) : y, z R}

{(0, y, z) : y, z R}

{(x, y, z) R3 : x = 0}.

4.9 Interse o, reunio e soma de subespaos

96

Observao 4.90.
de
de

Seja E um espao ve torial sobre K e seja X um sub onjunto


E . Prova-se que hXi o menor (no sentido da in luso) subespao ve torial
E que ontm X , isto :
se

subespao ve torial de

tal que

XH

ento

hXi H

Atendendo observao anterior, pode provar-se que a soma de dois subespaos o menor subespao que ontm a unio desses subespaos, isto :

Proposio 4.91.
paos ve toriais de

Seja

E.

um espao ve torial sobre

Ento

e sejam

subes-

F + G = hF Gi.

F + G = hF Gi, e atendendo observao


F + G o menor subespao ve torial que ontm
F G. Ou seja, tem que se mostrar que: (i) F G F + G e (ii) se H
um subespao ve torial de E tal que tal que F G H ento F + G H .

Demonstrao. Para provar que


anterior, basta mostrar que

(i)

Seja

u F G.

Ento

uF

ou

u G.

Se

uF

tem-se

u = u + 0E F + G.
uG
F G F + G.

Analogamente, se
que

(ii)

tem-se

Seja H um subespao ve torial


u F + G arbitrrio. Ento:

u = u1 + u2 ,
Como

u = 0E + u F + G.
de

tais que

tal que

u1 F,

Ou seja provou-se

F G H.
e

Seja ainda

u2 G

F G H , ento u1 , u2 H e, portanto, omo H


u = u1 + u2 H . Logo F + G H .

subespao

ve torial, ento

Teorema 4.92.

E um espao ve torial sobre K e sejam F e G subespaE tais que F = hu1 , . . . , un i e G = hv1 , . . . , vk i, para alguns
u1 , . . . , un , v1 , . . . , vk E . Ento
Seja

os ve toriais de

F + G = hu1 , . . . , un , v1 , . . . , vk i.

Exemplo 4.93.

No espao ve torial

R3

onsidere-se os subespaos ve toriais

F = {(x, y, z) R : x + y + 3z = 0}
Determine-se um sistema de geradores de
de

G,

G = h(1, 0, 1), (1, 1, 2)i

F +G.

J so onhe idos os geradores

basta agora determinar um sistema de geradores de

F:

= {(x, y, z) R : x = y 3z}
= {(y 3z, y, z) : y, z R}

= {y(1, 1, 0) + z(3, 0, 1) : y, z R}
= h(1, 1, 0), (3, 0, 1)i
Logo

F + G = h(1, 1, 0), (3, 0, 1), (1, 0, 1), (1, 1, 2)i.

4.10 Teorema das dimenses


Exer io 4.94.

No espao ve torial

S
T
S +T

Determine

97
R4

onsidere-se os subespaos ve toriais:

{(a, b, c, d) R4 : a b = 0 a = b + d}
h(1, 0, 0, 3), (2, 0, 0, 1)i

=
=

e indique uma sua base.

4.10 Teorema das dimenses


Teorema 4.95
um orpo

(Teorema das dimenses)

Sejam

subespaos ve toriais de

E,

um espao ve torial sobre

nitamente gerados. Ento

dim(F + G) = dim F + dim G dim(F G).


F = {0E } ou G = {0E }.
F + G = F . Como dim{0E } = 0, a

Demonstrao. Considere-se o aso parti ular em que


Neste aso,

F G = {0E }

F +G= G

ou

igualdade veri a-se trivialmente.

F 6= {0E } e G 6= {0E }. Repare-se que F e G tm


F G um subespao ve torial de F (e tambm de G)

Suponha-se ento que


dimenso nita; mais,

e, portanto, tambm tem dimenso nita.

F G 6= {0E } e seja BF G = (e1 , . . . , es ) uma base ordenada


e1 , . . . , es F e so linearmente independentes, pelo orolrio
4.70 possvel juntar ve tores de F base de F G por forma a obter uma base
de F . O mesmo se pode fazer para G. Assim, sejam BF = (e1 , . . . , es , t1 , . . . , tp ) e
BG = (e1 , . . . , es , g1 , . . . , gk ) bases ordenadas de F e G, respe tivamente. Ento
Suponha-se que

de

F G.

Como

F = he1 , . . . , es , t1 , . . . , tp i,
G = he1 , . . . , es , g1 , . . . , gk i
e

F + G = he1 , . . . , es , t1 , . . . , tp , g1 , . . . , gk i.
Prove-se que os ve tores

e 1 , . . . , e s , t1 , . . . , t p , g 1 , . . . , g k

so linearmente indepen-

dentes. Para tal onsidere-se a ombinao linear nula,

1 e1 + + s es + 1 t1 + + p tp + 1 g1 + + k gk = 0E ,

(4.3)

que equivalente a

1 e1 + + s es + 1 t1 + + p tp = (1 g1 ) + + (k gk ) .
{z
}
{z
} |
|
F =he1 ,...,es ,t1 ,...,tp i

Logo

G=he1 ,...,es ,g1 ,...,gk i

1 e1 + +s es +1 t1 + +p tp F G. Mas, omo BF G = (e1 , . . . , es )


F G, ento:

uma base de

1 e1 + + s es + 1 t1 + + p tp = 1 e1 + + s es ,

4.10 Teorema das dimenses


para alguns

1 , . . . , s K.

98

Assim,

(1 1 )e1 + + (s s )es + 1 t1 + + p tp = 0E .
e 1 , . . . , e s , t1 , . . . , t p
F , tem-se que:

Uma vez que


uma base de

so linearmente independentes pois formam

1 1 = = s s = 1 = = p = 0K .
Con lui-se assim que

1 = = p = 0K .

Substituindo em (4.3), obtm-se:

1 e1 + + s es + 1 g1 + + k gk = 0E ,
o que impli a que

1 = = s = 1 = = k = 0K ,
e1 , . . . , es , g1 , . . . , gk so linearmente independentes.
e1 , . . . , es , t1 , . . . , tp , g1 , . . . , gk so linearmente independentes e, portanto, estes ve tores formam uma base de F + G. Logo

j que

Con lui-se assim que

dim(F + G) = dim F + dim G dim(F G) .


{z
} |
{z
}
|
=s+p+k

=(s+p)+(s+k)s

F G = {0E }, onsidere-se BF = (t1 , . . . , tp ) e BG = (g1 , . . . , gk ) bases


F e G, respe tivamente. Por um ra io nio anlogo ao anterior,
prova-se que BF +G = (t1 , . . . , tp , g1 , . . . , gk ) uma base de F + G (demonstre!).
Se

ordenadas de

Logo, tambm neste aso,

dim(F + G) = dim F + dim G dim(F G) .


|
{z
} |
{z
}
=p+k

Observao 4.96.

=p+k0

No demonstrao do teorema anterior provou-se tambm

F G 6= {0E }, se BF G = (e1 , . . . , es ) uma base


de F G e BF = (e1 , . . . , es , t1 , . . . , tp ) e BG = (e1 , . . . , es , g1 , . . . , gk )
ordenadas de F e G, respe tivamente, ento

que, no aso de

ordenada
so bases

BF +G = (e1 , . . . , es , t1 , . . . , tp , g1 , . . . , gk )
uma base ordenada de
e

BG = (g1 , . . . , gk )

F + G.

No aso de

so bases ordenadas de

F G = {0E } e se BF = (t1 , . . . , tp )
F e G, respe tivamente, ento

BF +G = (t1 , . . . , tp , g1 , . . . , gk )
uma base ordenada de

F + G.

4.10 Teorema das dimenses


Exemplo 4.97.

99

Considere-se, no espao ve torial real

R3 ,

os subespaos ve -

toriais

F = {(x, y, z) R3 : x + y + 3z = 0}

G = h(1, 0, 1), (1, 1, 2)i.

No exemplo 4.81, viu-se que

F G =
=
=

 


5
1
y , 1,
:yR
2
2
5
1
h( , 1, )i
2
2
h(5, 2, 1)i

(5, 2, 1) 6= (0, 0, 0), (5, 2, 1) linearmente independente e, portanto,


BF G = ((5, 2, 1)) uma base ordenada de F G. Veja-se que dim F = 2. Ora
Como

= {(y 3z, y, z) : y, z R}
= {y(1, 1, 0) + z(3, 0, 1) : y, z R}
= h(1, 1, 0), (3, 0, 1)i

(1, 1, 0) e (3, 0, 1) ve tores linearmente independentes (verique!), enB = ((1, 1, 0), (3, 0, 1)) uma base ordenada de F e dim F = 2.
Assim, basta juntar um ve tor de F ao ve tor (5, 2, 1) que no seja
ombinao linear de (5, 2, 1) para se obter uma outra base de F . Por exemplo, (1, 2, 1) F e no ombinao linear de (5, 2, 1), pois (1, 2, 1) 6=
(5, 2, 1), para todo R. Como dim(F ) = 2 e (1, 2, 1), (5, 2, 1) F so

Sendo
to

linearmente independentes,

BF = ((1, 2, 1), (5, 2, 1))


Por outro lado,
ento

(5, 2, 1)

F.

base ordenada de

G = h(1, 0, 1), (1, 1, 2)i,

dim G 2. Como F G G,
G.
linear de (5, 2, 1), logo (1, 0, 1)

logo

um ve tor linearmente independente de

No entanto, (1, 0, 1) no ombinao


(5, 2, 1) so dois ve tores de G linearmente independentes e, onsequentemente, dim G 2. Con lui-se assim que BG = ((1, 0, 1), (5, 2, 1)) uma base
ordenada de G e que dim G = 2.
e

Finalmente,

BF = ((1, 2, 1), (5, 2, 1))


BG = ((1, 0, 1), (5, 2, 1))

base ordenada de

base ordenada de

e, portanto, pela observao 4.96:

BF +G = ((1, 2, 1), (5, 2, 1), (1, 0, 1))


Dado um espao ve torial sobre
da forma

F + G,

onde

K,

base ordenada de

F + G.

de entre os subespaos ve toriais de

so subespaos ve toriais de

importn ia os que veri am a seguinte ondio:

E,

tm parti ular

4.10 Teorema das dimenses


Para todo

uF +G

Denio 4.98.

100
u1 F
u = u1 + u2 .

existem um e um s
que

e um e um s

u2 G

tais

K e sejam F e G subespaos
F + G soma dire ta (ou que F e G esto em
soma dire ta), e representa-se por F G, se, para todo u F + G existem um
e um s u1 F e um e um s u2 G tais que
ve toriais de

E.

Seja

um espao ve torial sobre

Diz-se que

u = u1 + u2 .

Exemplos 4.99.

No espao ve torial real

R3 ,

onsidere-se os subespaos ve -

toriais:
1.

F = {(0, 0, z) : z R}

G = {(0, y, 0) : y R}.

F +G =
=
=

Ento

{(0, 0, z) + (0, y, 0) : y, z R}

{(0, y, z) : y, z R}
{(x, y, z) R3 : x = 0}.

u F + G tem-se que u = (0, b, c), para alguns


b, c F + G, pelo que u se es reve de modo ni o omo soma de um
elemento de F om um elemento de G, da forma u = (0, 0, c) + (0, b, 0),
onde (0, 0, c) F e (0, b, 0) G. Ento F est em soma dire ta om G.
Assim, qualquer que seja

2.

F = {(x, y, z) R3 : x = 0}

G = {(x, y, z) R3 : y = x}.

Ento

{(0, y, z) + (y , y , z ) : y, z, y , z R}
{(y , y + y , z + z ) : y, z, y , z R}

F +G =
=

R3

(verique!).

Tem-se que:

(1, 1, 1) = (0, 0, 1) + (1, 1, 0)


| {z } | {z }
F

e, por outro lado,

(1, 1, 1) = (0, 0, 1) + (1, 1, 2)


| {z } | {z }
F

Ento, existe
de

u F + G que
Ge

om um ve tor de

no se es reve de modo ni o omo um ve tor


portanto

no est em soma dire ta om

G.

O prximo resultado estabele e uma ara terizao para a soma dire ta de


dois subespaos ve toriais.

Proposio 4.100.
paos ve toriais de

(i)

A soma

Seja

E.

F +G

um espao ve torial sobre

e sejam

Ento, so equivalentes as seguintes armaes:

dire ta;

subes-

4.10 Teorema das dimenses


(ii)

101

O ve tor nulo es reve-se de modo ni o omo soma de um ve tor de


om um ve tor de

G;

(iii) F G = {0E }.
Demonstrao. Prova-se este resultado atravs da seguinte sequn ia de impli aes:

(i) (ii) (iii) (i)

(i) (ii) Resulta imediatamente da denio de soma dire ta.

(ii) (iii) Sabe-se que


Por outro lado, seja
ve torial de

{0E } F G.

(4.4)

u F G. Ento u F e u G. Como G um subespao


u G. Por denio de simtri o em E ,

tem-se que

u + (u) = 0E
|{z}
| {z }
F

e tem-se tambm

0E + 0E = 0E .
|{z}
|{z}
F

Por hiptese, o ve tor nulo es reve-se de modo ni o omo soma de um elemento


de

om um elemento de

G,

logo

u = 0E

e, onsequentemente,

F G {0E }
Por (4.4) e (4.5) on lui-se que

(4.5)

F G = {0E }.

(iii) (i) Seja u F + G e admita-se que

Ento

u1 + u2 ,

tais que

u =

u1

tais que u1

u1 u = u2 u
| {z }1 | {z }2
F

hiptese,

u2 ,

pelo que

e
e

u2 G
u2 G

u2 u2 F G.

Mas, por

F G = {0E }.

u1 = u1

u1 u1 F G

Logo

u1 u1 = 0E
ou seja,

u1 F

u =

u2 u2 = 0E

u2 = u2 .

Provou-se ento que ada ve tor

uF G
G.

se es reve de modo ni o omo

soma de um ve tor de

Exemplos 4.101.

1. No espao ve torial real

om um ve tor de

R4 ,

onsiderem-se os subes-

paos ve toriais

G =

{(x, y, z, t) R4 : x + y = 0 z + t = 0}

{(x, y, z, t) R4 : x = 0 t = 0}.

4.10 Teorema das dimenses

102

Atendendo a que

{(x, y, z, t) R4 : x + y = 0 z + t = 0 x = 0 t = 0}
{(0, 0, 0, 0)},

F G =
=
ento

G.

est em soma dire ta om

2. No espao ve torial real

P3 [x],

onsiderem-se os subespaos ve toriais

= {a0 + a1 x + a2 x2 + a3 x3 P3 [x] : a0 + a1 = 0 a3 = 0}
= {a0 + a1 x + a2 x2 + a3 x3 P3 [x] : a0 + a1 + a2 = 0}.

F
G

Atendendo a que

F G =
=
tem-se que

{a0 + a1 x + a2 x2 + a3 x3 P3 [x] : a0 + a1 = 0 a3 = 0

a0 + a1 + a2 = 0}
{a1 + a1 x : a1 R}

1x F G

e, portanto,

no soma dire ta.

Exer io 4.102.

No espao ve torial real

F G 6= {0E }.
R3 ,

Seja

o subespao gerado pelos ve tores

d.

(a)

F G;

(b)

F + G.

e seja

d = (2, 2, 2).

o subespao gerado pelos

Determine uma base para:

(sugesto: use a observao 4.96)

Teorema 4.103.
ve toriais de

F +G

onsidere os seguintes ve tores:

a = (1, 2, 1), b = (1, 2, 1), c = (1, 2, 3)


ve tores

Logo a soma

Seja

K e sejam F e G subespaos
S = F + G. Ento as armaes

um espao ve torial sobre

de dimenso nita. Seja ainda

seguintes so equivalentes:

(i) S = F G.

(ii) dim(F + G) = dim F + dim G.


(iii)

BF = (t1 , t2 , . . . , tp ) uma base ordenada de F e BG = (g1 , g2 , . . . , gk )


uma base ordenada de G, ento B = (t1 , t2 , . . . , tp , g1 , g2 , . . . , gk ) uma

Se

base ordenada de

F + G = S.

(i) (ii) Resulta de imediato do Teorema das dimenses. De


S = F G ento F G = {0E } e, onsequentemente, dim(F G) = 0.

Demonstrao.
fa to, omo
Logo

dim(F + G) = dim F + dim G dim(F G) dim(F + G) = dim F + dim G

4.11 Subespao omplementar

103

(ii) (i) Novamente, pelo Teorema das dimenses,


dim(F + G) = dim F + dim G dim(F G) = 0 F G = {0E }.
Daqui on lui-se que (i) (ii).
(ii) (iii) Por hiptese, dim(F + G) = dim F + dim G, logo, pelo teorema

dim(F G) = 0.

das dimenses tem-se

observao 4.96 obtm-se

Exer io 4.104.

(b) Averige se

F G = {0E }.

R3 , onsidere-se
G = {(0, y, z) : y, z R}.

No espao ve torial real

F = {(x, y, 0) : x, y R}
(a) Mostre que

Donde

(iii). A impli ao (iii) (ii) bvia.

so subespaos ve toriais de

Atendendo

os sub onjuntos

R3 .

R3 = F G.

4.11 Subespao omplementar


Denio 4.105.
ve torial de

E.

Seja

um espao ve torial sobre K e seja


F , de E , tal que

um subespao

A um subespao ve torial

E = F F ,

hama-se

subespao omplementar de

Teorema 4.106.

F.

E um espao ve torial sobre K


E tem pelo menos um subespao

Seja

subespao ve torial de
Demonstrao. Seja

Se

Se

Suponha-se que

um subespao ve torial de

F = {0E } ento um subespao


E (e ni o);

de dimenso

n.

Todo o

omplementar.

E.
F

omplementar de

o prprio espao

ve torial

F =E

ento um subespao omplementar de

{0E }

(e ni o);

F um subespao ve torial no trivial de E . Ento, seja


B = (f1 , . . . , fk ) uma base ordenada de F . Como F 6= E ento existem
ve tores em E que no perten em a F . Pode assim ompletar-se a base de
F por forma a obter uma base de E . Seja
BE = (f1 , . . . , fk , ek+1 , . . . , en )
essa base ordenada de

E.

Tome-se

S = hek+1 , . . . , en i.
E = F + S . Pelo Teorema 4.103, dim F + dim S = n e, portanto,
E = F S . Con lui-se assim que S um subespao omplementar de F .
Logo

4.11 Subespao omplementar

Exemplo 4.107.

No espao ve torial real

104

R4 ,

onsidere

F = {(x, y, z, w) R4 : x + y + z = y + 2z w = 0}.
Verique que

um subespao ve torial de

R4 .

Agora, vai-se determinar dois

F.
= ((1, 1, 0, 1), (1, 0, 1, 2))

subespaos omplementares de

uma base ordenada de F


Tem-se que BF
(prove!). Complete-se, de duas formas diferentes, esta base at obter uma base
4
de R .
1. a res entando os ve tores
Veja-se que os ve tores

(1, 0, 0, 0)

(0, 1, 0, 0).

(1, 0, 0, 0), (0, 1, 0, 0), (1, 1, 0, 1) e (1, 0, 1, 2) so


, , , R tais que

linearmente independentes. Sejam

(1, 0, 0, 0) + (0, 1, 0, 0) + (1, 1, 0, 1) + (1, 0, 1, 2) = (0, 0, 0, 0),


ou seja,

=0

+ =0
( , + , , + 2) = (0, 0, 0, 0)

=0

+ 2 = 0

E, portanto,

= = = = 0.

Logo, a ni a forma de es rever o

ve tor nulo omo ombinao dos ve tores dados a ombinao linear


nula trivial.
Logo os ve tores

(1, 0, 0, 0), (0, 1, 0, 0), (1, 1, 0, 1) e (1, 0, 1, 2) so linearR4 tem dimenso 4, estes ve tores formam

mente independentes. Como


uma base de R4 .

S = h(1, 0, 0, 0), (0, 1, 0, 0)i. Tem-se


subespao omplementar de F .

Seja
um

2. a res entando os ve tores


Veja-se que os ve tores

(1, 0, 0, 0)

que

R4 = F S .

Portanto

(0, 0, 1, 0).

(1, 0, 0, 0), (0, 0, 1, 0), (1, 1, 0, 1) e (1, 0, 1, 2) so


, , , R tais que

linearmente independentes. Sejam

(1, 0, 0, 0) + (0, 0, 1, 0) + (1, 1, 0, 1) + (1, 0, 1, 2) = (0, 0, 0, 0),


ou seja,

=0

=0
( , , + , + 2) = (0, 0, 0, 0).

+ =0

+ 2 = 0

4.11 Subespao omplementar


E, portanto,

(1, 1, 0, 1)

105

= = = = 0. Logo os
(1, 0, 1, 2) so linearmente

ve tores

(1, 0, 0, 0), (0, 0, 1, 0),

independentes. Mais uma vez


4
pode on luir-se que estes ve tores tambm formam uma base de R .
Seja

S1 = h(1, 0, 0, 0), (0, 0, 1, 0)i.

Tem-se que

outro subespao omplementar de

F.

R4 = F S 1 .

Portanto

S1

5. Apli aes lineares

5.1 Denio e propriedades

107

5.1 Denio e propriedades


Denio 5.1.

ao linear

em

Sejam

(ou

dois espaos ve toriais sobre

transformao linear

a toda a apli ao

: E E

ou ainda

K.

Chama-se

homomorsmo)

aplide

que satisfaz as seguintes ondies:

(i) u, v E , (u + v) = (u) + (v);

(ii) K, u E , (u) = (u).

Simpli ao de notao:
vez de

Se

u = (x1 , . . . , xn ),

es reve-se

(x1 , . . . , xn )

em

((x1 , . . . , xn )).

Exemplos 5.2.

1. A apli ao

R3
R2
(x, y, z) 7 (x, y, z) = (x + y, z)

uma apli ao linear de

(i)

Sejam

R3

em

(x, y, z), (a, b, c) R3

R2 .

De fa to,

arbitrrios. Ento

((x, y, z) + (a, b, c)) = (x + a, y + b, z + c)

por denio
de adio em

= ((x + a) + (y + b), z + c)
= ((x + y) + (a + b), z + c)

por denio de

por denio
de adio em

= (x, y, z) + (a, b, c)
Provou-se assim que, para quaisquer

pelas propriedades
da adio em

= (x + y, z) + (a + b, c)

R3

R2

por denio de

(x, y, z), (a, b, c) R3 ,

se tem

((x, y, z) + (a, b, c)) = (x, y, z) + (a, b, c).

(ii)

Sejam agora

(x, y, z) R3

((x, y, z)) = (x, y, z)

arbitrrios. Ento

por denio de multipli ao por

R3
de

um es alar em

= (x + y, z)

por denio

= ((x + y), z)

pela distributividade da multipli ao

= (x + y, z)

por denio de multipli ao por

em relao adio em

R2
de .

um es alar em

= (x, y, z)

por denio

Provou-se assim que, para quaisquer

(x, y, z) R3 ,

((x, y, z)) = (x, y, z).

se tem

5.1 Denio e propriedades


2. Seja

x0 R.

A apli ao

F (R) R
f
7 (f ) = f (x0 )

:
uma apli ao linear de

(i)

f, g F (R)

Sejam

108

F (R)

em

por denio de

por denio de

(f + g) = (f ) + (g),

Sejam

f F (R)

(f ) = (f )(x0 )
= (f (x0 ))
= (f )
Ou seja,

por denio de adio em

= (f ) + (g)

(ii)

De fa to,

arbitrrios. Ento

(f + g) = (f + g)(x0 )
= f (x0 ) + g(x0 )

Logo

R.

para quaisquer

arbitrrios. Ento

por denio de

f, g F (R).

por denio de multipli ao por um es alar em


por denio de

(f ) = (f ),

para quaisquer

f F (R)

R.

3. A apli ao

: M31
(R)
a
b
c

R2
a
b = (a2 , b + c 2)
c

no uma apli ao linear. De fa to,

mas

Logo,

F (R)


1
0 = (12 , 0 + 2 2) = (1, 0)
2



1
2
2 0 = 0 = (22 , 0 + 4 2) = (4, 2),
2
4


1
2 0 = 2(1, 0) = (2, 0).
2

1
(u) 6= (u), quando = 2 e u = 0.
2

F (R)

5.1 Denio e propriedades


Exer io 5.3.

109

Verique que a apli ao

R2
(x, y)

uma apli ao linear de

Proposio 5.4.

R2

R3
7 (x, y) = (2x, x y, 3y)
em

R3 .

E e E espaos
E em E . Ento:

Sejam

apli ao linear de

K.

ve toriais sobre

Seja ainda

uma

(a) (0E ) = 0E ;
(b) (u) = (u),

( ) (u v) = (u) (v),

Demonstrao. Prove-se
para a adio em

E,

u E;

para todo

para todo

u, v E .

(a). Como o ve tor nulo de E , 0E , o elemento neutro

ento

0E = 0E + 0E .

Donde

(0E ) = (0E + 0E )
(0E ) = (0E ) + (0E ).
Note-se que a ltima igualdade resulta do fa to de ser uma apli ao linear.

Como E um espao ve torial e (0E ) E , ento existe o seu simtri o em

E , (0E ). Mas ento, adi ionando a ambos os membros da igualdade anterior

esse simtri o, obtm-se

(0E ) + ((0E )) = (0E ) + (0E ) + ((0E ))


0E = (0E ) + 0E

pelos axiomas de espao

0E = (0E )

por denio de

Prove-se

(b).

ve torial

u E.
(u) em E ,

Seja

simtri o do ve tor

Pretende-se provar que o ve tor


ou seja,

(u) + (u) = (u + u)
= (0E )

pois

(u) = (u),

Prove-se

apli ao linear

pela alnea
para todo

(u)

Ora

por denio de simtri o em

= 0E
Portanto,

(u) + (u) = 0E .

(a).

u E.

( ). Sejam u, v E . Ento

(u v) = (u + (v))

0E .

por notao

= (u) + (v)
= (u) + ((v))

pois

= (u) (v)

por notao.

apli ao linear

pela alnea

(b)

5.1 Denio e propriedades

110

O prximo resultado estabele e uma nova ara terizao para apli aes
lineares.

Proposio 5.5.
ao de

E e E espaos ve toriais sobre K e seja


E . Ento uma apli ao linear se e s se
Sejam

uma apli-

em

(u + v) = (u) + (v),

, K, u, v E.

A demonstrao  a omo exer io.

espaos ve toriais sobre K. Representa-se por L(E, E ) o on

junto das apli aes lineares de E para E . Observe-se que, omo L(E, E ) um
Sejam

onjunto de apli aes, pode onsiderar-se as seguintes operaes:

Adio: Dadas , L(E, E ), +

a apli ao de

( + )(u) = (u) + (u), para

Multipli ao por um es alar:


apli ao de

em

tal que

Dados

K.

todo o

todo o

em

tal que

u E.

L(E, E )

()(u) = ((u)) , para

Teorema 5.6.

K,

u E.

E e E espaos ve toriais sobre K e sejam , L(E, E )


+ e so apli aes lineares. Mais, L(E, E ), munido om

Sejam

Ento

a adio e a multipli ao por um es alar denidas anteriormente, um espao


ve torial sobre

K.

Demonstrao. Considere-se

u, v E .

Ento

, L(E, E )

arbitrrias e sejam

( + )(u + v) = (u + v) + (u + v)

por denio de
adio em

= (u) + (v) + (u) + (v)


= ((u) + (u)) + ((v) + (v))

, K

L(E, E ).

pela Proposio 5.5


por

ser um espao

ve torial

= (( + )(u)) + (( + )(v))

por denio de
adio em

+
L(E, E ).

Portanto,
em

L(E, E ).

uma apli ao linear. Logo a adio uma operao interna

Analogamente, onsidere-se

L(E, E ), K

e sejam

, , K

5.1 Denio e propriedades


u, v E .

111

Ento

()(u + v) = ((u + v))

por denio de multipli ao


por um es alar em

L(E, E ).

= ((u) + (v))

pela Proposio 5.5

= ()(u) + ()(v)
= ()(u) + ()(v)

por

= (((u))) + (((v)))
= () (u) + () (v)

ser um espao ve torial

pela omutatividade em
por

ser um espao ve torial

por denio de multipli ao


por um es alar em

L(E, E ).

uma apli ao linear, e portanto L(E, E ).

Fi a omo exer io provar que L(E, E ) um espao ve torial sobre

Logo

om

as operaes anteriormente denidas.

Observao 5.7.

A apli ao

0L(E,E ) : E
u

E
7 0L(E,E ) (u) = 0E

uma apli ao linear de E para E e o ve tor nulo do espao ve torial


L(E, E ). Prove que, de fa to, 0L(E,E ) + = , para todo L(E, E ).

Mais, para ada L(E, E ), a apli ao

E
7 ()(u) = ((u))

: E
u

E em E
+ () = 0L(E,E ) .

uma apli ao linear de


que, de fa to,

Exer io 5.8.

L(E , E ).

Sejam

E, E

uma apli ao linear de

em

em

espaos ve toriais sobre

Prove que a apli ao, qual se hama

: E
u

e o simtri o de

L(E, E ).

Prove

K, L(E, E )

apli ao omposta,

E
7 ( )(u) = ((u))
E .

5.1.1 Classi ao de apli aes lineares


Denio 5.9.
linear de

Sejam E e E espaos ve toriais sobre


em E . Diz-se que um

K e seja uma apli ao

monomorsmo se inje tiva, isto , (u) = (v) u = v, u, v E ;


epimorsmo se sobreje tiva, isto , se o ontradomnio de E ;

5.1 Denio e propriedades

112

isomorsmo se bije tiva, isto , se inje tiva e sobreje tiva;


endomorsmo se E = E ;
automorsmo se um endomorsmo e um isomorsmo.

Exemplos 5.10.

1. A apli ao linear

: R3 R

(x, y, z) = x + y + z,

denida por

(x, y, z) R3

um epimorsmo.
2. A apli ao linear

: P3 [x] R4

denida por

(ax3 + bx2 + cx + d) = (a, b, c, d),

ax3 + bx2 + cx + d P3 [x]

um isomorsmo.
3. A apli ao linear

: P2 [x] P2 [x]

denida por

(ax2 + bx + c) = cx2 + bx + a,

ax2 + bx + c P2 [x]

um automorsmo.

5.1.2 Propriedades das apli aes lineares


O teorema seguinte exprime o omportamento das apli aes lineares em relao
dependn ia e/ou independn ia linear de ve tores.

Teorema 5.11.
linear de

(i)

Sejam E e E espaos ve toriais sobre

para E . Ento:

K e seja uma apli ao

v1 , . . . , vk E so linearmente dependentes em E
(v1 ), . . . , (vk ) so linearmente dependentes em E .

Se os ve tores
ve tores

(ii)

monomorsmo se e s se

v1 , . . . , vk impli ar
(v1 ), . . . , (vk ).

a independn ia linear dos ve tores


pendn ia linear dos ve tores
Demonstrao. Prove-se

(i).

Sejam

v1 , . . . , vk E

todos nulos tais que

1 v1 + + k vk = 0E .

a inde-

e suponha-se que estes ve -

tores so linearmente dependentes. Ento existem es alares

Como

ento os

uma apli ao linear, tem-se

(0E ) = 0E
(1 v1 + + k vk ) = 0E
1 (v1 ) + + k (vk ) = 0E .

1 , . . . , k K

no

5.1 Denio e propriedades

113
(v1 ), . . . , (vk ) em
(v1 ), . . . , (vk ) so linearmente

Ou seja, obtm-se uma ombinao linear nula dos ve tores


que os es alares no so todos nulos. Portanto,

dependentes em E .

(ii). () Suponha-se que monomorsmo, ou seja, inje tiva e


v1 , . . . , vk E ve tores linearmente independentes. Sejam 1 , . . . , k K

Prove-se
sejam

tais que

1 (v1 ) + + k (vk ) = 0E .

1 = = k = 0K , para se poder
(v1 ), . . . , (vk ) so linearmente independentes. Como
linear, tem-se (0E ) = 0E e

Pretende-se provar que a sua ni a soluo


on luir que os ve tores

uma apli ao

(1 v1 + + k vk ) = 1 (v1 ) + + k (vk ) = 0E = (0E ).


Como

inje tiva, se dois obje tos tm a mesma imagem, ento eles so iguais,

logo

1 v1 + + k vk = 0E .
Mas os ve tores

v1 , . . . , vk

so linearmente independentes, donde

1 = = k = 0K .
Portanto, os ve tores

(v1 ), . . . , (vk )

so linearmente independentes.

() Suponha-se agora que sempre que

pendentes ento
que

(v1 ), . . . , (vk )

v1 , . . . , vk

forem linearmente inde-

so linearmente independentes. Vai-se provar

inje tiva.

no um monomorsmo, isto ,
u, v E tais que u 6= v e (u) = (v). Seja w = u v .
w 6= 0E , donde w linearmente independente. Ento, por

Suponha-se, om vista a um absurdo,


suponha-se que existem
Como

u 6= v tem-se
(w) linearmente

hiptese,

independente. Mas, por outro lado,

(w) = (u v) = (u) (v) = 0E


e

0E

linearmente dependente. Logo

absurdo. Portanto,

(w)

linearmente dependente, o que

monomorsmo.

Resumindo:

Uma apli ao linear transforma ve tores linearmente dependentes em ve -

Para que uma apli ao linear transforme sempre ve tores linearmente

tores linearmente dependentes.

independentes em ve tores linearmente independentes ne essrio e su iente que esta apli ao seja inje tiva.

Exemplos 5.12.

1. Seja

uma apli ao linear de

(x, y, z) = (x + y, z),

para todo

R3

em

R2

(x, y, z) R3 .

tal que

5.1 Denio e propriedades


f il veri ar que

114

no um monomorsmo. De fa to,

(3, 2, 1) = (4, 1, 1) = (5, 1).


u1 = (1, 1, 1), v1 = (1, 3, 2) e
Note-se que so linearmente dependentes (prove!) e

Considere os ve tores

(u1 ) = (2, 1),

(v1 ) = (4, 2)

w1 = (1, 5, 3).

(w1 ) = (6, 3)

tambm so linearmente dependentes.

u2 = (1, 1, 0), v2 = (0, 3, 2) e


Prove que so linearmente independentes. No entanto,

Considere os ve tores

(u2 ) = (0, 0),

(v2 ) = (3, 2)

w2 = (1, 0, 3).

(w2 ) = (1, 3)

so linearmente dependentes (justique!).


2. Seja

uma apli ao linear de

para todo

ax + b P1 [x].

Prove

P1 [x]
que

em

R2

tal que

(ax + b) = (a, b),

um monomorsmo.

p1 (x) = x 1, q1 (x) = 3 e r1 (x) = 3x + 3.


Note-se que so linearmente dependentes pois dim(P1 [x]) = 2. Fa ilmente

Considere os polinmios
se veri a que

(p1 (x)) = (1, 1),

(q1 (x)) = (0, 3)

(r1 (x)) = (3, 3)

tambm so linearmente dependentes.

p2 (x) = 4x 3 e q2 (x) = x + 5. Prove que


so linearmente independentes e verique que tambm os ve tores

Considere agora os polinmios

(p2 (x)) = (4, 3)

(q2 (x)) = (1, 5)

so linearmente independentes.
O prximo resultado mostra que uma apli ao linear ujo domnio um espao ve torial de dimenso nita  a perfeitamente denida quando se onhe em
as imagens dos ve tores de uma qualquer base desse mesmo espao ve torial.

Proposio 5.13.

E e E espaos
B = (e1 , . . . , en ) uma

Sejam

dimenso nita. Seja

Ento existem uma e uma s apli ao linear

i {1, . . . , n}.
v = 1 e1 + + n en ,

K tais que E tem


E e sejam u1 , . . . , un E .
E para E tal que (ei ) = ui ,

ve toriais sobre
base de

de

para qualquer
Mais, se

Demonstrao. Seja

v E.

Como

ento

uma base de

ombinao linear dos ve tores da base


que

(v) = 1 u1 + + n un .

B,

v = 1 e1 + + n en .

v es reve-se omo
1 , . . . , n K tais

ento

ou seja, existem

5.1 Denio e propriedades


Dena-se a apli ao

i {1, . . . , n},

ento

115

: E E

(v) = 1 u1 + + n un .

tal que

Seja

ei = 0K e1 + + 0K ei1 + 1K ei + 0K ei+1 + + 0K en .

Por denio de

vem que

(ei ) = 0K u1 + + 0K ui1 + 1K ui + 0K ui+1 + + 0K un = ui .


A demonstrao de que

uma apli ao linear  a omo exer io.

Prove-se a uni idade. Suponha-se que existe uma outra apli ao linear
tal que
existem

(ei ) = ui , para todo i {1, . . . , n}.


1 , . . . , n K tais que

Seja

v E

arbitrrio. Ento

v = 1 e1 + + n en .
Donde

(v) = (1 e1 + + n en )
= 1 (e1 ) + + n (en )
= 1 u1 + + n un
= 1 (e1 ) + + n (en )

(v) = (v),

Exemplos 5.14.

para todo

v E,

pois

(ei ) = ui , i {1, . . . , n}
ui = (ei ), i {1, . . . , n}

pois

= (1 e1 + + n en )
= (v)

Como

pois

pois

apli ao linear

apli ao linear

pode on luir-se que

1. Considere-se a apli ao linear

(1, 1) = (1, 0, 1)

= .

: R2 R3

tal que

(1, 0) = (0, 2, 1).

Pretende-se determinar a expresso geral de

uma base ordenada de R2


(prove!). Em seguida, es reve-se um ve tor arbitrrio de R2 omo omPrimeiro, note-se que

B = ((1, 1), (1, 0))

binao linear dos ve tores

(1, 1)

(1, 0).

Se

(x, y) = (1, 1) + (1, 0)


ento

Ou seja,

x=+
=
y=

(x, y) = y(1, 1) + (x y)(1, 0).

= xy
=y

Ento

(x, y) = (y(1, 1) + (x y)(1, 0))


= y(1, 1) + (x y)(1, 0)

= y(1, 0, 1) + (x y)(0, 2, 1)
= (y, 2x 2y, x 2y).

Portanto,

(x, y) = (y, 2x 2y, x 2y),

para todo

(x, y) R2 .

5.2 Imagem e imagem re pro a

116

: R2 R2

(1, 0) = (0, 1) e (0, 1) = (1, 0)


y = x. Com efeito, atendendo a que
BR2 = ((1, 0), (0, 1)) a base anni a de R2 , para qualquer (x, y) R2 ,
tem-se que (x, y) = x(1, 0) + y(0, 1). Logo:

2. A apli ao linear

tal que

a simetria do plano em relao re ta

(x, y) = (x(1, 0) + y(0, 1))


= x(1, 0) + y(0, 1)
= x(0, 1) + y(1, 0) = (y, x).
(x, y) = (y, x),

Portanto,

Exer io 5.15.

(x, y) R2 .

para todo

: P2 [x] M22 (R) tal que





2 0
0
e (1 + 2x) =
.
0 1
1

Considere a apli ao linear

(1 + x) =

1 0
0 1

Verique que

ni a e determine

, (x2 x) =

1
0

(ax2 +bx+c), para todo ax2 +bx+c P2 [x].

5.2 Imagem e imagem re pro a


Denio 5.16.

Sejam

subespaos ve toriais de
E em E .

linear de

Chama-se

E e E espaos ve toriais sobre K e


E e E , respe tivamente. Seja ainda

imagem de

por

ao sub onjunto de

sejam

uma apli ao

denido por

(F ) = {(u) : u F }
ou seja, o onjunto de todas as imagens dos ve tores perten entes a F .

Ao onjunto dos ve tores uja imagem perten e a F , isto , ao sub onjunto


de

denido por:

1 (F ) = {v E : (v) F }

hama-se

imagem re pro a de

Observao 5.17.

por

As denies de imagem e imagem re pro a podem ser


E e E , respe tivamente.

apli adas a quaisquer sub onjuntos de

Teorema 5.18.
linear de

(a)

se
de

Sejam E e
em E . Ento:

F
E.

espaos ve toriais sobre

um subespao ve torial de

ento

(F )

K e seja uma apli ao


um subespao ve torial

5.2 Imagem e imagem re pro a


(b)

se

117
E

um subespao ve torial de

ento

1 (F )

um subespao ve -

E.

torial de

Demonstrao. Prove-se

(a).

Suponha-se que

subespao ve torial de

E.

Ento

(i) 0E (F ),

(ii)

Sejam

pois

, K

0E F .

0E = (0E )

u, v (F ).

Ento existem

u = (u1 )

u1 , v1 F

tais que

v = (v1 ).

Logo

u + v = (u1 ) + (v1 ) = (u1 + v1 ),


pois

apli ao linear e, portanto,

u + v = (w),
Donde

om

w = u1 + v1 F .

u + v (F ).

A demonstrao de

Exemplo 5.19.

(b) anloga e  a omo exer io.

Considere a apli ao linear

(x, y) = (x, x + y, x y),

: R2 R3

para todo

e onsidere os seguintes subespaos ve toriais de

F = {(0, y) : y R}

R2

denida por

(x, y) R2

R3 ,

respe tivamente:

F = {(a, 2a, 0) : a R}.

Ento,

(F ) = {(0, y) : y R}

= {(0, 0 + y, 0 y) : y R}
= {(0, y, y) : y R} = h(0, 1, 1)i

1 (F ) = {(x, y) R2 : (x, y) F }

= {(x, y) R2 : (x, x + y, x y) = (a, 2a, 0),


2

para algum

= {(x, y) R : x = a x + y = 2a x y = 0,
2

= {(x, y) R : x = a y = a x = y,

= {(a, a) : a R} = h(1, 1)i.

Exer io 5.20.

Sejam



a
c

S=



b
d


ve toriais de

a R}

para algum

para algum

a R}

a R}

: M22 (R) P2 [x] tal que




a b
M22 (R).
para todo
c d

Considere a apli ao linear

= (a + b)x2 + 2ax d,



x 0
: x R e G = {ax2 + bx + c P2 [x] : c = 0} subespaos
2x x
M22 (R) e P2 [x], respe tivamente. Determine (S) e 1 (G).

5.3 N leo e imagem

118

5.3 N leo e imagem


Denio 5.21.
ao linear de
Chama-se
junto de

Sejam E e

em E .

n leo de

espaos ve toriais sobre

e representa-se por

Nuc

(ou

e seja

Ker ),

uma apli-

ao sub on-

denido por:

Nuc = {u E : (u) = 0E } = 1 ({0E }) .

Chama-se

imagem de

e representa-se por

Im ,

ao sub onjunto de

denido por:

Proposio 5.22.
apli ao linear de

(a) Nuc
(b) Im

Im = {(u) : u E} = (E).
E

Sejam

em

um subespao ve torial de

um subespao ve torial de

Demonstrao. Prove-se

(i)

Como

(ii)

Sejam
omo

espaos ve toriais sobre

e seja

uma

. Ento:

E;

E.

(a).

(0E ) = 0E ,

ento

0E Nuc ;

u, v Nuc e , K.
apli ao linear,

Ento

(u) = 0E

(v) = 0E .

Logo,

(u + v) = (u) + (v) = 0E + 0E = 0E .
Donde

u + v Nuc .

Tambm se podia provar

(a)

um subespao ve torial de
A demonstrao de

Exemplo 5.23.
(x, y, z) R

atendendo a que

1 ({0E }) = Nuc

, pelo Teorema 5.18.

{0E }

(b)  a omo exer io.

Considere a apli ao linear

(x, y, z) = (x + 2z, y z).

: R3 R2

Ento o n leo de

tal que, para todo

Nuc = {(x, y, z) R3 : (x, y, z) = (0, 0)}

= {(x, y, z) R3 : (x + 2z, y z) = (0, 0)}.

Ora

(x + 2z, y z) = (0, 0)
Logo

Nuc = {(2z, z, z) : z R}.

x + 2z = 0

yz =0

A imagem de

x = 2z
y=z

Im = {(x, y, z) : (x, y, z) R }

= {(x + 2z, y z) : x, y, z R}
= {(x, 0) + (0, y) + (2z, z) : x, y, z R}

= {x(1, 0) + y(0, 1) + z(2, 1) : x, y, z R}

= h(1, 0), (0, 1), (2, 1)i = R2

(prove!)

5.3 N leo e imagem


Exer io 5.24.

119

Considere a apli ao linear

(ax2 + bx + c) = (c + b)x3 + ax2 ,


Determine

Nuc

: P2 [x] P3 [x]

para todo

tal que

ax2 + bx + c P2 [x].

Im .

Proposio 5.25.

Sejam

apli ao linear de
(v) = v ento

em

E e E
E . Seja

espaos ve toriais sobre K e seja uma

ainda v E . Se existe v E tal que

1 ({v }) = v + Nuc .
Demonstrao. De fa to,

1 ({v }) = {u E : (u) = v }
= {u E : (u) = (v)}

= {u E : (u v) = 0E }
= {u E : u v Nuc }

= {u E : u v = w, w Nuc }
= {v + w : w Nuc }
= v + Nuc .

Exemplos 5.26.

1. Seja

: R3 R2

(x, y, z) = (x + y, z),
Seja

(0, 1, 2) R3

para todo

(x, y, z) R3 .

e determine-se o onjunto de ve tores de

R3

om a

isto , uja imagem por o ve tor


(ou seja, determine-se 1 ({(1, 2)})). Tem-se

mesma imagem por

(0, 1, 2) = (1, 2)

uma apli ao linear denida por

que

(0, 1, 2),

Nuc = {(x, y, z) R3 : (x, y, z) = (0, 0), }


ou seja,

Logo

x+y =0

z=0

Nuc = {(x, x, 0) : x R}.

y = x
z=0

Con lui-se ento que

1 ({(1, 2)}) = (0, 1, 2) + {(x, x, 0) : x R}


= {(x, 1 x, 2) : x R}.

2. Seja
todo
uja

: P2 [x] P3 [x] tal que (ax2 + bx + c) = b + (c + a)x + ax3 , para


ax2 + bx + c P2 [x]. Determine-se o onjunto de polinmios
de P2 [x]

3
1
imagem por 1 + 2x , ou seja,
{1 + 2x3 } .

Atendendo a que
que!), tem-se

(2 x + 2x2 ) = 1 + 2x3

Nuc = {0P2 [x] }

(veri-






1 + 2x3 = 2 x + 2x2 + {0P2 [x] } = 2 x + 2x2 .

5.3 N leo e imagem

120

Exer io 5.27.

Seja : P2 [x] P1 [x] uma apli ao linear denida por


(ax2 +bx+c) = ax+b, para todo ax2 +bx+c P2 [x]. Determine 1 ({5x 1}).

Proposio 5.28.

E e E espaos ve toriais sobre K tais que E tem


dimenso nita. Seja B = (e1 , e2 , . . . , en ) uma base ordenada de E e seja ainda
uma apli ao linear de E em E . Ento
Sejam

Im = h(e1 ), (e2 ), . . . , (en )i.


Demonstrao. Para mostrar a igualdade entre os dois onjuntos tem de se
mostrar as duas in luses:

(i) h(e1 ), (e2 ), . . . , (en )i Im

(ii) Im h(e1 ), (e2 ), . . . , (en )i


Prove-se

(i). Esta in luso bvia. De fa to, seja u h(e1 ), (e2 ), . . . , (en )i.

Ento existem es alares

1 , 2 , . . . , n K

tais que

u = 1 (e1 ) + 2 (e2 ) + + n (en ).


Como

(ei ) Im , para todo i {1, . . . , n},


E , ento u Im .

(justique!) e

Im

subespao

ve torial de

Prove-se
lado, omo

(ii). Seja v Im . Ento existe u E

base de

E,

existem es alares

tal que v = (u). Por outro


1 , 2 , . . . , n K tais que

u = 1 e1 + 2 e2 + + n en
e, portanto,

v = (u) = (1 e1 + 2 e2 + + n en )

= 1 (e1 ) + 2 (e2 ) + + n (en )

o que equivale a dizer que

v h(e1 ), (e2 ), . . . , (en )i.

Observao 5.29.

Im

Como

subespao ve torial de

E,

ento

dim(Im ) dim(E )
e, analogamente, omo

Nuc

subespao ve torial de

dim(Nuc ) dim E.

E,

ento

5.3 N leo e imagem


Exemplo 5.30.

Seja

121

: P1 [x] R3

uma apli ao linear denida por

(ax + b) = (b + a, a, 2b),
Sabendo que

B = (1 + x, x)

uma base de

ax + b P1 [x].

P1 [x],

determine-se

Im .

Im = h(1 + x), (x)i

= h(2, 1, 2), (1, 1, 0)i

= {(x, y, z) R3 : z = 2x 2y}.

Denio 5.31.

(verique!)

K tais que E tem


E em E . dimenso de
Nuc hama-se nulidade de , e representa-se por n , e dimenso de Im
hama-se ara tersti a de , e representa-se por c .
Sejam

dimenso nita. Seja ainda

espaos ve toriais sobre

uma apli ao linear de

Exemplo 5.32.
para todo
n leo de

Seja : R3 R2 denida por (x, y, z) = (x + y + z, 2x y),


(x, y, z) R3 . Determine-se a nulidade e a ara tersti a de . O
:

Nuc = {(x, y, z) R3 : (x, y, z) = (0, 0)}

= {(x, y, z) R3 : (x + y + z, 2x y) = (0, 0)}.

Ora

(x + y + z, 2x y) = (0, 0)

x+y+z =0

2x y = 0

z = 3x
y = 2x

Logo

Nuc = {(x, 2x, 3x) : x R} = {x(1, 2, 3) : x R} = h(1, 2, 3)i.


Como

(1, 2, 3) 6= (0, 0, 0) on lui-se que o ni o gerador de Nuc linearmente


B = ((1, 2, 3)) uma base de Nuc e, portanto, n = 1.
imagem de

independente. Assim,
A

Im = {(x, y, z) : (x, y, z) R3 }

= {(x + y + z, 2x y) : x, y, z R}
= {x(1, 2) + y(1, 1) + z(1, 0) : x, y, z R}

= h(1, 2), (1, 1), (1, 0)i = R2 .


Logo

(justif ique!)

c = 2 .

Exer io 5.33.

: R3 R4 denida por
todo (x, y, z) R3 . Determine

Considere a apli ao linear

(x, y, z) = (x z, 0, y + 2z, x y + z),


a nulidade e a ara tersti a de .

para

5.3 N leo e imagem

122

O prximo resultado apresenta uma ondio ne essria e su iente para a


inje tividade de uma apli ao linear, usando o n leo dessa apli ao linear.

Proposio 5.34.

Sejam

apli ao linear de

em

E e E espaos ve toriais sobre K. Seja ainda uma


E . A apli ao um monomorsmo se e s se

Nuc = {0E }.
Demonstrao. () Suponha-se que

u Nuc (u) = 0E
(u) = (0E )

se

Nuc = {0E }.

() Suponha-se agora que

(u) = (v)

ento

u = v.

pois

inje tiva.

Nuc = {0E }.

(u v) = 0
u v Nuc
u v = 0E
u=v

em

Prove-se que, para todo

por

pois

apli ao linear

por hiptese,
por

u, v E ,

ser um espao ve torial

por denio de

Nuc

Nuc = {0E }

ser um espao ve torial.

inje tiva.

Teorema 5.35. (Teorema da dimenso)


sobre

Ento

Nuc

Ora

u Nuc .

pelas propriedades de apli ao linear

(u) = (v) (u) (v) = 0E

Portanto

inje tiva e seja

por denio de

u = 0E

Logo

K tais que E
E . Ento:

Sejam

tem dimenso nita. Seja ainda

E e E espaos ve toriais
uma apli ao linear de

dim E = dim(Nuc ) + dim(Im )


ou, abreviadamente,

dim E = n + c .
E = {0E } ento Nuc = {0E } e Im = {0E }. Logo
dim E = dim(Nuc ) + dim(Im ). Suponha-se que E 6= {0E } e seja B =
(e1 , e2 , . . . , en ) uma base de E . Se Nuc = {0E } ento, pela proposio 5.34,
um monomorsmo. Pela alnea (ii) do teorema 5.11 e omo os ve tores e1 , . . . , en
so linearmente independentes, ento (e1 ), . . . , (en ) so linearmente indepenDemonstrao. Se

dentes. Pela proposio 5.28 tem-se

Im = h(e1 ), (e2 ), . . . , (en )i.


B = ((e1 ), (e2 ), . . . , (en )) uma base ordenada
dim(Im ) = n. Como dim(Nuc ) = 0 e dim E = n, tem-se
Logo

de

dim E = n = 0 + n = dim(Nuc ) + dim(Im ).

Im .

Donde,

5.3 N leo e imagem

123

Nuc 6= {0E }. Seja BNuc = (u1 , . . . , up ) uma base de


p dim E . Pelo orolrio 4.70 possvel juntar ve tores de E
Nuc por forma a obter uma base de E . Seja

Suponha-se agora que

Nuc ,

onde

base de

B = (u1 , . . . , up , ep+1 , . . . , ep+k )


uma base de
uma base de

E , onde k = n p. Agora, prova-se


Im . Pela proposio 5.28 tem-se

que

((ep+1 ), . . . , (ep+k ))

Im = h(u1 ), . . . , (up ), (ep+1 ), . . . , (ep+k )i


= h0E , . . . , 0E , (ep+1 ), . . . , (ep+k )i
= h(ep+1 ), . . . , (ep+k )i

Resta provar que os ve tores


dentes. Sejam

p+1 , . . . , p+k

((ep+1 ), . . . , (ep+k ))
K tais que

so linearmente indepen-

p+1 (ep+1 ) + + p+k (ep+k ) = 0E .


Ento

(p+1 ep+1 + + p+k ep+k ) = 0E ,

ou seja,
que

p+1 ep+1 + + p+k ep+k Nuc .

Logo, existem

1 , . . . , p K

tais

p+1 ep+1 + + p+k ep+k = 1 u1 + + p up ,


donde,

p+1 ep+1 + + p+k ep+k 1 u1 p up = 0E


uma ombinao linear nula dos ve tores
estes ve tores formam uma base de

E,

u1 , . . . , up , ep+1 , . . . , ep+k . Mas omo

so linearmente independentes, logo a

ni a ombinao linear nula destes ve tores a trivial. Consequentemente,


p+1 = = k = K e os ve tores (ep+1 ), . . . , (ep+k so linearmente in

dependentes. Provou-se assim que ((ep+1 ), . . . , (ep+k )) uma base de Im e

dim(Im ) = k .

Portanto,

Exemplo 5.36.

dim E = p + k = n + c .

Nuc =
=
=
=
Logo

: R3 R3
(x, y, z) R3 . Ento

Considere a apli ao linear

(x, y, z) = (x + 2y, y z, y),

para todo

denida por

{(x, y, z) R3 : (x, y, z) = (0, 0, 0)}

{(x, y, z) R3 : (x + 2y, y z, y) = (0, 0, 0)}


{(x, y, z) R3 : x + 2y = 0 y z = 0 y = 0}

{(0, 0, 0)}.

um monomorsmo. Alm disso, pelo Teorema das dimenses,


Im um subespao ve torial de R3 , Im = R3 , ou seja,

Logo, omo

epimorsmo. Donde

um isomorsmo.

c = 3 .
um

5.3 N leo e imagem


Exer io 5.37.

Seja

124
uma apli ao linear de

(1, 0, 0) = (1, 0),


,

Classique

em

R2

denida por

(0, 0, 1) = (0, 1).

quanto inje tividade e sobreje tividade.

Observao 5.38.

E
ainda

Sejam

dimenso nita. Seja

(i)

(0, 1, 0) = (1, 1)

R3

espaos ve toriais sobre

uma apli ao linear de

um monomorsmo se e s se

n = 0 ;

em

K tais que E
E . Ento:

tem

de fa to, pela proposio ante-

rior,

(ii)

Nuc = {0E } dim(Nuc ) = 0 n = 0.

monomorsmo

um epimorsmo se e s se

Im = E

se e s se

c = dim(E );

de

fa to,

(iii)

epimorsmo

Im = E
dim(Im ) = dim(E ) c = dim(E ).

um isomorsmo se e s se

Proposio 5.39.

Sejam

n = 0

c = dim(E ) = dim E .

monomorsmo se e s
Demonstrao. Seja

K
E.

espaos ve toriais sobre

uma apli ao linear


se um epimorsmo.

dimenso (nita) e seja

p = dim E = dim(E ).

de

em

om a mesma
Ento

um

Pelo Teorema das dimenses,

p = n + c .
um monomorsmo se s se n = 0 se s se p = c se s se dim(E ) = c
se um epimorsmo.

Logo,
se s

Resulta desta proposio que para que uma apli ao linear entre espaos
ve toriais om a mesma dimenso seja bije tiva basta que seja inje tiva ou
sobreje tiva.

Exemplo 5.40.

(a, b, c) = (2a, b + c, b c),

je tiva.

: R3 R3 denida por
(a, b, c) R3 . Averige-se se bi-

Considere a apli ao linear


para todo


dim E = dim R3 = dim(E ), basta mostrar que sobreje tiva pois
proposio anterior garante que se sobreje tiva ento tambm inje tiva.
Como

Ora

(R3 ) = {(a, b, c) : (a, b, c) R3 }

= {(2a, b + c, b c) : a, b, c R}
= {(2a, 0, 0) + (0, b, b) + (0, c, c) : a, b, c R}

= {a(2, 0, 0) + b(0, 1, 1) + c(0, 1, 1) : a, b, c R}


= h(2, 0, 0), (0, 1, 1), (0, 1, 1)i = R3 .

(justique!)

5.3 N leo e imagem

125

sobreje tiva, logo inje tiva e, onsequentemente,

Con lui-se assim que


bije tiva.

Teorema 5.41.
linear de

Sejam E e E espaos ve toriais sobre

em E e seja F um subespao ve torial de

(1)

se

(2)

F = hv1 , . . . , vk i,

Se

K. Seja uma apli ao


E . Ento:

(F ) = h(v1 ), . . . , (vk )i;

ento

nitamente gerado, ento

(F )

tambm o e

dim((F )) dim F.
Demonstrao. Prove-se

(1). Para mostrar a igualdade entre os dois onjuntos

tem de se mostrar as duas in luses:

(i) (F ) h(v1 ), . . . , (vk )i

(ii) h(v1 ), . . . , (vk )i (F )


Prove-se

(i).

lado, omo

Seja v (F ). Ento existe u F tal que v = (u). Por


F = hv1 , . . . , vk i, existem es alares 1 , . . . , k K tais que

outro

u = 1 v1 + + k vk
e, portanto,

v = (u) = (1 v1 + + k vk )
= 1 (v1 ) + + k (vk )
v h(v1 ), . . . , (vk )i.

o que equivale a dizer que


Prove-se
que

(ii). Seja v h(v1 ), . . . , (vk )i. Ento existem 1 , . . . , k K tais


v = 1 (v1 ) + + k (vk ).

(vi ) (F ), pois vi F , para todo i {1, . . . , k}, e (F ) subespao


E , ento v (F ).
Prove-se (2). Se F nitamente gerado ento F = hv1 , . . . , vk i, para alguns
ve tores v1 , . . . , vk F , e, pela alnea anterior, (F ) = h(v1 ), . . . , (vk )i, ou
seja, (F ) nitamente gerado.
Agora, se F = {0E } ento (F ) = {(0E )} = {0E . Logo dim((F ))
dim F . Se F 6= {0E } seja (v1 , . . . , vk ) uma base de E , omo
Como

ve torial de

(F ) = h(v1 ), . . . , (vk )i,


ento

dim((F )) k = dim E .

Observao 5.42.

Sejam

F
dim((F )) = dim(F ).

ao linear inje tiva e


ento

espaos ve toriais sobre

um subespao ve torial de

E,

K.

Se

uma apli-

nitamente gerado,

5.4 Isomorsmos

126

5.4 Isomorsmos
Denio 5.43.

isomorfos

dizem-se
de

espaos ve toriais sobre K. Os espaos E e E

, e representa-se por E E , se existe um isomorsmo

Sejam

em

E.

Proposio 5.44.
E

morsmo de

Sejam E e E espaos ve toriais sobre K. Se

em E , ento
um isomorsmo de E em E .

um iso-

bije tiva, tambm 1 bije tiva (justique!). Resta


tambm apli ao linear.

Sejam , K e u, v E . Como sobreje tiva, existem a, b E tais que


(a) = u e (b) = v . Donde

Demonstrao. Como
1

mostrar que

1 (u + v) = 1 ((a) + (b)).
Assim,

1 (u + v) = 1 ((a + b))

= 1 (a + b)
= a + b
1

Teorema 5.45.

(u) +

E, E

Sejam

(v)

pois

apli ao linear

por denio de apli ao omposta


pois

pois

a funo identidade em

a=

(u)eb =

espaos ve toriais sobre

K.

(v).

Ento,

(a) E E ;

(b)

se

( )

E E,

se

E E

E E;

ento
e

E E ,

(a).

Demonstrao. Prove-se
denida por

ento

idE (u) = u,

E E .

Basta onsiderar a apli ao

para todo

linear bije tiva.


Prove-se

(b).

Prove-se

( ). Se E E

u E.

Prove que

idE

idE : E E

uma apli ao

ento existe uma apli ao linear de E em E


1
bije tiva. Assim, pela proposio anterior,
tambm uma apli ao linear

bije tiva e, portanto, E E .


Se

E E,

existe um isomorsmo

existe um isomorsmo de E em

de E em E . Logo a apli ao

E e se E E
: E E

uma apli ao linear bije tiva, pois a omposta de duas apli aes bije tivas

bije tiva e a omposta de duas apli aes lineares linear (veja-se o Exer io
5.8).

Teorema 5.46.

Sejam

menso nita. Ento

e E espaos ve toriais sobre K tais que E tem die E so isomorfos se e s se dim E = dim(E ).

5.4 Isomorsmos

127

Demonstrao. () Como

so isomorfos, existe um isomorsmo de


E em E . Como inje tiva, pela proposio 5.34 vem que Nuc = {0E } e
Im = E , ou seja dim(Nuc ) = 0 e dim(Im ) = dim E . Logo, pelo teorema
e

da dimenso, tem-se

dim E = dim(Nuc ) + dim(Im ) = dim E .


B = (e1 , . . . , en ) bases de ordenadas de E e
E , respe tivamente. Considere a apli ao linear de E em E denida por
(ei ) = ei , para i = 1, . . . , n. Como bije tiva (prove!) ento E e E so

() Sejam

B = (e1 , . . . , en )

isomorfos.

Corolrio 5.47.
E

isomorfo a

Seja

um espao ve torial sobre

tal que

dim E = n.

Ento

Kn .

Exemplo 5.48.

Os espaos ve toriais reais

M23 (R)

R6

so isomorfos, pois

tm dimenso nita e

dim(M23 (R)) = 6 = dim R6 .


Por exemplo, a apli ao



a
d

b
e

c
f



: M23 (R) R6

= (a, b, c, d, e, f ),

denida por

para todo

a
d


b c
M23 (R),
e f

M23 (R) em R6 . De fa to, inje tiva e uma apli ao


linear. Prove-se que inje tiva. Ora





a b c
a b c
Nuc =
M23 (R) :
= (0, 0, 0, 0, 0, 0)
d e f
d e f



a b c
=
M23 (R) : (a, b, c, d, e, f ) = (0, 0, 0, 0, 0, 0)
d e f


0 0 0
=
0 0 0

um isomorsmo de

Logo

inje tiva. Como

quentemente, bije tiva.

dim (M23 (R)) = dim R6 , sobreje tiva e, onseA prova de que uma apli ao linear  a omo

exer io.

Exer io 5.49.

Mostre que

Pn [x]

Rn+1

so isomorfos.

5.5 Matriz de uma apli ao linear

128

5.5 Matriz de uma apli ao linear


Na se o que se segue todos os espaos ve toriais tm dimenso nita.

Denio 5.50.

espaos ve toriais sobre K de dimenso n e


p, respe tivamente. Sejam ainda B1 = (e1 , . . . , en ) uma base ordenada de E ,
B2 = (e1 , . . . , ep ) uma base ordenada de E e uma apli ao linear de E

em E . Ento a matriz de em relao s bases B1 e B2 , denotada por


M (; B1 , B2 ), a matriz do tipo p n dada por:

a11 a12 a1n


a21 a22 a2n

M (; B1 , B2 ) = .
.
.
..
.
.
..
.
.
.
ap1 ap2 apn
onde

Sejam

(e1 ) = a11 e1 + a21 e2 + + ap1 ep

(e2 ) = a12 e1 + a22 e2 + + ap2 ep

ou seja,

.
.
.

(en ) = a1n e1 + a2n e2 + + apn ep

(ei ) = (a1i , a2i , . . . , api )B2 ,

para todo

i {1, . . . , n}.

M (; B1 , B2 )
i {1, . . . , n}.

onstituda pelas

: R2 R3 uma apli ao
(x, y) = (2x, x y, 3y), para todo (x, y) R2 . Sejam

linear denida por

Por outras palavras, a oluna


oordenadas de

(ei )

Exemplos 5.51.

na base

B2 ,

da matriz

para todo

1. Seja

BR2 = ((1, 0), (0, 1))


bases ordenadas de

R2

(a) Cal ule-se

B = ((1, 1), (1, 2))

BR3 = ((1, 0, 0), (0, 1, 0), (0, 0, 1))


bases ordenadas de

B = ((1, 1, 1), (1, 1, 0), (1, 0, 0))

R3 .

M (; BR2 , BR3 ).

Tem-se

(1, 0) = (2, 1, 0) = 2(1, 0, 0) + 1(0, 1, 0) + 0(0, 0, 1) = (2, 1, 0)BR3


(0, 1) = (0, 1, 3) = 0(1, 0, 0) + (1)(0, 1, 0) + 3(0, 0, 1)
= (0, 1, 3)BR3 ,
donde

2
M (; BR2 , BR3 ) = 1
0

0
1 .
3

5.5 Matriz de uma apli ao linear


(b) Cal ule-se

M (; B, B ).

129

Tem-se

(1, 1) = (2, 0, 3) = 3(1, 1, 1) + (3)(1, 1, 0) + 2(1, 0, 0) = (3, 3, 2)B


(1, 2) = (2, 3, 6) = 6(1, 1, 1) + (9)(1, 1, 0) + 1(1, 0, 0)
= (6, 9, 1)B ,

donde

2. Seja
para

3
6
M (; B, B ) = 3 9 .
2
1

: R3 R2 uma apli ao
todo (a, b, c) R3 , e sejam

linear tal que

B = ((1, 1, 2), (0, 2, 6), (0, 0, 4))


bases de

R3

R2 ,

(a, b, c) = (2a + b, c),

B = ((1, 0), (0, 2))

respe tivamente. Determine-se

M (; B, B ).

Tem-se

(1, 1, 2) = (3, 2) = 3(1, 0) + (1)(0, 2) = (3, 1)B


(0, 2, 6) = (2, 6) = 2(1, 0) + (3)(0, 2) = (2, 3)B
(0, 0, 4) = (0, 4) = 0(1, 0) + 2(0, 2) = (0, 2)B ,
pelo que

M (; B, B ) =

Exer io Resolvido 5.52.

Seja

BP2 [x] = (1, x, x2 )

3
2
1 3

: P2 [x] R3

(ax2 + bx + c) = (2b, b 3a, a),


Sejam ainda

para todo

a base anni a de

0
2

uma apli ao linear tal que

ax2 + bx + c P2 [x].
P2 [x]

B = ((1, 0, 1), (1, 1, 0), (0, 0, 1))


uma base ordenada de

R3 .

Determine

M (; BP2 [x] , B).

Resoluo: Tem-se

(1) = (0, 0, 0) = 0(1, 0, 1) + 0(1, 1, 0) + 0(0, 0, 1) = (0, 0, 0)B


(x) = (2, 1, 0) = 1(1, 0, 1) + 1(1, 1, 0) + (1)(0, 0, 1) = (1, 1, 1)B
(x2 ) = (0, 3, 1) = 3(1, 0, 1) + (3)(1, 1, 0) + (2)(0, 0, 1) = (3, 3, 2)B .
Logo,

0
1
3
1 3 .
M (; BP2 [x] , B) = 0
0 1 2

5.5 Matriz de uma apli ao linear


Exer io 5.53.

Sejam

130

: P2 [x] P1 [x]

(ax2 + bx + c) = (a b)x + 2c,

uma apli ao linear tal que

para todo

ax2 + bx + c P2 [x].

B = (3, 2 + x, x2 1) e B = (x 2, x)

respe tivamente. Determine M (; B, B ).

Sejam ainda

bases de

P2 [x]

P1 [x],

Uma apli ao linear tambm poder  ar denida a partir de uma sua


matriz em relao a bases previamente xadas no espao ve torial domnio e no
espao ve torial hegada.

Teorema 5.54.

Sejam E e E espaos ve toriais sobre K e sejam B1 e B2 bases

e E , respe tivamente. Sejam ainda uma apli ao linear de

ordenadas de E
em E tal que

A = M (; B1 , B2 ) e u E . Se C a matriz oluna formada


u E relativamente base B1 ento AC a matriz

pelas oordenadas de (u) E em relao base B2 .

pelas oordenadas do ve tor


oluna formada

B1 = (e1 , . . . , en )
1 , . . . , n K tais

B2 = (e1 , . . . , ep )

Demonstrao. Suponha-se que

u E.

que

Ento existem es alares

e seja

u = 1 e1 + + n en .
T
2 . . . n , isto , C a matriz oluna formada pelas oorSeja C = 1
denadas do ve tor u E relativamente base B1 e sejam 1 , 2 , . . . , p K tais
T

. . . p . Como uma apli ao linear e atendendo a
que AC = 1
 2

e2 . . . ep Ai , onde Ai representa a oluna i da matriz A,
que (ei ) = e1
para i = 1, . . . , n, tem-se


(u) = 1 (e1 ) + + n (en )



= (e1 ) (e2 )

= e1

e2


2
(en ) .
..
n


ep AC

= 1 e1 + 2 e2 + + p ep
= (1 , 2 , . . . , p )B2 .

Portanto,

AC

relao base

a matriz oluna formada pelas oordenadas de

B2 .

Exemplo 5.55.
nada de

R3

(u) E

em

Considere-se B = ((1, 1, 2), (0, 2, 6), (0, 0, 4)) uma base ordeB = ((1, 0), (0, 2)) uma base ordenada de R2 . Seja : R3 R2

uma apli ao linear tal que

A = M (; B, B ) =

3
1

2 0
3 2

5.5 Matriz de uma apli ao linear


Determine-se

131

(1, 3, 6).

ne essrio primeiro determinar as oordenadas do ve tor


na base

B.

Ora

u = (1, 3, 6)

(1, 3, 6) = 1 (1, 1, 2) + 2 (0, 2, 6) + 3 (0, 0, 4)


equivalente a

1 = 1
1 = 1
1 + 22 = 3
2 = 2

21 + 62 43 = 6
3 = 1

Pode ento es rever-se:

(1, 3, 6) = 1(1, 1, 2) + (2)(0, 2, 6) + (1)(0, 0, 4) = (1, 2, 1)B .


De a ordo om o teorema anterior, as oordenadas de

(1, 3)

(u)

na base

pois

so

Logo





1
1
3
2 0
1
2 =
A 2 =
.
1 3 2
3
1
1

(u) = (1, 3)B

e, portanto,

(1, 3, 6) = 1(1, 0) + 3(0, 2) = (1, 6).

Exer io Resolvido 5.56.

: R2 R3

Seja

em relao s bases ordenadas

B1 = ((1, 1), (1, 2))


de

R2

R3 ,

a apli ao linear uja matriz

B2 = ((1, 1, 1), (1, 1, 0), (1, 0, 0))

respe tivamente, :

Cal ule

(1, 0)

(x, y),

3
6
A = 3 9 .
2
1

para todo

(x, y) R2 ,

re orrendo matriz

Resoluo: Ora

(1, 0) = (1, 1) + (1, 2)


Donde

(1, 0) =

1
2
3 , 3 B1 . Ento

2
3
31

=1

+ 2 = 0

= 23
= 31

3
6  2 
0
3
= 3 9
= 1
13
2
1
1

A.

5.5 Matriz de uma apli ao linear

132

e, portanto,

(1, 0) = (0, 1, 1)B2 = 0(1, 1, 1) + 1(1, 1, 0) + 1(1, 0, 0) = (2, 1, 0).


Analogamente, verique que

(x, y) =
Donde

y+2x
3
yx
3

so as oordenadas de

y + 2x
yx
(1, 1) +
(1, 2).
3
3

3
6 
= 3 9
2
1

(x, y)

na base

B2 .

y+2x
3
yx
3

Logo

3y
= x 4y
x+y

(x, y) = (3y, x 4y, x + y)B2


= 3y(1, 1, 1) + (x 4y)(1, 1, 0) + (x + y)(1, 0, 0)
= (2x, x y, 3y),

para todo

(x, y) R2 .

Matriz da adio, da multipli ao por um es alar e da omposta de


apli aes lineares

Sejam E e E espaos ve toriais sobre K. Sejam e apli aes lineares de E

em E . Como j se viu + uma apli ao linear de E em E e, qualquer que


seja

K,

o mesmo su ede a

Ento:

Teorema 5.57.
lineares de
seja

K.

Seja E e E espaos ve toriais sobre K e sejam e apli aes


em E . Sejam ainda B e B bases de E e E , respe tivamente, e

Se

A = M (; B, B )

C = M (; B, B )

ento,

M ( + ; B, B ) = A + C
e

M (; B, B ) = A.
Demonstrao. Prove!
Pode tambm denir-se a matriz da omposta de duas apli aes lineares
tendo por base as matrizes de ada uma das apli aes lineares.

Teorema 5.58.

E , E e E espaos ve toriais sobre K. Sejam B , B


e B bases ordenadas de E , E e E , respe tivamente. Seja uma apli ao
linear de E em E uja matriz em relao s bases B e B
Sejam

A = M (; B, B ).

5.5 Matriz de uma apli ao linear


Seja uma apli ao linear de
B

em

133
uja matriz em relao s bases

C = M (; B , B ).

Ento

M ( ; B, B ) = M (; B , B )M (; B, B ) = CA.

Exemplo 5.59.

: R3 R2

Seja

uma apli ao linear tal que

(x, y, z) = (x + y + z, y 2z),
Determine-se a matriz de

para todo

(x, y, z) R3 .

em relao s bases anni as de

R3

R2 .

Tem-se

(1, 0, 0) = (1, 0) = 1(1, 0) + 0(0, 1) = (1, 0)BR2


(0, 1, 0) = (1, 1) = 1(1, 0) + 1(0, 1) = (1, 1)BR2
(0, 0, 1) = (1, 2) = 1(1, 0) + (2)(0, 1) = (1, 2)BR2 .
Assim,

A = M (; BR3 , BR2 ) =
Seja

: R2 R3

tal que

Determine-se a matriz de

1
0

1
1
1 2

(x, y) = (x, x + y, x y),

para todo
R2 e

em relao s bases anni as de

(x, y) R2 .
R3 . Tem-se

(1, 0) = (1, 1, 1) = 1(1, 0, 0) + 1(0, 1, 0) + 1(0, 0, 1) = (1, 1, 1)BR3


(0, 1) = (0, 1, 1) = 0(1, 0, 0) + 1(0, 1, 0) + (1)(0, 0, 1) = (0, 1, 1)BR3 .
Logo

1
C = M (; BR2 , BR3 ) = 1
1

Por m, determine-se a matriz de

0
1 .
1

em relao base anni a de

sem denir a apli ao omposta. Sabe-se que

M ( ; BR3 , BR3 ) = M (; BR2 , BR3 )M (; BR3 , BR2 ) = CA.


Assim

1
M ( ; BR3 , BR3 ) = 1
1

0 
1 1
1
0 1
1

1
2

1 1
= 1 2
1 0

1
1 .
3

R3 ,

5.5 Matriz de uma apli ao linear

134

5.5.1 Isomorsmo entre L(E, E ) e Mpn (K)

espaos ve toriais sobre K tais que dim E = n e dim E

Re orde-se que L(E, E ) o onjunto das apli aes lineares de E em

Sejam

= p.
E e

Mpn (K) o onjunto das matrizes p n om entradas em K. Tambm j se viu

que o onjunto L(E, E ) munido om as operaes usuais de adio de apli aes


e multipli ao de uma apli ao por um es alar um espao ve torial sobre K
e que o onjunto Mpn (K) munido om as operaes de adio de matrizes e
de multipli ao de uma matriz por um es alar um espao ve torial sobre K.
Prova-se que:

Teorema 5.60.

espaos ve toriais sobre K tais que dim E = n


dim E = p. Sejam B e B bases ordenadas de E e E , respe tivamente. A

apli ao : L(E, E ) Mpn (K) tal que () = M (; B, B ), para todo

L(E, E ), um isomorsmo.
Sejam

1 0
.. ..
= . .
0 0

...
...

Observao 5.61.

M1,1

dim Mpn (K) = pn pois as matrizes

... 0
0 0 ... 0
. ..
.
.
. , . . . , Mp,n = .
. ,
.
.
.
.
... 0
0 0 ... 1

simples veri ar que

0
0 1

.
.
.
.
. , M1,2 = .
.
.
.
0
0 0

ou seja, na forma geral, as matrizes

onde

mij = 1

mks = 0,

Logo



Mi,j = mks Mpn (R),

para

k 6= i

s 6= j ,

onstituem uma base de

Mpn (K).

dim (L(E, E )) = pn,


pois, pelo teorema anterior, os espaos

L(E, E )

Mpn (K)

so isomorfos.

5.5.2 Matrizes invertveis e isomorsmos


Teorema 5.62.

Sejam E e E espaos ve toriais sobre K om a mesma dimen

so n e sejam B e B bases ordenadas de E e E , respe tivamente. Seja uma

apli ao linear de E em E tal que A = M (; B, B ). Ento A uma matriz

invertvel se e s se

um isomorsmo. Mais,

Demonstrao. () Suponha-se que

A1 = M (1 ; B , B).

inje tiva e, pela Proposio


5.34, basta mostrar que Nuc = {0E }. Seja v Nuc e seja X a matriz oluna
formada pelas oordenadas de v na base B . Ento
invertvel. Pretende-se provar que

bije tiva. Pela Proposio 5.39, basta provar que


v Nuc (v) = 0E AX = 0

T
0 ,

5.6 Matriz de mudana de base

135

pela Proposio 5.54. Note-se que 0E = (0, 0, . . . , 0)B . Como


A1 e, multipli ando esquerda a igualdade anterior por

existe

A
A1

invertvel,
em ambos

os membros, obtm-se


A1 (AX) = A1 0

T


X = 0

T
0 ,

v = (0, 0, . . . , 0)B . Logo v = 0E , donde Nuc = {0E }. Provou-se que


E e E tm a mesma dimenso ento sobreje tiva. Portanto,

isto ,

inje tiva. Como

um isomorsmo.

() Re ipro amente, suponha-se que um isomorsmo. Como bije 1


tiva ento a apli ao inversa
est bem denida e tambm um isomorsmo.

1
Seja C = M
; B , B . Ento,

AC = M (; B, B ) M 1 ; B , B

= M 1 ; B , B
= M (idE ; B , B )

= In .

Portanto,

invertvel e

A.

a matriz inversa de

5.6 Matriz de mudana de base


Denio 5.63.
B

Seja

um espao ve torial sobre

bases ordenadas de

E.

P = M (idE ; B, B )

hama-se
M (B, B ).

K de

dimenso nita. Sejam

matriz

matriz de mudana de base de

para

B,

e representa-se por

Re orde-se que

idE : E
u
Dado um ve tor

v E,

obter as oordenadas de
De fa to, se
ento

PX

a matriz de mudana de base atrs denida, permite

na base B a partir das oordenadas de v na base B .

for a matriz oluna formada pelas oordenadas de

ser a matriz oluna formada pelas oordenadas de

Exer io 5.64.

No espao ve torial real

B = ((1, 0, 1), (0, 1, 0), (1, 0, 1)


Determine as oordenadas de
base

B,

E
7 idE (u) = u

R3 ,

na base
B.

na base

onsidere-se as seguintes bases:

B = ((1, 1, 1), (1, 1, 0), (1, 0, 0)) .

u = (1, 2, 3)

na base

usando as matrizes de mudana de base.

e de

v = (2, 0, 4)

na

5.6 Matriz de mudana de base


Teorema 5.65.

136

Uma matriz de mudana de base invertvel.

Demonstrao. Uma matriz de mudana de base uma matriz da apli ao


linear

idE ,

onde

um espao ve torial sobre

K.

Como

idE

um isomorsmo,

ento qualquer matriz de mudana de base invertvel.

Teorema 5.66.

Toda a matriz invertvel pode ser onsiderada uma matriz de

mudana de base.
Atendendo ao Teorema 5.62, f il provar

Teorema 5.67.
B

Seja

um espao ve torial sobre

so duas bases ordenadas de

E.

M (B, B ) = (M (B , B))

Exemplo 5.68.

No espao ve torial real

B = ((1, 1), (2, 1))


al ular as oordenadas de

matriz de mudana de base de

R2 ,

onsidere-se as seguintes bases:

B = ((1, 0), (1, 1)) .

(a) Suponha que as oordenadas de

de dimenso nita tal que

Ento

v R2

na base

para

na base B so (2, 3). Pretende-se


B . Primeiro vai-se determinar a
B . Ora

idR2 (1, 1) = (1, 1) = 2(1, 0) + (1)(1, 1) = (2, 1)B


idR2 (2, 1) = (2, 1) = 3(1, 0) + (1)(1, 1) = (3, 1)B .


2
3

Portanto, P = M (B, B ) =
. Como
1 1


 

2
3
2
13
=
,
1 1
3
5
ento

v = (13, 5)B .

Veri ao: Se

v = (2, 3)B ,

ento

v = 2(1, 1) + 3(2, 1) = (8, 5).

Por outro

lado,

(13, 5)B = 13(1, 0) + (5)(1, 1) = (8, 5) = v.


w = (1, 2)B e determine-se as oordenadas de
Comea-se por determinar a matriz de mudana de base de

(b) Suponha-se agora que


na base
para

B.

B.

Note-se que

M (B , B) = M (idR2 ; B , B)
=
=


1
M id1
R2 ; B, B
1
(M (idR2 ; B, B ))
1

=P

w
B

por denio de matriz de mudana de base


pelo Teorema 5.62
pois

2
id1
R2 = idR

5.7 Relao entre matrizes de uma mesma apli ao linear


Verique que

P 1 =

so dadas por

e, portanto,

1 3
1
2

. Donde as oordenadas de



1 3
1
2

1
2

7
5

137
na base

w = (7, 5)B .

5.7 Relao entre matrizes de uma mesma apli ao linear

Sejam E e E espaos ve toriais sobre K e seja uma apli ao linear de E em

E . Sejam ainda B1 e B2 bases ordenadas de E e B1 e B2 bases ordenadas de


E . Suponha-se que

A = M (; B1 , B1 )

C = M (; B2 , B2 ).

Considere-se o seguinte diagrama:

A
E

E
(B1 )

(B 1 )

idE

idE

(B2 )

(B 2 )

C
Tem-se que

= idE idE

e, matri ialmente, pelo Teorema 5.58,

M (; B2 , B2 ) = M (idE ; B1 , B2 ) M (; B1 , B1 ) M (idE ; B2 , B1 ).
ou seja,

C = P AQ.

Observao 5.69.
em

E,

Exemplo 5.70.
R

Note-se que

so duas matrizes de mudana de base

respe tivamente.

: R2 R3 a apli ao
(x, y) = (2x, x y, 3y). Sejam
Seja

B1 = ((1, 0), (0, 1))


bases ordenadas de

R2

linear tal que, para todo

(x, y)

B2 = ((1, 1), (1, 2))

B1 = ((1, 0, 0), (0, 1, 0), (0, 0, 1))

B2 = ((1, 1, 1), (1, 1, 0), (1, 0, 0))

5.7 Relao entre matrizes de uma mesma apli ao linear


bases ordenadas de

R3 .

Sejam ainda

2
0
A = M (; B1 , B1 ) = 1 1
0
3

Obtenha-se

138

a partir de

C = M (; B2 , B2 ).

e de matrizes de mudana de base onvenientes.

Esquemati amente, tem-se,

A
R2
(B1 )
Q

R3
(B 1 )

idR2

(B2 )
R2

idR3

(B 2 )

C
onde

Q = M (idR2 ; B2 , B1 ) = M (B2 , B1 ) =
e

1 1
1
2

0
0
1
1 1 ,
P = M (idR3 ; B1 , B2 ) = M (B1 , B2 ) = 0
1 1
0

pois

(1, 1) = (1, 1)B1


(1, 2) = (1, 2)B1
(1, 0, 0) = 0(1, 1, 1) + 0(1, 1, 0) + 1(1, 0, 0) = (0, 0, 1)B2
(0, 1, 0) = 0(1, 1, 1) + 1(1, 1, 0) + (1)(1, 0, 0) = (0, 1, 1)B2
(0, 0, 1) = 1(1, 1, 1) + (1)(1, 1, 0) + 0(1, 0, 0) = (1, 1, 0)B2 .
Assim,

0
C = P AQ = 0
1

Exer io 5.71.

Seja

0
1
2
1 1 1
1
0
0
: R2 R3

0 
1
1
1
3

1
2

3
= 3
2

uma apli ao linear tal que

1
M (; BR2 , BR3 ) = 1
0

2
1 .
1

6
9 .
1

5.7 Relao entre matrizes de uma mesma apli ao linear

139

BR2 e BR3 so as bases anni as de R2 e R3 , respe tivamente. Determine


M (; B, B ), onde B = ((1, 1), (1, 2)) e B = ((1, 1, 1), (1, 1, 0), (1, 0, 0)) so
2
3
bases ordenadas de R e R , respe tivamente.
onde

Denio 5.72.

Sejam A e C matrizes do tipo p n om entradas em K. DizC so equivalentes se existem matrizes invertveis P Mpp (K) e
Q Mnn (K) tais que
C = P AQ.
se que

A e C so matrizes quadradas de ordem n


S Mnn (K) tal que C = S 1 AS , ento diz-se
Se

e existe uma matriz invertvel


que

so

semelhantes.

Teorema 5.73.

Sejam A, C Mpn (K). As matrizes A e C so equivalentes


se e s se representam a mesma apli ao linear em relao a determinadas
bases.

Teorema 5.74.

Sejam A, C Mnn (K). As matrizes A e C so semelhantes


se e s se representam o mesmo endomorsmo em relao a diferentes bases.

K de dimenso n, B
E tal que A = M (; B, B). Como A
e C so semelhantes, existe uma matriz S Mnn (K) invertvel tal que C =
S 1 AS . Pelo teorema 5.66 existe uma base B de E tal que S = M (B , B). Ento
C = M (; B , B ).
() Re ipro amente, se A e C representam o mesmo endomorsmo ento
1
pelo teorema anterior, C = P AQ. E fa ilmente se veri a que P = Q
e,
onsequentemente, A e C so semelhantes.
Demonstrao. () Sejam
uma base de

um espao ve torial sobre

o endomorsmo de

Observao 5.75.

Se A a matriz de um endomorsmo em relao a uma


B e C a matriz do mesmo endomorsmo mas em relao a uma base B
ento A e C so semelhantes.
base

Exemplo 5.76.
anni a de

Seja

o endomorsmo de

Determine-se a matriz de

A=

1 0
0 1
1 0

R3

uja matriz em relao base

1
1 .
1

B = ((1, 0, 1), (0, 1, 1), (0, 0, 1)),


C = M (; B , B ). Ento, esquemati-

em relao base

usando matrizes de mudana de base. Seja


amente,

A
R3
(BR3 )

R3

(BR3 )
P 1

P
(B )
R3

(B )

R
C

5.7 Relao entre matrizes de uma mesma apli ao linear


e, portanto,

C = P 1 AP ,

onde

1 0
P = M (B , BR3 ) = 0 1
1 1

Logo,

1
0 0
1 0
1 0 0 1
C= 0
1 1 1
1 0

Note-se que

Exer io 5.77.

(x, y) R

0
0
1

verique!

0
0 .
1

1
0
= M (BR3 , B ) = 0
1
1 1

P 1

1
1
1 0
1
1

verique!


0 0
0 1
1 0 = 1
2
1 1
1
0

so semelhantes.

: R2 R3 a apli ao linear
(x, y) = (x + y, x y, x + 2y). Sejam ainda
Seja

B = ((1, 1), (0, 1))


R2

bases ordenadas de

140

R3 ,

(a) Determine a matriz de

1
1 .
1

tal que, para todo

B = ((1, 1, 0), (0, 1, 1), (1, 1, 1))

respe tivamente.

em relao s bases anni as dos espaos on-

em relao s bases

siderados.
(b) Determine a matriz de

B:

(i) usando a denio;


(ii) usando matrizes de mudana de base.
( ) Determine

Teorema 5.78.

(2, 3),
Sejam

usando ambas as representaes matri iais de

espaos ve toriais sobre

nadas de E e E , respe tivamente. Seja ainda


E tal que A = M (; B, B ). Ento,

r(A)

Observao 5.79.

representa a ara tersti a da matriz

Resulta deste teorema que:

r(A) = dim(E );

(i)

um epimorsmo se e s se

(ii)

um monomorsmo se e s se

bases orde-

uma apli ao linear de

dim(Im ) = r(A).
Re orde-se que

r(A) = dim E ;

A.

em

5.7 Relao entre matrizes de uma mesma apli ao linear


(iii)

r(A) = dim E = dim(E ).

um isomorsmo se e s se

Exemplo 5.80.
vamente, e seja

B e B bases dos espaos ve toriais R4


: R R3 a apli ao linear tal que

1 2 1
0
0 .
A = M (; B, B ) = 0 1 1
1 0 1 1
Sejam
4

Cal ulando a ara tersti a da matriz


Es alonando a matriz

1 2
0 1
1 0

141

A,

averige-se se

R3 ,

respe ti-

sobreje tiva.

tem-se,

1
0
1
0
1 1

1 2
0 1
L3 := L3 + L1
0 2

1 2

0 1
L3 := L3 2L2
0 0

1
0
1
0
2 1

1
0
1
0 .
0 1

dim(Im ) = r(A) = 3. Como Im um subespao ve torial de R3 e


dim(Im ) = dim(R3 ), ento Im = R3 e, onsequentemente, sobreje tiva.
Logo,

Exer ios 5.81.

1. Considere a apli ao linear de R3 em R2 uja matriz


em relao base anni a de R3 e base B = ((1, 1), (1, 1)) de R2

A=
Usando a matriz

A,

determine


1
.
0

1 1
2 1

Nuc

e lassique a apli ao quanto

inje tividade e sobreje tividade.


2. Considere a apli ao linear

de

R3

em

R4

(1, 1, 1) = (1, 0, 0, 0), (1, 1, 0) = (1, 0, 0, 0)


onde

denida por
e

(1, 0, 0) = (k, 1, k, k 1),

um parmetro real.

Diga para que valores de

a apli ao

(a) monomorsmo;
(b) epimorsmo.
Sugesto: use uma matriz de

em relao a bases onvenientemente

es olhidas.

6. Valores e ve tores prprios

6.1 Valores e ve tores prprios

143

6.1 Valores e ve tores prprios


Denio 6.1.
de

E.

ve tor

E um espao ve torial sobre K e seja um endomorsmo


K. Diz-se que um valor prprio de se existe um
que v 6= 0E e

Seja

Seja ainda

vE

tal

(v) = v.

ve tor prprio de asso iado ao valor prprio .


Denio 6.2. Chama-se espe tro de , e representa-se por (), ao onjunto

Ao ve tor

hama-se

de todos os valores prprios de

Exemplo 6.3.
morsmo

Determine-se os valores prprios e ve tores prprios do endo-

de R3

denido, em relao base anni a desse espao, pela matriz

1
0
1
A = 1
1 1

en ontrar R para o qual existe (x, y, z)


(x, y, z) = (x, y, z). Ora, pelo Teorema 5.54, pode passar-se

Determinar um valor prprio de


no nulo tal que

1
0 .
0

para representao matri ial:

1
0 1
x
x
(x, y, z) = (x, y, z) 1
1 0 y = y
1 1 0
z
z


1
0 1
x
x
0
1
1 0 y y = 0
1 1 0
z
z
0



1
0 1
0 0
x
1 0 0 0 y
1
1 1 0
0 0
z

0
x
1
0
1
0 y = 0 .
1 1
0
z
1
1

Portanto, ne essrio en ontrar

0
= 0
0

para o qual o sistema anterior admite,

pelo menos, uma soluo no trivial, ou seja, tal que a ara tersti a da matriz
do sistema seja inferior a

3, ou, equivalentemente, tal


tal que


1
0
1

1
1 0 = 0.

1
1

no seja invertvel. Assim,

que a matriz do sistema

6.1 Valores e ve tores prprios


Assim:


1

1

1

0
1
1 0
1

144




= 0 (1 )(1 )() + 1(1 (1 )) = 0



(1 )2 1 = 0
= 0 (1 )2 1 = 0
= 0 = 2.

Assim, o espe tro de

o onjunto

() = {0, 2}.

Determinem-se os ve tores prprios de asso iados a


(x, y, z) R3 \ {(0, 0, 0)} tais que (x, y, z) =

so todos

= 0. Por denio,
0(x, y, z), ou seja,

(x, y, z) = (0, 0, 0). Novamente equivalente a resolver o sistema


1
0 1
x
0
x+z =0
z = x
1

1 0
y = 0
x + y = 0
y=x

1 1 0
z
0
xy =0
Logo, os ve tores prprios de

asso iados a

=0

so da forma

x R \ {0}.

(x, x, x),

om

Note-se que se determinaram as oordenadas dos ve tores prprios


3
na base anni a de R :

(x, x, x)BR3 = x(1, 0, 0) + x(0, 1, 0) x(0, 0, 1) = (x, x, x).


Determinem-se agora os ve tores prprios de

asso iados a

= 2.

Usando

o mesmo ra io nio, basta resolver o sistema

1
0 1
x
x
1
1
1 0 y = 2 y 1
1 1 0
z
z
1

0
1
x
0
1
0 y = 0 .
1 2
z
0

Passando para matriz ampliada, obtm-se:

1
0
1 0
0
1
1

L2 := L2 L1
1 1
0 0 L
0
1
1

3 := L3 + L1
1 1 2 0
0 1 1

1
0
1 0

0 1 1 0
L3 := L3 L2
0
0
0 0

Logo,

= 2.

(z, z, z),

Exer io 6.4.



a
c

om

z R \ {0},

b
d

2c
b 2c

so os ve tores prprios de

Considere o endomorsmo



0
0
0

a+c
,
d

x=z
y = z

asso iados a

M22 (R) denido por




a b
todo
M22 (R).
c d

de

para

Determine os seus valores prprios e os ve tores prprios asso iados.

6.1 Valores e ve tores prprios

145

Atendendo ao exemplo anterior, es reva-se o teorema:

Teorema 6.5.

Seja

base ordenada de

(a) K

E.

K de dimenso n e B uma
E e A = M (; B, B). Ento:

um espao ve torial sobre

Seja

um endomorsmo de

valor prprio de

se e s se

|A In | = 0.

(b) v E
X0

ve tor prprio de

se e s se, sendo
B , X0 uma soluo

asso iado ao valor prprio

a matriz oluna das oordenadas de

na base

no nula do sistema de equaes lineares

(A In )X = 0.

(a). Seja K. Ento valor prprio de se e


v E tal que v 6= 0E e (v) = v , ou seja, se e s se existe
X0 Mn1 (K) \ {0}, tal que AX0 = X0 , onde X0 a matriz oluna das
oordenadas de v na base B , que equivalente a
Demonstrao. Prove-se
s se existe

AX0 X0 = 0 (A In )X0 = 0.
Ou seja, se e s se o sistema homogneo
minado, isto , se e s se

(A In )X = 0

possvel e indeter-

|A In | = 0,
pelos Teorema 3.43 e Teorema 3.44.
A demonstrao de

Observao 6.6.
grau

Se

n.

Teorema 6.7.

Se

(b)  a omo exer io.

A Mnn (K)

A, B Mnn (K)

ento

|A In |

um polinmio em

de

so duas matrizes semelhantes ento

|A In | = |B In | .
Demonstrao. Como

S Mnn (K)

so matrizes semelhantes ento existe uma matriz


B = S 1 AS . Logo

invertvel tal que



|B I| = S 1 AS S 1 In S


= S 1 (A In )S


= S 1 |(A In )| |S|
1
=
|(A In )| |S|
|S|
= |B I| .

6.1 Valores e ve tores prprios

146

Pelo teorema anterior e pelo teorema 5.74 tem-se que se

A, B Mnn (K)

so duas matrizes que representam o mesmo endomorsmo ento os polinmios

|A In |

|B In |

Denio 6.8.

so iguais. Temos ento a denio seguinte.

E um espao ve torial sobre K de dimenso n e B uma


E . Seja um endomorsmo de E e A = M (; B, B). Ao
polinmio |A In | hama-se polinmio ara tersti o de , e representase por p ().
A equao p () = 0 designa-se por equao ara tersti a de .
Seja

base ordenada de

em relao a uma
|A I| = 0, ou seja, as razes do
valores prprios de .

Viu-se, no teorema 6.5, que, sendo


base xa,

valor prprio de

polinmio ara tersti o de

Denio 6.9.

uma matriz de

se e s se

so os

E um espao ve torial sobre K e seja um endomorsmo


K um valor prprio de . Chama-se multipli idade
algbri a de , e representa-se por ma (), multipli idade de enquanto raiz
do polinmio ara tersti o de , p ().
de

E.

Seja

Seja ainda

Re orde-se que 0 raiz de multipli idade k de um polinmio p() se e s se


p() = ( 0 )k q(), onde q() um polinmio que no admite 0 omo raiz.

Exemplo 6.10.
anni a de

Seja

o endomorsmo de

2
0

A=
0
0

R4

uja matriz, em relao base

1
0 0
2
0 0
.
0 1 0
0
0 1

Determine-se a multipli idade algbri a dos valores prprios de

Pelo teorema

anterior, omo


2
1
0
0

0
2
0
0

p () = |A I4 | =
0
1
0
0
0
0
0
1
= (2 )(2 )(1 )(1 )
= (2 )2 (1 )(1 ),

so

ma (2) = 2

ento os valores prprios de

Exer io Resolvido 6.11.


relao base

1, 1

2,

onde

ma (1) = ma (1) = 1.

o endomorsmo
B = ((1, 1, 1), (1, 1, 0), (1, 0, 0)) de R3 ,

1 1 0
2 1 .
A = 1
0
1 1
Seja

de

R3

uja matriz, em

6.1 Valores e ve tores prprios


Determine os valores prprios de

147
as respe tivas multipli idades algbri as e

os ve tores prprios asso iados.


Resoluo: Pelo teorema anterior, omo


1
1
0

2
1
p () = |A I3 | = 1
0
1
1

= (1 ) ((2 )(1 ) 1) (1) ((1 ) 0)

= (1 )2 (2 ) (1 ) + (1)(1 )
= (1 )((1 )(2 ) 1 1)
= (1 )((1 )(2 ) 2)

= (1 )(2 3)

= (1 )( 3),
ento os valores prprios de

so

0, 1

e, portanto,

ma (0) = ma (1) = ma (3) = 1.


Determinem-se os ve tores prprios asso iados a
o sistema

(A0I3 )X = 0 AX = 0, onde X

= 0.

Tem que se resolver

a matriz oluna das oordenadas

B . Donde,

0
1 1 0

0 L2 := L2 + L1 0
1 1
0
0
1 1

1 1 0

1 1
L3 := L3 L2 0
0
0 0

de um ve tor prprio genri o, em relao base

1 1 0
1
2 1
0
1 1

omo

0
0
0

0
0 ,
0

vem que as oordenadas de um ve tor prprio tm que satisfazer o seguinte


sistema

xy =0

y+z =0

Assim os ve tores prprios asso iados a

x=y
z = y

=0

so ve tores da forma:

v = (y, y, y)B = y(1, 1, 1) + y(1, 1, 0) + (y)(1, 0, 0) = (y, 2y, y),


om

y R \ {0}.

Determinem-se agora os ve tores prprios asso iados a


mente, omo

A 1I3

0
= 1
0

= 1.

1 0 0
1
1 1

1 1 0 L2 L1 0 1 0
1 0 0
0
1 0

1
1 1

0 1 0
L3 := L3 + L2
0
0 0

Analoga-

0
0
0

0
0 ,
0

6.1 Valores e ve tores prprios


vem que

148

x + y + z = 0

y = 0

Donde os ve tores prprios asso iados a

=1

z=x
y=0

so ve tores da forma:

v = (x, 0, x)B = x(1, 1, 1) + 0(1, 1, 0) + x(1, 0, 0) = (2x, x, x),


om

x R \ {0}.

Por m, determinem-se os ve tores prprios asso iados a

A 3I3

ento

x y + z = 0

y 2z = 0

= 3.

Como

1 1
1 0
2 1
0 0

0 0
= 1 1
1 0 L2 L1 2 1
0
1 2 0
0
1 2 0

1 1
1 0

0
1 2 0
L2 := L2 2L2
0
1 2 0

1 1
1 0

0
1 2 0 ,
L3 := L3 L2
0
0
0 0

x = z
,
y = 2z

=3

pelo que os ve tores prprios asso iados a

so ve tores da forma:

v = (z, 2z, z)B = (z)(1, 1, 1) + 2z(1, 1, 0) + z(1, 0, 0) = (2z, z, z),


om

z R \ {0}.

Exer io 6.12.
base

B=

Considere o endomorsmo de
((1, 0, 1), (1, 1, 0), (0, 0, 1)) de R3 ,

2 1
A= 2 3
3 3

R3

uja matriz, em relao

1
2 .
4

Determine os valores prprios, os respe tivos ve tores prprios asso iados e as


respe tivas multipli idades algbri as.
Seja

um espao ve torial sobre

domorsmo de

E,

K.

Dado um valor prprio

de um en-

os ve tores prprios asso iados so determinados a partir de

solues no nulas de um sistema homogneo indeterminado. Uma vez que um


sistema homogneo indeterminado tem um onjunto innito de solues que
formam um subespao ve torial de

laro que, se ao onjunto dos ve tores

prprios juntar-se o ve tor nulo do espao ve torial


ve torial de

E.

De fa to,

E,

se obtm um subespao

6.1 Valores e ve tores prprios


Teorema 6.13.
de

E.

E um espao
K. Dena-se

Seja

Seja ainda

149
ve torial sobre

Ke

seja

um endomorsmo

U = {v E : (v) = v}.
Ento:

(a) U

um subespao ve torial de

(b) U 6= {0E }

se e s se

Demonstrao. Prove-se

E.

valor prprio de

(a). Note-se que:

(i) 0E U , pois (0E ) = 0E .

(ii)

Sejam

, K

e sejam

u, v U .

Ento

(u + v) = (u) + (v)

pois

= u + v
= (u + v)
Logo

(u) = u

(v) = v .

Donde

apli ao linear

por hiptese

u U

v U

pelos axiomas de espao ve torial

u + v U .

Pelo Teorema 4.15,

um subespao ve torial de

E.

Prove-se (b). () Suponha-se que U 6= {0E }. Ento existe v 6= 0E tal que


v U , ou seja, (v) = v e, portanto, por denio, valor prprio de .
() Re ipro amente, suponha-se que (). Ento existe v 6= 0E tal que
(v) = v e, portanto, v U , ou seja, U 6= {0E }.

Denio 6.14.
de

E.

Se

Seja

um espao ve torial sobre

um valor prprio de

K e seja um endomorsmo
E denido por

ao sub onjunto de

U = {v E : (v) = v}

hama-se

subespao prprio de

asso iado ao valor prprio

Denio 6.15.

Seja E um espao ve torial sobre K e seja um endomorsmo


E . Se K um valor prprio de , hama-se multipli idade geomtri a
de , e representa-se por mg (), dimenso de U , ou seja,

de

mg () = dim(U ).

Exemplo 6.16.
base anni a de

Considere o endomorsmo
R3 ,

1 1
A = 0 2
0 2

de

0
2 .
5

R3

uja matriz, em relao

Determine-se a multipli idade geomtri a dos valores prprios de

6.1 Valores e ve tores prprios

150

Primeiro, determinem-se os valores prprios de


1
1

2
p () = |A I3 | = 0
0
2

Como

0
2
5

= (1 ) ((2 )(5 ) 4)

= (1 )(2 7 + 6)

= (1 )( 1)( 6)

ento os valores prprios de

so

ma (1) = 2 e ma (6) = 1.
= 1.

0
0 1 0 0

2 := L2 L1
0 0 2 0
0 L
L3 := L3 2L1
0 0 4 0
0

0 1 0 0

L3 := L3 2L2 0 0 2 0
0 0 0 0
e

6,

om

Determine-se o subespao prprio asso iado a

A 1I3

0
= 0
0

vem que

Logo

e, portanto,

1 0
1 2
2 4

y=0

2z = 0

vem que

mg (1) = 1

(justique!).

5
= 0
0


Donde

e, portanto,

y=0
z=0

Determine-se agora o subespao prprio asso iado a

A 6I3

U1 = {(x, 0, 0) : x R} = h(1, 0, 0)i

omo

= 6.

Analogamente,

1 0
1
0 0 5
4
2 0 L3 = L3 + 21 L2 0 4 2
0
0 0
2 1 0
5x + y = 0

4y + 2z = 0

0
0 ,
0

y = 5x
z = 10x

U6 = {(x, 5x, 10x) : x R} = h(1, 5, 10)i

mg (6) = 1

(justique!).

Exer io 6.17.

Considere o endomorsmo
3
base anni a de R ,

1 1
A = 2 2
1 1

de

1
0 .
1

R3

uja matriz, em relao

Determine a multipli idade geomtri a dos valores prprios de

6.2 Endomorsmos diagonalizveis

151

O prximo resultado estabele e uma relao entre a multipli idade algbri a


e a multipli idade geomtri a de um valor prprio de um endomorsmo.

Teorema 6.18.
de

E.

Se

Seja

um espao ve torial sobre

um valor prprio de

Ke

seja

um endomorsmo

ento,

1 mg () ma ().

Observao 6.19.

Resulta do teorema anterior que, se

uma raiz simples do polinmio ara tersti o, ento

ma () = 1,
mg () = 1.

ou seja, se

6.2 Endomorsmos diagonalizveis


O prximo resultado garante que a valores prprios distintos orrespondem ve tores prprios linearmente independentes.

Teorema 6.20.

Seja E um espao ve torial sobre K e seja um endomorsmo


E . Sejam ainda 1 , 2 , . . . , p K valores prprios de , distintos dois a dois.
Se u1 , u2 , . . . , up E so ve tores prprios de asso iados a 1 , 2 , . . . , p ,
respe tivamente, ento u1 , u2 , . . . , up so linearmente independentes.

de

p.

p = 1, u1
u1 6= 0E .
quaisquer p 1 ve tores

Demonstrao. A demonstrao feita por induo em

Para

linearmente independente pois, por denio de ve tor prprio,


Suponha-se agora que, por hiptese de induo,

p 1 valores prprios distintos so linearmente indepenu1 , u2 , . . . , up E ve tores prprios de asso iados aos valores prprios 1 , 2 , . . . , p , respe tivamente, onde i 6= j , para todo i, j
{1, . . . , p} e i 6= j . Sejam 1 , 2 , . . . , p K tais que

prprios asso iados a


dentes. Sejam

1 u1 + 2 u2 + + p up = 0E .

(6.1)

Ento,

(1 u1 + 2 u2 + + p up ) = (0E )

1 (u1 ) + 2 (u2 ) + + p (up ) = 0E


e, omo

u1 , u2 , . . . , up

so ve tores prprios, tem-se:

1 (1 u1 ) + 2 (2 u2 ) + + p (p up ) = 0E .
Multipli ando ambos os membros da igualdade (6.1) por

(6.2)

obtm-se:

1 1 u1 + 2 1 u2 + + p 1 up = 0E .
Subtraindo membro a membro (6.2) a (6.3), resulta que:

2 (2 1 )u2 + + p (p 1 )up = 0E ,

(6.3)

6.2 Endomorsmos diagonalizveis

152

p1 ve tores prprios asso iados


p 1 valores prprios distintos; donde, por hiptese de induo, estes ve tores

ou seja, tem-se uma ombinao linear nula de


a

so linearmente independentes e, portanto,

2 (2 1 ) = = p (p 1 ) = 0K .
Como, para todo

i {2, . . . , p}, i 6= 1 ,

ento

2 = = p = 0K .
2 , . . . , p por 0K , vem 1 u1 = 0E , ou seja,
u1 6= 0E , 1 = 0K . Logo u1 , u2 , . . . , up so linearmente independentes.

Substituindo em (6.1) os es alares


omo

Corolrio 6.21.
de

Seja E um espao ve torial sobre K e seja um endomorsmo


E . Sejam ainda 1 , 2 , . . . , p K valores prprios de , distintos dois a dois.

Ento,

U1 + U2 + + Up
uma soma dire ta.
Demonstrao. Para se provar que a soma dire ta prove-se que qualquer ele-

v U1 +U2 + +Up se es reve de forma ni a omo soma de elemenU1 , U2 ,. . . ,Up . Seja v U1 + U2 + + Up .

mento

tos dos subespaos ve toriais


Suponha-se que

v = v1 + v2 + + vp ,

om

vi Ui ,

para todo

i {1, . . . , p}

v = u1 + u2 + + up ,

om

ui Ui ,

para todo

i {1, . . . , p}.

Assim,

0E = v v = (v1 u1 ) + (v2 u2 ) + + (vp up ).


Note-se que
ve torial de

vi ui Ui , para
E . Repare-se que se

todo

i {1, . . . , p},

pois

Ui

um subespao

tem uma ombinao linear nula dos ve tores

referidos om es alares no todos nulos, o que impli aria que esses ve tores fossem linearmente dependentes. Como a valores prprios distintos orrespondem
ve tores prprios linearmente independentes, o ve tor
tor prprio asso iado a

i ;

i {1, . . . , p}.

donde

vi ui = 0E ,

vi ui no pode ser ve vi = ui , para todo

ou seja,

Assim, note-se que no aso em que

E = U1 U2 Up
: basta onU1 , U2 , . . . , Up .
Suponha-se que B = (e1 , e2 , . . . , en ) uma base de E onstituda por ve tores
prprios de . Suponha-se que ei um ve tor prprio asso iado ao valor prprio
ento existe uma base de

onstituda por ve tores prprios de

siderar a unio das bases das vrias par elas

6.2 Endomorsmos diagonalizveis


i ,

para todo

153

i {1, . . . , n}. Note-se que 1 , 2 , . . . , n

no so ne essariamente

distintos. Ento:

(e1 ) = 1 e1 = (1 , 0, 0, . . . , 0, 0)B
(e2 ) = 2 e2 = (0, 2 , 0, . . . , 0, 0)B
.
.
.

(en ) = n en = (0, 0, 0, . . . , 0, n )B ,
Assim,

1
0

M (; B, B) = .
..
0

0
2
.
.
.

..

Deste modo apresenta-se a seguinte denio:

Denio 6.22.
de

E.

Seja

Diz-se que

ve tores prprios de

um espao ve torial sobre

diagonalizvel

0
0

. .
.
.
n
K e seja um endomorsmo
E formada por

se existe uma base de

Atendendo ao que foi visto anteriormente pode es rever-se o seguinte resultado:

Teorema 6.23.

Seja

E . Ento
M (; B, B) uma

de

Demonstrao.

um espao ve torial sobre

Ke

seja

um endomorsmo

diagonalizvel se e s se existe uma base


matriz diagonal.

()

de

Veja-se o que foi es rito antes da denio 6.22.

B = (e1 , e2 , . . . , en ) de E tal

1 0 0
0 2 0

M (; B, B) = diag(1 , 2 , . . . , n ) = .
.
. .
..
.
.
..
.
.
.
0
0 n

()

tal que

Suponha-se que existe uma base

que

(ei ) = (0, . . . , 0, i , 0, . . . , 0)B , onde i o upa a i-sima po(ei ) = i ei , para todo i {1, . . . , n}. Ou seja, ei
um ve tor prprio de asso iado ao valor prprio i , para todo i {1, . . . , n}.
Logo B uma base de E formada por ve tores prprios de .

Isso signi a que


sio do

n-uplo,

ou seja,

Observao 6.24.

Seja

um endomorsmo de
base

de

E.

um espao ve torial sobre

Suponha-se que

formada por

de dimenso

e seja

diagonalizvel. Ento existe uma


ve tores prprios de . Seja ainda B uma outra

6.2 Endomorsmos diagonalizveis


base de

tal que

A = M (; B , B ).

154

Ento, esquemati amente, tem-se:

A
E

(B )

(B )

idE

(B)

P 1

idE

(B)
E

E
D
Logo

onde
tos).

1
0

D = M (; B, B) = P 1 AP = .
..
0

1 , 2 , . . . , n K

Note-se que

onde

Xi

so valores prprios de

P = M (B, B ),

.
.
.

..

0
0

.
.
.
n

(no ne essariamente distin-

ou seja,


P = X1


Xn .

B , do i-simo
prprio i , para

a i-sima oluna onstituda pelas oordenadas, na base

B , que um ve tor
i {1, . . . , n}. Note-se que P

ve tor da base
todo

0
2

base. A esta matriz hama-se

prprio asso iado ao valor

invertvel pois uma matriz mudana de

matriz diagonalizante de

A.

O prximo resultado forne e uma ondio ne essria e su iente para um


endomorsmo ser diagonalizvel.

Teorema 6.25.

Seja

um endomorsmo de

Ento,

E
E.

K de dimenso n e seja
1 , 2 , . . . , p K os valores prprios de

um espao ve torial sobre


Sejam ainda

diagonalizvel se e s se

mg (1 ) + mg (2 ) + + mg (p ) = n.

Exemplo 6.26.

Averige-se se os seguintes endomorsmos so diagonalizveis

e, em aso armativo, es reva-se a matriz diagonal que o representa relativamente a uma erta base do espao (formada por ve tores prprios desse mesmo
endomorsmo) e a matriz diagonalizante utilizada.

um endomorsmo de R2 tal que (x, y) = (x + y, 3x y), para


(x, y) R2 . Note-se que (1, 0) = (1, 3) e (0, 1) = (1, 1). Donde,


1
1
A = M (; BR2 , BR2 ) =
.
3 1

1. Seja

todo

6.2 Endomorsmos diagonalizveis

155

Determinem-se os valores prprios de


1
|A I2 | =
3

Ora



1

1

= (1 )(1 ) 3

= 2 4 = ( + 2)( 2).

Assim,

tem dois valores prprios

= 2

= 2.

Sabe-se que

ma (2) = 1 mg (2) = 1
ma (2) = 1 mg (2) = 1.
Assim, omo

mg (2) + mg (2) = 2 = dim R2 ,

pelo Teorema 6.25,

diagonalizvel.
Determine-se agora uma base de
Para

= 2,

R2

formada por ve tores prprios de

determine-se o subespao prprio asso iado. Pretende-se as(x, y) R2 tais que

sim en ontrar os ve tores

(A 2I2 )

x
y

0
0

0
0

ou seja, en ontrar a soluo do sistema

1
3



1
3

x
y

Resolvendo o sistema orrespondente, obtm-se

U2 = {(x, x) : x R} = h(1, 1)i.


Re orde-se que
Para

= 2,

a matriz de

em relao base anni a.

al ule-se o subespao prprio asso iado. Analogamente,


(x, y) R2 tais que

pretende-se en ontrar os ve tores

(A + 2I2 )

x
y

0
0

0
0

ou seja, en ontrar a soluo do sistema

3
3

1
1



x
y

Assim,

U2 = {(x, 3x) : x R} = h(1, 3)i.


Como ve tores prprios de

asso iados a valores prprios distintos so

linearmente independentes, os ve tores

(1, 1)

(1, 3)

so linearmente

6.2 Endomorsmos diagonalizveis

156

independentes e, portanto, formam uma base de


2
uma base de R formada por ve tores prprios

R2

(justique!). Donde

B = ((1, 1), (1, 3)).


Sabe-se que

(1, 1) = 2(1, 1) = (2, 0)B

Logo

D = M (; B, B) =

(1, 3) = 2(1, 3) = (0, 2)B .




2
0
0 2

Note-se que, esquemati amente

A
R2
(BR2 )
P

R2

(BR2 )

idR2

P 1

idR2

(B)

(B)

R2

R2
D

Donde

D = P 1 AP ,

onde

um endomorsmo
(x, y) R2 .

2. Seja

Note-se que

P = M (B, BR2 ) e P 1 = M (BR2 , B),




1
1
P =
.
1 3
de

(1, 0) = (1, 2)

R2

tal que

igual a

(0, 1) = (0, 1). Logo,




1 0
A = M (; BR2 , BR2 ) =
.
2 1


1
0
|A I2 | =
2
1

2.

para todo

Determine-se os valores prprios de

Pelo que

(x, y) = (x, 2x + y),

ou seja,

Ora



= (1 )(1 ) = (1 )2 .

tem um ni o valor prprio

= 1, om multipli idade

Ento mg (1) = 1 ou mg (1) = 2. Se mg (1) = 2,


mg (1) = 1, no diagonalizvel.
Determine-se o subespao prprio asso iado a
(x, y) R2 tais que

(A 1I2 )

x
y

0
0

diagonalizvel; se

= 1.

en ontrar os ve tores

algbri a

Pretende-se assim

6.2 Endomorsmos diagonalizveis

157

ou seja, en ontrar a soluo do sistema

0
2

0
0



x
y

0
0

Assim,

e, onsequentemente,

U1 = {(0, y) : y R} = h(0, 1)i


mg (1) = 1

Exer io Resolvido 6.27.

no diagonalizvel.

um endomorsmo de R3 denido por


(x, y, z) = (x 3y + 3z, 3x 5y + 3z, 6x 6y + 4z), para todo (x, y, z) R3 .
Averige se diagonalizvel e, em aso armativo, es reva a matriz diagonal que o representa relativamente a uma erta base de R3 (formada pelos
Considere

seus ve tores prprios) e a matriz diagonalizante utilizada.


Resoluo: Note-se que

(1, 0, 0) = (1, 3, 6),


Logo,

(0, 1, 0) = (3, 5, 6)

(0, 0, 1) = (3, 3, 4).

1 3 3
A = M (; BR3 , BR3 ) = 3 5 3 .
6 6 4

Determinem-se os valores prprios de


1
3
3

5
3
|A I3 | = 3
6
6
4

Ora

= (1 )((5 )(4 ) + 18) + (3)(1)1+2 (3(4 ) 18)+


+ 3(18 6(5 ))
= (1 )(5 )(4 ) + 18(1 ) + 9(4 ) 3 18
3 18 18(5 )

= (1 )(5 )(4 ) + 18 18 + 36 9 108 + 90 + 18


= (1 )(5 )(4 ) + 9(4 )
= (4 )[(1 )(5 ) + 9]
= (4 )(2 + 4 + 4)
= (4 )( + 2)2 .

tem dois valores prprios = 2 e = 4, onde ma (2) = 2 e


ma (4) = 1. Tem-se ento que mg (4) = 1 e mg (2) = 1 ou mg (2) = 2.
Assim, diagonalizvel se e s se mg (2) = 2.
Determine-se ento o subespao prprio asso iado a = 2. Pretendem-se
os ve tores (x, y, z) R3 tais que

x
0
(A + 2I3 ) y = 0 ,
z
0
Assim,

6.2 Endomorsmos diagonalizveis

158

ou seja, en ontrar a soluo do sistema

3 3 3
x
0
3 3 3 y = 0 .
6 6 6
z
0

Assim,

U2 = {(y z, y, z) : y, z R}

= {y(1, 1, 0) + z(1, 0, 1) : y, z R}
= h(1, 1, 0), (1, 0, 1)i.

(1, 1, 0) e (1, 0, 1) so linearmente independentes, ((1, 1, 0), (1, 0, 1))


U2 e, portanto, mg (2) = 2, ou seja, diagonalizvel.
3
Para determinar uma base de R onstituda por ve tores prprios ne essrio ainda determinar o subespao prprio asso iado a = 4. Assim, para = 4,
pretende-se en ontrar os ve tores (x, y, z) R3 tais que

x
0
(A 4I3 ) y = 0 .
z
0
Como

uma base de

ou seja, en ontrar a soluo do sistema

3 3
3 9
6 6

Assim,

3
x
0
3 y = 0 .
0
z
0

U4 = {(y, y, 2y) : y R} = h(1, 1, 2)i.

Como ve tores prprios de

asso iados a valores prprios distintos so li-

nearmente independentes,

B = ((1, 1, 2), (1, 1, 0), (1, 0, 1)).


uma base de

R3 ,

D = P 1 AP ,

formada por ve tores prprios de

onde

Assim,

0
0
2
0
0 2

1 1
1
0
0
1

4
D = M (; B, B) = 0
0
1
P = M (B, BR3 ) = 1
2

P 1 = M (BR3 , B) =

1
2
21

21
3
2

1
2
21

6.2 Endomorsmos diagonalizveis


Exer io 6.28.
base

159

um endomorsmo de R3
B = ((1, 1, 1), (1, 1, 0), (1, 0, 0))

1 3 3
A = 3 5 3 .
6 6 4

Averige se

Considere

uja matriz em relao

diagonalizvel e, em aso armativo, es reva a matriz diagonal

que o representa relativamente a uma erta base do espao (formada pelos seus
ve tores prprios) e a matriz diagonalizante utilizada.
O seguinte resultado um orolrio do teorema 6.25.

Corolrio 6.29.

Seja

um endomorsmo de

E um espao ve torial sobre K de


E . Se admite n valores prprios

dimenso

e seja

distintos ento

diagonalizvel.
De fa to, omo a multipli idade algbri a de ada valor prprio
tipli idade geomtri a de ada um dos valores prprios tambm
a soma das multipli idades geomtri as dos
igual a

n,

ou seja, existe uma base de

Note-se que se

no admite

1.

1,

a mul-

Por isso,

valores prprios distintos de

formada por ve tores prprios de

valores prprios distintos nada se pode on-

luir.

Observao 6.30.

Re orde-se L(E, E)
= Mnn (K) (veja-se Teorema 5.60).
A Mnn (K), existe um endomorsmo de E e uma base B de
E tal que A = M (; B, B). Ora isso permite que todas as denies dadas neste
aptulo possam ser rees ritas para uma matriz quadrada de ordem n. Assim,
diz-se que K valor prprio de A se valor prprio de . Analogamente,
v E um ve tor prprio de A asso iado ao valor prprio se v um ve tor
prprio de asso iado ao valor prprio .
Alm disso, diz-se que A diagonalizvel se for semelhante a uma matriz
diagonal, isto , se existem matrizes quadradas de ordem n, P e D , om P
invertvel e D diagonal, tais que
Assim, dada

D = P 1 AP.
Re orde-se que matriz

hama-se matriz diagonalizante de

Do que foi dito anteriormente diz-se que, se

A.

uma matriz quadrada de

n ve tores prprios linearmente indeA, tem por olunas as oordenadas


na base B dos ve tores prprios de A linearmente independentes. Se P uma
matriz diagonalizante de A e D = P 1 AP , ento os elementos diagonais de D
so os valores prprios de A orrespondentes s olunas de P .
ordem

n, A

diagonalizvel se e s se tem

pendentes. Uma matriz

Observao 6.31.

diagonalizante de

Note-se que se duas matrizes tm o mesmo polinmio a-

ra tersti o nada se pode on luir quanto semelhana entre elas. Por exemplo,
onsiderem-se as matrizes

A=

1 1
0 1

I2 =


1 0
.
0 1

6.2 Endomorsmos diagonalizveis

160

f il veri ar que tm o mesmo polinmio ara tersti o,

(1 )2

(verique!)

e, no entanto, no so semelhantes. De fa to, se A e I2 fossem semelhantes,


1
existia S M22 (R) invertvel tal que A = S
I2 S = I2 , o que absurdo! Logo

I2

no so semelhantes.

Exer io 6.32.

Verique se as matrizes

2
A= 0
0

so semelhantes.

1
0
1 1
2
4

Exer io Resolvido 6.33.


morsmo

de

R2

2
C = 0
0

0 0
3 0
0 2

A que representa
B = ((1, 1), (1, 2)) de R2 :


1 3
A=
2 2

Considere a matriz

um endo-

em relao base

1. Cal ule os valores prprios de

2. Determine o subespao prprio asso iado ao valor prprio de maior valor


absoluto.
3. Indique, aso exista, uma matriz diagonal semelhante a

A,

justi ando a

existn ia dessa matriz.


Resoluo:

1. Os valores prprios de

en ontram-se determinando as solues da sua

equao ara tersti a. Ora,

A I2 =
Assim,

det(AI) = 0 se e s

se

1
3
2
2

(1)(2)6 = 0, o que equivalente


2 3 4 = 0, ou seja, = 1 ou

a determinar as solues da equao

= 4.
Logo, os valores prprios de
2. Como

|4| > | 1|,

ou seja,

so

4.

o valor prprio de maior valor abso-

luto, o subespao prprio pedido o subespao prprio asso iado a


denio:

U4 = {(x, y) R2 : (x, y) = 4(x, y)}.


Tem-se ento:

(x, y) = 4(x, y) (A 4I2 )X = 0,

4.

Por

6.2 Endomorsmos diagonalizveis


X

onde

a matriz oluna das oordenadas de

ampliada do sistema :

161

A 4I2

Assim,

3
3 0
2 2 0

(x, y)

na base

L2 = L2 + 3 L1

B.

3 3
0 0

A matriz

0
0

U4 = {(x, x)B : x R}

= {x(1, 1) + x(1, 2) : x R}
= {(0, 3x) : x R} = h(0, 3)i

3. Repare-se que existir uma matriz diagonal semelhante a

o mesmo

que existir uma matriz diagonal que representa o mesmo endomorsmo

(re orde-se que matrizes que representam o mesmo endomorsmo so

semelhantes). O que signi a veri ar que

diagonalizvel. Pelo que

j foi dito, basta veri ar se a soma das multipli idades geomtri as dos
2
valores prprios de igual dimenso de R .
Ora,

ma (1) = 1 mg (1) = 1
ma (4) = 1 mg (4) = 1.

2 = dim R2 = mg (1) + mg (4). Logo diagonalizvel, o que


2
signi a que existe uma base de R para a qual a matriz de diagonal.

Ento,

E essa base a base formada por ve tores prprios de

linearmente inde-

pendentes e a matriz a matriz diagonal formada pelos valores prprios


de

Note-se que

U1 = {(x, y) R2 : (x, y) = (x, y)}.


Prove que

U1




3
=
y, y
:yR
2
B


3
= y(1, 1) + y(1, 2) : y R
2



5 1
=
y, y : y R = h(5, 1)i
2 2

Assim,

D = M (; B , B ) =
om

B = ((0, 3), (5, 1)).

Note-se que

4
0
0 1

(0, 3) = a(1, 1) + b(1, 2) a = 1 b = 1


(5, 1) = a(1, 1) + b(1, 2) a = 3 b = 2.

6.2 Endomorsmos diagonalizveis

162

Donde

P = M (B , B) =

1
1

3
2

Esquemati amente,

A
R2

R2
(B)
P

(B)

idR2

P 1

idR2

(B )
R2

(B )

D
Con lui-se assim que

D = P 1 AP ,

Exer ios 6.34.

1. Considere o endomorsmo

ou seja,

a matriz diagonalizante de

de

R3

A.

denido por

(x, y, z) = (x, 2y + z, z),


(x, y, z) R3 . Verique
M (; B, B) uma matriz diagonal
para todo

se existe uma base

de

R3

tal que

e, em aso armativo, indique essa

base.
2. Seja

um endomorsmo de

R3

tal que

onde

B R3

valores de

1
M (; BR3 , BR3 ) k + 1
0
a base anni a de

R3

k
1
1

0
1
k1

um parmetro real. Indique os

para os quais existe um ve tor no nulo

(a, b, c) = (a, b, c).

(a, b, c)

tal que

7. Produto interno

7.1 Denio e exemplos

164

7.1 Denio e exemplos


Nesta se o apenas se iro onsiderar espaos ve toriais reais, isto , espaos
ve toriais sobre

Denio 7.1.

em

R.
Seja

um espao ve torial real. Chama-se

a qualquer apli ao

priedades:

: E E R

produto interno

que satisfaz as seguintes pro-

(a) linearidade relativamente ao primeiro argumento:


, R

e quaisquer

para quaisquer

u, u , v E ,

(u + u , v) = (u, v) + (u , v);

(b) linearidade relativamente ao segundo argumento:


, R

e quaisquer

para quaisquer

u, v, v E ,

(u, v + v ) = (u, v) + (u, v );

( ) simetria: para

quaisquer

u, v E ,
(u, v) = (v, u);

(d) denida positiva:


(i) (u, u) 0, para qualquer u E ;
(ii) se (u, u) = 0 ento u = 0E .
Resumindo, uma apli ao de

EE

em

um produto interno em

se

for bilinear (isto , linear relativamente ao primeiro e segundo argumentos),


simtri a e denida positiva.

Observao 7.2. Observe-se que as propriedades (a) e ( ) impli am a propriedade (b). Portanto, quando se pretende provar que uma apli ao um
produto interno, basta mostrar as propriedades (a), ( ) e (d).
Exemplos 7.3.

1. A apli ao

: R2 R2 R

((x, y), (x , y )) = xx + yy ,
um produto interno em

R2 .

para todo

De fa to,

denida por

(x, y), (x y ) R2 ,

7.1 Denio e exemplos


(a)

dados

165

(x, y), (x , y ), (z, w) R2

, R

quaisquer, tem-se

((x, y) + (x , y ), (z, w)) = ((x + x , y + y ), (z, w))


pela denio de adio e multipli ao por um es alar em

R2

= (x + x )z + (y + y )w
por denio de
= (xz + yw) + (x z + y w)
pelas propriedades da adio e multi-

R2
= ((x, y), (z, w)) + ((x , y ), (z, w))
pli ao por um es alar em

por denio de

( )

sejam

(x, y), (x , y ) R2

quaisquer, ento

((x, y), (x , y )) = xx + yy

por denio de

=xx+y y

pela omutatividade
da multipli ao em

= ((x , y ), (x, y))


Portanto,

(d)

seja

por denio de

bilinear e simtri a.

(x, y) R2

qualquer, ento:

(i) ((x, y), (x, y)) = x2 + y 2 0;


(ii) ((x, y), (x, y)) = 0 x2 + y 2 = 0 (x, y) = (0, 0).
Logo,

2. A apli ao

tambm denida positiva e portanto um produto interno.

: R2 R2 R

denida por

((x1 , x2 ), (y1 , y2 )) = x1 y1 ,
no um produto interno em

(0, 0).

Exer io 7.4.

Seja

E.
E.

((0, 1), (0, 1)) = 0

Seja

um espao ve torial real e sejam


, R+ . Mostre que

Sejam ainda

um espao ve torial real e seja

u, v E . O produto
representa-se por u v .
jam ainda

porque

(x1 , x2 ), (y1 , y2 ) R2 ,

interno entre

(0, 1) 6=

e dois produtos
+ um produto
E . Se(u, v), e

um produto interno em

o nmero real

Tambm usual es rever o produto interno entre

hu, vi.

Portanto, no denida positiva.

internos em
interno em

R2 ,

para todo

omo

u|v

ou ainda

7.2 Norma de um ve tor


Exemplos 7.5.

166

1. A apli ao de

Rn Rn

em

R,

denida por

(x1 , x2 , . . . , xn ) (y1 , y2 , . . . , yn ) = x1 y1 + x2 y2 + + xn yn =
para todos

(x1 , x2 , . . . , xn ), (y1 , y2 , . . . , yn ) Rn ,
R2 R2

xi yi ,

i=1

um produto interno
Rn .

produto interno anni o em

(prove!), ao qual se hama


2. A apli ao de

n
X

em

denida por

(x1 , x2 ) (y1 , y2 ) = x1 y1 + x1 y2 + x2 y1 + 2x2 y2 ,


para todos

Exer io 7.6.

(x1 , x2 ), (y1 , y2 ) R2 ,

um produto interno em

Mostre que a apli ao de

P2 [x] P2 [x]

em

R2

(prove!).

denida por

(a1 x2 + b1 x + c1 ) (a2 x2 + b2 x + c2 ) = a1 a2 + b1 b2 + c1 c2 ,
para todos

P2 [x].

a1 x2 + b1 x + c1 , a2 x2 + b2 x + c2 P2 [x],

Proposio 7.7.

Seja

um espao ve torial real munido de um produto in-

terno. Ento, para qualquer

u E,

u E.

Ento:

Demonstrao. Seja

um produto interno em

0E u = (0 0E ) u
= 0(0E u)

tem-se

0E u = 0.

pelas propriedades de espao ve torial


pela bilineariedade do produto interno

= 0.

7.2 Norma de um ve tor


Denio 7.8.
e seja

u E.

Seja

Chama-se

um espao ve torial real munido de um produto interno

norma de

u,

no negativo dado por

kuk =

Exemplo 7.9.
plo

e representa-se por

u u.

kuk,

ao nmero real

Rn o produto interno anni o denido


anterior. Ento, para (x1 , x2 , . . . , xn ) Rn ,
p
k(x1 , x2 , . . . , xn )k = (x1 , x2 , . . . , xn ) (x1 , x2 , . . . , xn )
q
= x21 + x22 + + x2n
Considere em

R2 o ve tor (1, 2)
. A norma de (1, 2)

de R2 k(1, 2)k =
12 + 22 = 5

Por exemplo, onsidere em


ao produto interno anni o

no exem-

em relao

7.2 Norma de um ve tor

167

Note-se que o on eito de norma depende do produto interno denido no


espao ve torial.

Exemplo 7.10.

Considere em

R2 ,

o produto interno

(x1 , x2 ) (y1 , y2 ) = x1 y1 + x1 y2 + x2 y1 + 2x2 y2 .


A norma de

(1, 2)

em relao a este produto interno

(1, 2) (1, 2) = 1 + 2 + 2 + 8 = 13.

k(1, 2)k =

Teorema 7.11.
(a) kuk = 0

E um espao ve torial real munido de um produto inu, v E e R so vlidas as seguintes propriedades:

. Seja

terno. Para quaisquer

u = 0E ;

se e s se

(b) kuk = || kuk;

( ) Desigualdade de S hwarz: |u v| kuk kvk;

(d) |u v| = kuk kvk

se e s se

so linearmente dependentes;

(e) Desigualdade triangular: ku + vk kuk + kvk;


(f) ku + vk = kuk + kvk se e

s se um dos ve tores se obtm do outro atravs

da multipli ao deste por um es alar no negativo.

(a). Seja u E .
kuk = 0; ento uu = 0. Pelas propriedades do produto

Demonstrao. Prove-se
() Suponha-se que

interno,

u = 0E .

() Suponha-se agora que

u = 0E ;

Prove-se

(b). Sejam u E

p
(u) (u)
p
= 2 (u u)

= 2 u u
= || kuk.

kuk =

Prove-se

( ).

R.

ento

uu=0

e, portanto,

kuk = 0.

Ento

por denio de norma


pela bilinearidade do produto interno

Note-se primeiro que se

u = 0E ou v = 0E , a desigualdade
u 6= 0E e v 6= 0E e seja R.

trivialmente satisfeita. Suponha-se ento que

Como o produto interno uma apli ao denida positiva ento

(u + v) (u + v) 0,
ou seja, pela bilinearidade do produto interno,

2 (u u) + (u v) + (v u) + v v 0.

7.2 Norma de um ve tor

168

Como o produto interno tambm simtri o, obtm-se

2 kuk2 + 2(u v) + kvk2 0,


para qualquer

R.

Considerando

=
obtm-se

uv
,
kuk2

(u v)2
(u v)2
2
kuk

2
+ kvk2 0
kuk4
kuk2

o que equivalente a

(u v)2 kuk2 kvk2 .

Portanto,

|u v| kuk kvk.
Prove-se

(d).

() Se

u = 0E ento laramente que u e v so linearmente dependentes


6 0E e que
e tambm se tem |u v| = kuk kvk. Suponha-se ento que u =
|u v| = kuk kvk. Ento (u v)2 = kuk2 kvk2 , donde
(u v)2
= kvk2
kuk2
e, portanto,

Como

tem-se

(u v)2
kvk2 = 0.
kuk2

2
uv

(u v)2
2

kvk =
u + v
,
kuk2
kuk2

Pela propriedade

(a),

(justique!)



uv


= 0.
u
+
v
kuk2

tem-se ne essariamente que

uv
u + v = 0E .
kuk2

Portanto,

v=

uv
u,
kuk2

v um mltiplo de u, o que impli a que u e v so linearmente dependentes.


u e v so linearmente dependentes. Sem perda
generalidade, pode supor-se que v = u, om R. Ento

isto ,
de

() Suponha-se agora que

|u v| = |u (u)| = || kuk2.

7.3 ngulo entre ve tores

169

Por outro lado,

kuk kvk = kuk kuk = || kuk2 .


Logo,

|u v| = kuk kvk.

Prove-se

(e). Sejam u, v E . Vai-se provar que


ku + vk2 (kuk + kvk)2 .

Usando a denio de norma e a desigualdade de S hwarz, tem-se

ku + vk2 = (u + v) (u + v)
=uu+uv+vu+vv
= kuk2 + 2(u v) + kvk2

kuk2 + 2kuk kvk + kvk2


2

= (kuk + kvk) .

Portanto,

ku + vk kuk + kvk.

Prove-se

(f). Pela demonstrao da propriedade anterior, tem-se a igualdade

ku+vk = kuk+kvk se e s se uv = kukkvk. Prove-se agora que uv = kukkvk


se e s se um dos ve tores se obtm do outro atravs da multipli ao deste por
um es alar no negativo. Observe-se que omo
Logo, pela propriedade

kuk kvk 0 ento u v = |u v|.

(d), sabe-se que |u v| = kuk kvk se e s se os ve tores

u e v so linearmente dependentes. Sem perda de generalidade, suponha-se que


v = u, om R. Como u v = kuk2 e |u v| = || kuk2, ento > 0.
Re ipro amente, se v = u, om > 0, ento laramente u v = kuk kvk.

Denio 7.12.
u
vers(u),

e seja

Seja

um espao ve torial real munido de um produto interno

um ve tor no nulo de

E.

Chama-se

ao ve tor

vers(u) =

Observao 7.13.
norma

1.

versor de

e representa-se por

1
u
kuk

O versor de um ve tor no nulo

A um ve tor de norma

u,

hama-se

sempre um ve tor de

ve tor unitrio.

7.3 ngulo entre ve tores


Sejam

u, v E

tais que

u 6= 0E

v 6= 0E .

Ento, pela desigualdade de S hwarz,

|u v| kuk kvk.
E, portanto,

kuk kvk u v kuk kvk,

7.3 ngulo entre ve tores

170

isto ,

1
Assim, existe um valor

[0, ]

uv
1.
kuk kvk

tal que

cos =

Denio 7.14.

Seja

um espao ve torial real munido de um produto interno.

Dados dois ve tores no nulos

(u, v),

representa-se por

uv
.
kuk kvk

ao valor

cos =

(u, v) = arccos

tores

(1, 3)

1. Em

(2, 1).

R2 ,

om

uv
kuk kvk

2. Considere, em

R2 ,

onsidere o produto interno anni o e os ve -

Determine-se

o produto interno denido por

(x1 , x2 ) (y1 , y2 ) = x1 y1 + x1 y2 + x2 y1 + 2x2 y2 .


Determine-se

((1, 3), (2, 1)).

Neste aso,

(1, 3) (2, 1)
k(1, 3)k k(2, 1)k


15

= arccos
5 10
!
3 10
= arccos
.
10

((1, 3), (2, 1)) = arccos

Exer io 7.16.

Considere, em

R3 ,

o produto interno denido por

(x1 , x2 , x3 ) (y1 , y2 , y3 ) = 2x1 y1 + x1 y2 + x2 y1 + x2 y2 + x3 y3 .


Determine

((1, 3), (2, 1)). Ora




(1, 3) (2, 1)
((1, 3), (2, 1)) = arccos
k(1, 3)k k(2, 1)k


2+3

= arccos
12 + 32 22 + 12
!
2

= arccos
= .
2
4

v,

uv
,
kuk kvk

isto ,

Exemplos 7.15.

ngulo de

v de E , hama-se
[0, ] tal que

((1, 2, 1), (1, 1, 1)).

7.3 ngulo entre ve tores

171

Teorema 7.17.

um espao ve torial real munido de um produto interno

e sejam

Seja E
u, v E \ {0E }

, R \ {0}.

(a) (u, u) = 0;

Ento:

(b) (u, v) = (v, u);


( ) (u, v) = (u, v) se e s se e
(d) (u, v) = (u, v)

se e s se

> 0.

e tm o mesmo sinal, isto ,

e tm sinais ontrrios, isto ,

< 0.

(a). Ora

Demonstrao. Prove-se

(u, u) = arccos
Prove-se

uu
kuk kuk

Provem-se

kuk2
kuk2

= arccos 1 = 0.

uv
kuk kvk

= arccos

vu
kvk kuk

= (v, u).

( ) e (d). Note-se que u, v E \ {0E }. Logo,

(u, v) = arccos

= arccos

(b). Ora, pela simetria do produto interno,

(u, v) = arccos

E, portanto,

(u) (v)
kuk kvk

= arccos

()(u v)
|| kuk kvk

> 0 || =


uv
= (u, v),
(u, v) = arccos
kuk kvk

< 0 || =


(u, v) = arccos

uv
kuk kvk

= (u, v).

Exer io Resolvido 7.18.


espao ve torial real

ei ei = 1
Sejam

u = e1 + e2

Seja B = (e1 , e2 , e3 ) uma base ordenada de um


munido de um produto interno, tal que

e
e

v = e2 2e3

(a) kuk;
Resoluo:

ei ej = 0,

para todo

i, j {1, 2, 3}

dois ve tores de

(b) kvk;

E.

Determine:

(c) (u, v).

i 6= j.

7.3 ngulo entre ve tores

172

(a) Por denio de norma e pela bilinearidade do produto interno, tem-se:

uu
p
= (e1 + e2 ) (e1 + e2 )
p
= e1 (e1 + e2 ) + e2 (e1 + e2 )

= e1 e1 + e1 e2 + e2 e1 + e2 e2

= 1 + 0 + 0 + 1 = 2.

kuk =

(b) Analogamente,

vv
p
= (e2 2e3 ) (e2 2e3 )
p
= e2 (e2 2e3 ) 2e3 (e2 2e3 )
p
= e2 e2 2(e2 e3 ) 2(e3 e2 ) + 4(e3 e3 )

= 1 0 0 + 4 = 5.

kvk =

( ) Como

u v = (e1 + e2 ) (e2 2e3 )


= e1 (e2 2e3 ) + e2 (e2 2e3 )

= e1 e2 2(e1 e3 ) + e2 e2 2(e2 e3 )
= 0 0 + 1 0 = 1,

usando as alneas anteriores, obtm-se

(u, v) = arccos

Exer io 7.19.
e sejam

ke1 k = 2,
Para

Seja

e1 , e2 , e3 E

uv
kuk kvk

= arccos

10

= arccos

!
10
.
10

um espao ve torial real munido de um produto interno

tais que

ke2 k = ke3 k = 1,

u = e1 3e2

v = e1 + e3 ,

(a) kuk;

e2 e1 = 0,

(e1 , e3 ) =

(e2 , e3 ) =

determine:

(b) kvk;

(c) (u, v).

.
2

7.4 Ve tores ortogonais

173

7.4 Ve tores ortogonais


E um espao ve torial real munido de
v so ve tores de E no nulos, ento

Seja

um produto interno. Note-se que se

uv
= 0 cos ((u, v)) = 0 (u, v) = .
kuk kvk
2

uv =0

Apresenta-se ento a seguinte denio.

Denio 7.20.

Seja

u, v E .
u v = 0.

um espao ve torial real munido de um produto interno

ortogonal a

v,

e sejam

Diz-se que

e representa-se por

Exemplo 7.21.

Considere, no espao ve torial real

R2 ,

u v,

(1, 0)

os ve tores

se

(0, 1).
(a) Relativamente ao produto interno anni o,

(1, 0)

(0, 1)

so ortogonais.

(1, 0) (0, 1) = 0,

ou seja,

(b) Relativamente ao produto interno denido por

(x1 , x2 ) (y1 , y2 ) = x1 y1 + x1 y2 + x2 y1 + 2x2 y2 ;


tem-se

(1, 0) (0, 1) = 1,

isto ,

(1, 0)

(0, 1)

no so ortogonais.

Note-se que, de a ordo om o exemplo anterior, dois ve tores podem ser


ortogonais em relao a um produto interno e no serem ortogonais em relao
a outro.

Exer io 7.22.

Considere em

R3

o seguinte produto interno:

(x1 , x2 , x3 ) (y1 , y2 , y3 ) = 2x1 y1 x2 y1 x1 y2 + x2 y2 + x3 y3 .


Verique se os ve tores

u = (1, 1, 1), v = (1, 1, 1) e w = (2, 1, 1) so

ortogonais

dois a dois.

Teorema 7.23.
(a)

se

uv

E um espao
u, v E ,

Seja

Ento, para todos

ento

(b) 0E u;
( ) u u

(d)

se

v u;

se e s se

uv

ento

ve torial real munido de um produto interno.

u = 0E ;

u v ,

para todo

R.

7.5 Sistema ortogonal e sistema ortonormado


Demonstrao.
Prove-se

u E.
Prove-se
Prove-se

(a)

(b).

resulta do fa to de que

Como

0E u = 0,

174

u v = v u,

para todo

u E,

para todos

ento

0E u,

u, v E .
para todo

( ). Repare-se que u u se e s se u u = 0 se e s se u = 0E .
(d). Se u v

ento

u v = 0.

Logo

(u v) = 0, para todo R.
u (v) = 0, para

Assim, usando a bilinearidade do produto interno, tem-se


todo

R.

Exer io 7.24.

u, v E

Determine para que valores do


ve tor

E munido de um produto
kuk = 1, kvk = 2 e (u, v) = 3 .
parmetro , o ve tor u + v ortogonal ao

Considere, num espao ve torial real

interno, dois ve tores

tais que

2u + 3v .

7.5 Sistema ortogonal e sistema ortonormado


Denio 7.25.
Sejam ainda

Seja E um espao ve torial real munido de um produto interno.


v1 , v2 , . . . , vk E . Diz-se que os ve tores v1 , v2 , . . . , vk formam um

sistema ortogonal se ada um dos ve tores ortogonal a ada um dos outros,


ou seja,

vi vj = 0,

para todo

i, j {1, 2, . . . , k}

i 6= j.

v1 , v2 , . . . , vk forem unitrios (ou normados), isto ,


i {1, 2, . . . , k}, diz-se que esses ve tores formam um sistema ortonormado, ou seja, v1 , v2 , . . . , vk formam um sistema ortonormado
se, para todo i, j {1, 2, . . . , k},

0 se i 6= j
vi vj =
1 se i = j
Se, alm disso, os ve tores

kvi k = 1,

para todo

Exemplos 7.26.

R3 munido do produto interno anni o, os ve tores


u = (1, 0, 1), v = (2, 0, 2) e w = (0, 5, 0) formam um sistema ortogonal.
(a) Em

De fa to, verique que

u v = u w = v w = 0.
Mas no

2 6= 1.
(b) Em

Rn

formam um sistema ortonormado porque, por exemplo,

munido do produto interno anni o, a base anni a de

stitui um sistema ortonormado.

kuk =

Rn

on-

7.6 Base ortogonal e base ortonormada


Exer io 7.27.

No espao ve torial real

R2 ,

175
onsidere o seguinte produto in-

terno:

(x1 , x2 ) (y1 , y2 ) = x1 y1 + 2x1 y2 + 2x2 y1 + 5x2 y2 .


u = (2, 1)

Mostre que os ve tores


mado.

v = (1, 0)

formam um sistema ortonor-

Teorema 7.28.

Seja E um espao ve torial real munido de um produto interno.


v1 , v2 , . . . , vk E no nulos. Se v1 , v2 , . . . , vk formam um sistema
ento v1 , v2 , . . . , vk so linearmente independentes.

Sejam ainda
ortogonal,

Demonstrao. Sejam

1 , 2 , . . . , k R

tais que

1 v1 + 2 v2 + + k vk = 0E .
i {1, . . . , k}

Ento, para ada

tem-se

(1 v1 + 2 v2 + + k vk ) vi = 0E vi ,
ou seja, pela bilinearidade do produto interno,

1 (v1 vi )+ +i1 (vi1 vi )+i (vi vi )+i+1 (vi+1 vi )+ +k (vk vi ) = 0


v1 , v2 , . . . , vk formam um sistema ortogonal, vj vi = 0 para i 6= j . Logo,
i (vi vi ) = 0. Por outro lado, omo o produto interno denido
positivo, vi vi 6= 0, pois vi 6= 0E . Donde i = 0. Con lui-se ento que 1 =
2 = = k = 0, ou seja, que v1 , v2 , . . . , vk so linearmente independentes.

Como

obtm-se

7.6 Base ortogonal e base ortonormada


Denio 7.29.

A um espao ve torial real de dimenso nita munido de um

produto interno hama-se

Denio 7.30.

espao eu lidiano.

n e seja B =
u1 , u2 , . . . , un formam
um sistema ortogonal diz-se que B uma base ortogonal de E .
Se u1 , u2 , . . . , un formam um sistema ortonormado diz-se que B uma base
ortonormada de E .
(u1 , u2 , . . . , un )

Seja

um espao eu lidiano de dimenso

uma base ordenada de

Exemplo 7.31.

E.

Se os ve tores

Considere a base anni a de

R2 , BR2 = ((1, 0), (0, 1)).

R2 est munido do produto interno anni o, ento (1, 0) (0, 1) = 0,


k(1, 0)k = 1 e k(0, 1)k = 1, logo BR2 uma base ortonormada.

(a) se

7.6 Base ortogonal e base ortonormada


(b) se

R2

176

est munido do produto interno denido por

(x1 , x2 ) (y1 , y2 ) = x1 y1 + x1 y2 + x2 y1 + 2x2 y2 ,


ento (1, 0) (0, 1) = 1 6= 0 e, portanto, BR2 no uma base ortogonal de
R2 e, onsequentemente, tambm no uma base ortonormada de R2 .

Exer io 7.32.

Seja

um espao eu lidiano tal que

base ortogonal. Seja ainda

u = e1 + e2 .

Mostre que

B = (e1 , e2 )

uma sua

kuk2 = ke1 k2 + ke2 k2 .


(Observe-se que este resultado o Teorema de Pitgoras para

E = R2

munido

do produto interno anni o.)

Exer io 7.33.
U =< u >.

Seja

um espao eu lidiano,

Determine as bases ortonormadas de

u um
U.

ve tor no nulo de

7.6.1 Mtodo de ortonormalizao de Gram-S hmidt


Nesta se o vai-se mostrar que um espao eu lidiano

admite sempre uma

base ortonormada, provando-se que a apli ao do algoritmo seguinte, denominado

mtodo de ortonormalizao de Gram-S hmidt,

a uma base de

qualquer produz uma sua base ortonormada.

Algoritmo 7.34. Mtodo de ortonormalizao de Gram-S hmidt

E um espao eu lidiano e
E . Aplique-se os n passos
w1 , w2 , . . . , wn .
Passo 1: w1 = vers(e1 ) = e1 .
ke1 k
Seja

nada de

seja

B = (e1 , e2 , . . . , en ) uma base ordeB , onstruindo-se os ve tores

seguintes a

Passo 2:

(a) z2 = e2 (e2 w1 )w1 .


(b) w2 = vers(z2 ) =

z2
.
kz2 k

Em geral, apli a-se o passo i, ao onjunto ordenado

(w1 , w2 , . . . , wi1 , ei , . . . , en )
obtido no nal do passo

Passo i:

i 1.

(a) zi = ei (ei w1 )w1 (ei w2 )w2 (ei wi1 )wi1 ;

7.6 Base ortogonal e base ortonormada


(b) wi = vers(zi ) =
No nal do passo

177

zi
kzi k ,

obtm-se o onjunto ordenado

(w1 , w2 , . . . , wn ),

que o

resultado do algoritmo.
O teorema seguinte mostra que a apli ao do mtodo de ortonormalizao
de Gram-S hmidt a uma base qualquer de um espao eu lidiano produz uma
base ortonormada desse espao.

Teorema 7.35.
ordenada de

ortonormalizao

E um espao eu lidiano, B = (e1 , . . . , en ) uma base


B = (w1 , . . . , wn ) o resultado da apli ao do mtodo de

de Gram-S hmidt a B . Ento B uma base ortonormada de

Sejam

E.
Demonstrao. A prova faz-se mostrando que em ada passo, o algoritmo onstri uma base de

por substituio de um ve tor na base onstruda no passo

anterior, o qual forma um sistema ortonormado juntamente om os ve tores


introduzidos nos passos anteriores.

||w1 || = 1, isto , w1 forma um sistema ortonormado.


(w1 , e2 , . . . , en ) uma base de E (prove!).
Prove-se agora que (w1 , w2 , e3 , . . . , en ) obtido no passo 2, uma base de E
em que w1 e w2 formam um sistema ortonormado. Como e2 e w1 so linearmente
independentes, z2 = e2 (e2 w1 )w1 um ve tor no nulo de E . Alm disso,
z2 w1 , pois
Note-se que no passo 1,

Alm disso,

z2 w1 = (e2 (e2 w1 )w1 ) w1

= e2 w1 (e2 w1 )(w1 w1 )
= e2 w1 (e2 w1 )kw1 k
=0

pela bilinearidade do produto interno


porque
porque

w1 w1 = kw1 k2
kw1 k = 1.

Logo pela bilinearidade do produto interno,

w2 w1 =

z2
1
w1 =
(z2 w1 ) = 0.
||z2 ||
||z2 ||

Assim w1 , w2 formam um sistema ortogonal e, omo ||w2 || = 1


vers(z2 ), obtm-se que w1 , w2 formam um sistema ortonormado.

pois

w2 =

Para provar o aso geral, suponha-se que no m do passo i-1 obtm-se a base

E , (w1 , w2 , . . . , wi1 , ei , . . . , en ), onde w1 , w2 , . . . , wi1 formam um sistema


ortonormado e prove-se que (w1 , w2 , . . . , wi1 , wi , ei+1 . . . , en ) obtido no nal
do passo i uma base de E em que w1 , w2 , . . . , wi1 , wi formam um sistema
ortonormado. Como ei , w1 , w2 , . . . , wi1 so linearmente independentes, zi um
ve tor no nulo de E . Alm disso, para j {1, . . . , i 1},

de

zi wj = [ei (ei w1 )w1 (ei wj1 )wj1 (ei wj )wj

(ei wj+1 )wj+1 (ei wi1 )wi1 ] wj


= ei wj (ei w1 )(w1 wj ) (ei wj1 )(wj1 wj )

(ei wj )(wj wj ) (ei wj+1 )(wj+1 wj ) (ei wi1 )(wi1 wj ),

7.6 Base ortogonal e base ortonormada

178

pela bilineariedade do produto interno, e portanto, zi wj = 0, porque w wj =


0 se 6= j e wj wj = ||wj ||2 = 1. Logo wi = ||zzii || tal que wi wj ,

j {1, . . . , i 1}, e kwi k = 1. Con lui-se assim que w1 , w2 , . . . , wi1 , wi

formam

um sistema ortonormado. Fa ilmente se v que o onjunto ordenado

(w1 , w2 , . . . , wi1 , wi , ei+1 , . . . , en )


uma base de

(prove!).

Deste modo, no m do passo n, o mtodo de ortonormalizao de GramS hmidt produz uma base ortonormada,

Corolrio 7.36.

Seja

(w1 , w2 , . . . , wn ),

de

E.

um espao eu lidiano no trivial. Ento

admite pelo

menos uma base ortonormada.

Exer io Resolvido 7.37.

R3 munido do
R3 , apli ando o

Considere o espao ve torial real

produto interno anni o. Obtenha uma base ortonormada de


mtodo de ortonormalizao de Gram-S hmidt base

B = ((0, 1, 0), (1, 2, 1), (0, 1, 2)).


Resoluo: Considere-se
Como

ke1 k = 1,

e1 = (0, 1, 0), e2 = (1, 2, 1)

e3 = (0, 1, 2).

ento

w1 = vers(e1 ) =

e1
= e1 = (0, 1, 0).
ke1 k

Assim

z2 = e2 (e2 w1 ) w1 = (1, 2, 1) 2 (0, 1, 0) = (1, 0, 1) .


e, portanto, omo

ento

kz2 k = k(1, 0, 1)k = 2,


!

2
2
z2
w2 = vers(z2 ) =
. Por m,
=
, 0,
kz2 k
2
2
z3

Como

kz3 k =

= e3 (e3 w1 ) w1 (e3 w2 ) w2

2
2
= (0, 1, 2) (0, 1, 0) 2
= (1, 0, 1).
, 0,
2
2

2,

ento

z3
w3 = vers(z3 ) =
=
kz3 k

R3

!
!!
2
2
2
2
,
.
, 0,
, 0,
2
2
2
2

Assim uma base ortonormada de

B =

(0, 1, 0),

!
2
2
.

, 0,
2
2

7.7 Matriz da mtri a


Exer io 7.38.

179

Para o espao ve torial real

R2

munido do produto interno

denido por

(x1 , x2 ) (y1 , y2 ) = 3x1 y1 + x1 y2 + x2 y1 + x2 y2 ,


obtenha uma base ortonormada, apli ando o mtodo de ortonormalizao de
2
Gram-S hmidt base anni a de R , BR2 = ((1, 0), (0, 1)).

7.7 Matriz da mtri a


Sejam
ainda

E um espao
u, v E e

eu lidiano e

B = (e1 , e2 , . . . , en )

u = (x1 , x2 , . . . , xn )B
as oordenadas de

v,

uma base de

Sejam

v = (y1 , y2 , . . . , yn )B

respe tivamente, em relao base

u = x1 e1 + x2 e2 + + xn en

E.

B,

isto ,

v = y1 e1 + y2 e2 + , yn en .

Ento, pela bilinearidade do produto interno,

uv

= (x1 e1 + x2 e2 + + xn en ) (y1 e1 + y2 e2 + + yn en )
= x1 y1 (e1 e1 ) + + x1 yj (e1 ej ) + + x1 yn (e1 en )
++
xi y1 (ei e1 ) + + xi yj (ei ej ) + + xi yn (ei en )
++

xn y1 (en e1 ) + + xn yj (en ej ) + + xn yn (en en )

y1
e1 e1 e1 ej e1 en
..

.
.
.
.
.
.
.

.
.
.




e
= x1 xi xn
i
j
i
n yj
i 1
.

.
.
.
.
.
.
..

.
.
.
en e1 en ej en en
yn

isto ,

onde

X =

na base

B,

x1
.
.
.

xn

Y =

respe tivamente, e

da mtri a

u v = X T GY,

y1
.
.
.

so os ve tores das oordenadas de

yn
G = [ei ej ].

A matriz

do produto interno em relao base

ei ej = ej ei , i, j = 1, . . . , n,

simtri a.

G denominada matriz
B . Observe-se que omo

tem-se que a matriz da mtri a uma matriz

7.7 Matriz da mtri a

180

Exer io Resolvido 7.39.

Considere o espao ve torial real

R3

munido do

produto interno

(x1 , x2 , x3 ) (y1 , y2 , y3 ) = 2x1 y1 + x1 y2 x1 y3 + x2 y1 + 3x2 y2 x3 y1 + x3 y3


e a base de

R3 , B = (e1 , e2 , e3 ) onde e1 = (1, 0, 0), e2 = (1, 1, 0) e e3 = (1, 1, 1).

a) Determine a matriz da mtri a

do produto interno em relao base

B.
b) Cal ule

(1, 1, 0) (2, 2, 1) usando

a matriz da mtri a

obtida na alnea

anterior.

Resoluo:
a)

e1 e1 = 2, e1 e2 = 3, e1 e3 = 0, e2 e2 = 7, e2 e3 = 2, e3 e3 = 2
(verique!). Logo, omo G simtri a, obtm-se,

2 3
0
G = 3 7 2 .
0 2 2

b)

(1, 1, 0) = (0, 1, 0)B

(2, 2, 1) = (2, 1, 1)B (verique!). Assim,

0
2

 2 3
(1, 1, 0) (2, 2, 1) = 0 1 0 3 7 2 1 = 3.
0 2 2
1

Exer io 7.40.

Sejam

E um
E.

espao eu lidiano de dimenso 3 e

(e1 , e2 , e3 )

uma base ortonormada de


a) Mostre que

(e1 + e2 , e3 , e1 e2 )

uma base de

b) Determine a matriz da mtri a do produto interno relativamente base

(e1 + e2 , e3 , e1 e2 ).
O prximo teorema mostra omo se rela ionam duas matrizes da mtri a
que representam o mesmo produto interno.

Teorema 7.41.

Sejam E um espao eu lidiano, B = (e1 , e2 , . . . , en ) e B =

(e1 , e2 , . . . , en ) duas bases de E e G e G as matrizes da mtri a do produto

interno em relao s bases B e B , respe tivamente. Ento

G = P T GP,
onde

P = M (idE ; B , B).

7.7 Matriz da mtri a


x

ve tores arbitrrios de E , X e

nas bases B e B , respe tivamente, e Y e

nas bases B e B , respe tivamente. Logo

Demonstrao. Sejam
oordenadas de
oordenadas de

x
y

181

x y = X T GY
Como

X = P X

Y = P Y ,

onde

X
Y

os ve tores das
os ve tores das

x y = X T G Y .

P = M (idE ; B , B),

obtm-se

x y = X T GY = (P X )T G(P Y ) = X T P T GP Y ,
on luindo-se assim que

X T (P T GP )Y = X T G Y .
G = P T GP , onsiderem-se i, j {1, . . . , n} e x = ei

0
0
..
.
.
.
.

i
j , e portanto,
1
e Y = 1

.
.
.
..
.
0
0

Para mostrar que

Ento

X =

que o elemento

y = ej .

ei ej = X T (P T GP )Y ,

(i, j)

da matrix

P T GP .

Po outro lado, tem-se que

ei ej = X T G Y ,

que o elemento (i, j) da matrix G . Assim, on lui-se que os elementos (i, j)


T

das matrizes P GP e G so iguais para quaisquer i, j {1, . . . , n}, ou seja,


G = P T GP .

Exemplo 7.42.

Considere-se o espao ve torial real

R3

munido do produto

interno

(x1 , x2 , x3 ) (y1 , y2 , y3 ) = 2x1 y1 + x1 y2 x1 y3 + x2 y1 + 3x2 y2 x3 y1 + x3 y3 .

2 3
0
7 2 a matriz da mtri a em relao base B =
Como G = 3
0 2 2
((1, 0, 0), (1, 1, 0), (1, 1, 1)) (ver Exer io Resolvido 7.39) tem-se que a matriz da mtri a em relao base anni ade R3 G = P T GP , onde P =
1 1 2
1 (verique!) obtm-se
M (idR3 , BR3 , B). Como P = 0 1
0 0
1

1 0 0
2 3
0
1 1 2
2 1 1
1 = 1 3 0 .
G = 1 1 0 3 7 2 0 1
2 1 1
0 2 2
0 0
1
1 0 1

7.8 Complemento ortogonal e proje es ortogonais


Sejam

182

G a matriz da mtri a em relao a uma


B = (w1 , w2 , . . . , wn ) de E . Como wi wj = 0, se i =
6 j, e
i, j {1, . . . , n}, tem-se que G = In .

um espao eu lidiano e

base ortonormada

wi wi = 1,

para

Teorema 7.43.

Seja

Demonstrao. Seja

um espao eu lidiano. A matriz da mtri a em relao

a uma base qualquer de

invertvel.

uma base de

a matriz da mtri a em relao a

Vai-se provar que G invertvel.

Seja B uma base ortonormada de

B.

E . Ento G = P T G P , onde G = In
B e P = M (idE ; B, B ). Logo, omo G

matriz da mtri a em relao base


P (e portanto P T ) so invertveis, G tambm o .

a
e

7.8 Complemento ortogonal e proje es ortogonais


Denio 7.44.
e

Seja

um espao ve torial real munido de um produto interno

um sub onjunto no vazio de

E.

O ortogonal de

X = {y E : y x x X}.

Teorema 7.45.
e

Seja

um espao ve torial real munido de um produto interno


E . Ento X um subespao ve torial de E .

um sub onjunto no vazio de

X E por denio de X .

Como 0E x x X , 0E X , e portanto, X
6= .

fe hado em relao adio e multipli ao por


Para provar que X

um es alar onsiderem-se y1 , y2 X
e , R arbitrrios e demonstre-se que

y1 + y2 X , ou seja, que (y1 + y2 ) x = 0 x X . De fa to, se x


um ve tor arbitrrio de X , tem-se que (y1 + y2 ) x = (y1 x) + (y2 x),
pela bilinearidade do produto interno, e portanto, omo y1 x = y2 x = 0,

pois y1 e y2 perten em a X , tem-se que (y1 + y2 ) x = 0, o que prova que

y1 + y2 X .

Con lui-se assim que X


um subespao ve torial de E .
Demonstrao. Note-se que

Denio 7.46.
X
X.

Seja

um espao ve torial real munido de um produto interno


E . A X hama-se o subespao ortogonal de

um sub onjunto no vazio de

Exemplo 7.47.

Considere-se o espao ve torial real

interno anni o e

X = {(1, 0)}.
X

=
=

R2

O subespao ortogonal de

munido do produto

{(x, y) R2 : (x, y) (1, 0) = 0}

{(x, y) R2 : x = 0}.

7.8 Complemento ortogonal e proje es ortogonais


Teorema 7.48.
X

(a)

se

Seja

um espao ve torial real munido de um produto interno

sub onjuntos no vazios de

X Y

ento

(b) X (X ) ;
( )

seja

E.

Ento

Y X ;

o menor subespao ve torial de

interse o de todos os subespaos de

(d)

se

X X 6=

183

ento

que ontm

que ontm

X ).

(isto ,

Ento

S a
= X ;

X X = {0E }.

(a)

Demonstrao. Para demonstrar


, suponha-se que X Y e prove-se que
Y X . Seja z Y arbitrrio. Ento z y = 0 y Y . Logo, omo X Y

tem-se que z y = 0 y X , o que signi a que z X , on luindo-se assim


que

Y X .

(b)

Prove-se
. Seja x X arbitrrio. Ento, por denio de X , x z =

z X , o que impli a que x (X ) , on luindo-se que X (X ) .

Prove-se
. Como X S on lui-se por
que S
X . Para provar a

( )

(a)

in luso ontrria, ome e-se por observar que um ve tor arbitrrio de

per-

ten e a S se e s se for ombinao linear de ve tores de X (prove!). Considere-se


y X arbitrrio e prove-se que y S . Como y X ,

y x = 0 x X.

(7.1)

y S , tome-se um ve tor z de S arbitrrio e mostre-se


y z = 0. Como z S , existem x1 , x2 , . . . , xk X e 1 , 2 , . . . , k R tais
z = 1 x1 + 2 x2 + + k xk . Ento
Para provar que

yz

que
que

= y (1 x1 + 2 x2 + + k xk )
= 1 (y x1 ) + 2 (y x2 ) + + k (y xk ),

pela bilinearidade do produto interno. Logo, omo por (7.1), y xi = 0, i =

tem-se que y z = 0, on luindo-se assim que y S . Logo X


S.

Prove-se
. Suponha-se que X X
6= e onsidere-se x X X

arbitrrio. Ento omo x X e x X , tem-se que x x = 0, o que signi a

que x = 0E . Logo X X
= {0E }.

1, . . . , k ,

(d)

Exemplo 7.49.

Considere-se o espao ve torial real

interno anni o e

y R}

X = {(1, 0)}.

(ver Exemplo 7.47). Logo,

(X )

R2

O subespao ortogonal de

munido do produto
X X = {(0, y) :

= {(a, b) R2 : (a, b) (x, y) = 0 (x, y) X }

= {(a, b) R2 : (a, b) (0, y) = 0 y R}


= {(a, b) R2 : by = 0 y R}

= {(a, b) R2 : b = 0}.
Note-se que

X (X )

mas

X 6= (X ) .

7.9 Subespao ortogonal de um subespao ve torial


Teorema 7.50.

Sejam

um espao ve torial real munido de um produto in-

terno, v1 , v2 , . . . , vk E , S =< v1 , v2 , . . . , vk
y S se e s se yvi , i = 1, . . . , k .

>

um ve tor de

Demonstrao. Note-se que o menor subespao ve torial de

( )

onjunto de ve tores X = {v1 , v2 , . . . , vk } S . Logo, por


S = X , on luindo-se assim que y S se e s se y
se

184

E.

Ento

que ontm o

do Teorema 7.48,
X , ou seja, se e s

yvi , i = 1, . . . , k .

7.9 Subespao ortogonal de um subespao ve torial


Nesta se o vo-se onsiderar sub onjuntos de
de

Denio 7.51.
e

que so subespaos ve toriais

e os seus ortogonais.

Seja

um espao ve torial real munido de um produto interno


E . Ao subespao ortogonal de F , F , hama-se

um subespao ve torial de

omplemento ortogonal de

Exemplo 7.52.

F.

Considere o espao ve torial real

R4

munido do produto interno

4
X
1
xj yj
(x1 , x2 , x3 , x4 ) (y1 , y2 , y3 , y4 ) = (x1 y2 + x2 y1 ) +
2
j=1

F =< (1, 1, 0, 0), (1, 0, 12 , 0), (0, 2, 1, 2) >.


omplementar de F .

e o subespao ve torial
o subespao

Determine-se

Pelo Teorema 7.50 tem-se que

(a, b, c, d) F

Ou seja

(a, b, c, d) (1, 1, 0, 0) =

(a, b, c, d) (1, 0, 12 , 0)
=

(a, b, c, d) (0, 2, 1, 2)
=

b
a =
c = 3b

d =
0

0
0
0

F = {(a, b, c, d) R4 : a = b, c = 3b, d = 0}.

O teorema seguinte apli a-se a espaos ve toriais de dimenso nita.

Teorema 7.53.
E.

Sejam

Ento

(a) E = F F ;

um espao eu lidiano e

um subespao ve torial de

7.9 Subespao ortogonal de um subespao ve torial

185

(b) (F ) = F .
(a).

pois F e F
so
O ve tor 0E perten e a F F

subespaos ve toriais de E . Logo F F


6= , o que impli a (por
do Teorema

7.48) que F F
= {0E }.

Prove-se agora que E = F + F . bvio que F + F


E . Para provar que

Demonstrao. Prove-se

(a)

a in luso ontrria verdadeira onsiderem-se os seguintes asos:

Caso 1: F = {0E }. Neste aso F


= E e onsequentemente E =

Caso 2: F = E . bvio que E = F + F .

F + F .

F 6= {0E } e F 6= E . Como E tem dimenso nita e F um subespao


E , tem-se que F admite uma base. Seja B = (f1 , f2 , . . . , fk )

uma base ortonormada de F . Para provar que E F + F , onsidere-se um


ve tor x E arbitrrio. Ento
Caso 3:

ve torial no nulo de

x =
=
k
X

k
X
i=1

(x fi )fi +

x1 + x2 ,

i=1

(x fi )fi

k
X

(x fi )fi .
i=1
i=1
Como x2 ombinao linear dos ve tores da base B de F , tem-se que
x2 F . Vai-se provar que x1 F . Pelo Teorema 7.50 basta mostrar que
x1 fj , j = 1, . . . , k . Tem-se ento que, para j {1, . . . , k},
onde

x1 = x

(x fi )fi

k
X

x1 fj

x2 =

(x

k
X
i=1

x fj

(x fi )fi ) fj
k
X
i=1

(x fi )(fi fj ),

pela bilinearidade do produto interno. Como


que

fi fj = 0

i 6= j

x1 F , o que
E = F + F .

Logo
que

se

fj fj = 1

uma base ortonormada tem-se

e portanto

x1 fj = x fj x fj = 0.
impli a que

x = x2 + x1 F + F ,

on luindo-se assim

(b).


Pelo Teorema 7.48, tem-se que F (F ) , logo, F um


subespao ve torial de (F ) . Para provar que F = (F )
basta provar que

, E = F F
e omo F
tambm um subespao
dim F = dim (F ) . Por

ve torial de E , E = F
(F ) . Logo dim E = dim F + dim F e dim E =


dim F
+ dim (F ) , on luindo-se assim que dim F = dim (F ) , o que
Prove-se

(a)

prova

(b).

Pelo Teorema 7.53, tem-se que se E um espao eu lidiano e


E = F F . Assim, dado um ve tor v

e so ni os, os ve tores f1 F e f2 F
tais que v = f1 + f2 .
subespao ve torial, ento

F um seu
E existem,

7.10 Distn ia entre ve tores


Denio 7.54.
E

Sejam

um ve tor de

E.

186
F um subespao
f2 F tais que

um espao eu lidiano,

Sejam ainda

f1 F

ve torial de

v = f1 + f2 .
O ve tor

f1

designa-se proje o ortogonal de

f1 . O ve tor f2
PF (v) = f2 .

e representa-se PF (v) =

sobre F
e representa-se

R4

munido do produto interno

sobre

designa-se proje o ortogonal de

Exemplo 7.55.

Considere o espao ve torial real

(x1 , x2 , x3 , x4 ) (y1 , y2 , y3 , y4 ) =

4
X
1
(x1 y2 + x2 y1 ) +
xj yj ,
2
j=1

1
o subespao ve torial F =< (1, 1, 0, 0), (1, 0, 2 , 0), (0, 2, 1, 2) > e o ve tor x =
4

(1, 2, 3, 4) R . Determine-se PF (x) e PF (x).


1 1 1
Note-se que B = ((1, 1, 0, 0), ( 2 , 2 , 2 , 0), (0, 0, 0, 1)) uma base ortonormada de F (verique!). Assim, tem-se que

PF (1, 2, 3, 4) = ((1, 2, 3, 4) (1, 1, 0, 0))(1, 1, 0, 0) +


1 1 1
1 1 1
+((1, 2, 3, 4) ( , , , 0))( , , , 0) +
2 2 2
2 2 2
+((1, 2, 3, 4) (0, 0, 0, 1))(0, 0, 0, 1)
11 19 15
= ( , , , 4)
8 8 8
e

3 3 9
PF (1, 2, 3, 4) = (1, 2, 3, 4) PF (1, 2, 3, 4) = ( , , , 0).
8 8 8

7.10 Distn ia entre ve tores


Denio 7.56.
Dados

a, b E ,

Seja

um espao ve torial real munido de um produto interno.

dene-se a distn ia entre

b, d(a, b),

omo

d(a, b) = ||a b||.


Como visto na Se o 7.2 a norma de um ve tor depende do produto interno
onsiderado, a onte endo portanto o mesmo om a distn ia entre ve tores.

Exemplo 7.57.
(3, 4)

de

Considere-se o espao ve torial real

R2

e os ve tores

(1, 2)

R2 .

(a) Relativamente ao produto interno anni o,

d((1, 2), (3, 4)) = ||(1, 2)(3, 4)|| = ||(2, 2)|| =

(2)2 + (2)2 = 2 2.

7.10 Distn ia entre ve tores

187

(b) Relativamente ao produto interno denido por

(x1 , x2 ) (y1 , y2 ) = x1 y1 + x1 y2 + x2 y1 + 2x2 y2 ;

d((1, 2), (3, 4)) = ||(1, 2) (3, 4)|| = ||(2, 2)|| = 4 + 4 + 4 + 8 = 2 5.

Teorema 7.58.
terno e

Sejam

a, b, c E .

um espao ve torial real munido de um produto in-

Ento

(a) d(a, b) = 0 a = b;

(b) d(a, b) = d(b, a);

( ) d(a, b) d(a, c) + d(c, b).


Demonstrao. As seguintes demonstraes de orrem do Teorema 7.11.
Prove-se

(a):

d(a, b) = 0 ||a b|| = 0 a b = 0E a = b.


Prove-se

(b):

d(a, b) = ||a b|| = || (b a)|| = | 1| ||b a|| = ||b a|| = d(b, a).
Prove-se

( ): observe-se que d(a, b) = ||a b|| = ||a c + c b||. Como, pela

desigualdade triangular,

||a c + c b|| ||a c|| + ||c b||,

tem-se que

d(a, b) ||a c|| + ||c b|| = d(a, c) + d(c, b).

Bibliograa
[1 A. Monteiro, G. Pinto, C. Marques, lgebra Linear e Geometria Analti a

- Problemas e Exer ios, M Graw Hill, 2000.


[2 A. Monteiro, lgebra Linear e Geometria Analti a, M Graw Hill, 2001.
[3 W. K. Ni holson, lgebra Linear, M Graw Hill, 2006.
[4 I. Cabral, C. Perdigo, C. Saiago, lgebra Linear, Es olar Editora, 2009.
[5 A. P. Santana, J. F. Queir, Introduo lgebra Linear, Gradiva, 2010.
[6 I. Matos, Tpi os de lgebra Linear, DEETC-ISEL, 2007.
[7 L. Johnson, R. D. Riess, J. T. Arnold, Introdu tion to Linear Algebra,
Pearson Edu ation, 2009.
[8 B. Kolman, D. R. Hill, Elementary Linear Algebra, Pearson Prenti e Hall,
2008.

Das könnte Ihnen auch gefallen